.
. centre de ressources dilemmes et doutes le visage humain de mathématiques Qui sommes-nous Problème de mois activités de promotion babillard
Centrale des maths - centraledesmaths.uregina.ca
Dilemmes & doutes
« D & D »
. .
topic card  

Sujet:

angle

liste de
sujets
. .
nouvelle recherche

1020 articles trouvés pour ce sujet.
 
Page
1/1
un triangle 2011-11-07
Haelita pose la question :
La corde élastique a une longueur de 60cm au repos . Quelle est la nouvelle longueur de la corde si on l'écarte de 11 cm en la tirant par son milieu ?
Claude Tardif lui répond.
Un triangle isocèle 2009-10-08
lauwick pose la question :
ABC un triangle isocèle en A de périmètre 16 cm. De plus son aire est égale au quart de l'aire du carré construit sur sa base bc. Quelles sont les longueurs des cotés de ce trangle? Merci
Claude Tardif lui répond.
Probleme de trigonométrie 2008-12-06
Maryne pose la question :
On donne AB=15cm, AC=30,5cm et A=35°. Calcule BC à 0;01cm près. Calcule B et C à 0,01° près. Calcule l'aire du triangle à 0,01cm² près. Sachant que le triangle n'est pas rectangle. Merci
Pierre-Louis Gagnon lui répond.
Maximiser l'aire d'un triangle isocèle 2006-10-31
Flav pose la question :
ABC est un triangle isocèle en A tel que AB= 1 pour quelles valeurs de BC, l'aire de ce triangle est-elle maximale?
Claude Tardif lui répond.
problème de trigonométrie 2005-11-01
Manon pose la question :
Soit un triangle ABC, de cotes a , b et c, avec A=45° et B=60°.
Apres avoir demontre que 3a²-2b²=0 (que jai fais en utilisant la regle du sinus) et a²-2c²+2ac=0 (que jai fait en utilisant la regle du cosinus), vous montrerez que laire du triangle ABC peut sexprimer: { [ 3 + racine carre 3 ] / 12 } b² unites.

Je ne sais pas comment arriver a cette expression de laire. Je narrive pas a exprimer c² et ac en fonction de b².. et je pense que ca pourrait vraiment aider..
puisque je suis arrive a letape:
ac[(racine carre 3) / 4]
avec ac = a² + c² - b²
et a² = 2/3 b²
ms c²= 1/3 b² + ac

Claude Tardif lui répond.
Un triangle équilatéral 2002-10-27
Un eleve pose la question :
Soit ABC un triangle équilatéral construit dans le sens direct. Le point D est symétrique de A par rapport à la droite (BC), et le point E est symétrique de B par rapport au point C. L'intersection des droites (AD) et (BC)est notée H. On pose AB=a.
  1. Je dois calculer les longueurs AD et AE en fonction de a.

  2. Je dois montrer que le triangle ADE est équilatéral. J'arrive a prouver qu'il est isocèle en E mais j'aimerai trouver que AD=DE.

Claude Tardif lui répond.
Les dimensions d'un rectangle 2001-04-09
Un elévè pose la question :
1. Les dimensions d'un rectangle sont a et b Si on augmente a de 3cm et b de 2cm, l'aire du rectangle augmente de 37 cm2, mais si on diminue a de 2cm et b de 1cm, l'aire du rectangle diminue de 16cm2. Quelles sont les valeurs de a et b?

2. Quelle est la valeur numerique de l'expression (ac+bd)+(ad-bc) sachant que a+b=1 et que c+d=1? Quelles sont les regles essentielles qui me manquent pour resoudre ceci?


Claude Tardif lui répond.
Un polyedre ayant comme face 20 triangles 2000-08-02
Sonia pose la question :
J'aimerai savoir comment s'appelle un polyedre ayant comme face 20 triangles équilatéraux égaux.
Chris Fisher lui répond.
La bissectrice de l'angle ABC 2000-02-16
Laurent Gauthier pose la question :
Est-ce redondant d'ecrire "angle ABC" avec un accent circonflexe sur le B? Peut-on ou devrait-on se contenter d'ecrire soit ABC (avec l'accent), soit l'angle ABC (sans accent) ? Le contexte dans lequel ceci apparait est le syntagme "la bissectrice de l'angle ABC", c'est-a-dire que je me demande si on devrait ecrire "la bissectrice ABC " (avec accent), "la bissectrice de l'angle ABC" (sans accent) ou "la bissectrice de l'angle ABC" (avec accent).
Claude Tardif lui répond.
La longueur des segments formant un triangle 1999-10-05
Michel Provencher pose la question :
Connaissant la longueur des segments formant un triangle,comment détermine-t-on chacun de ses angles si:
  1. il S'agit d'un triangle rectangle

    Sachant que la somme des angles d'un triangle est de 180 degrés et sachant par conséquant qu'un des angle est de 90 degrés (triangle rectangle) il reste donc, 90 degrés à partager entre les 2 angles restant. Si les 2 segments formant l'angle droit sont de même longueur on obtient un angle de 45 degrés pour les angles restant soit 1/2 angle droit ce qui ne me pose évidement aucun problème. Quel relation, S'il y en a une, y a t-il entre la longueur de ces 2 segments et les angles restants.

  2. il S'agit d'un triangle quelconque

Claude Tardif lui répond.
Maths 1999-01-11
Stephane Roissard pose la question :
Soit ABC un triangle dans lequel les trois médianes sont de meme longueur. Montrer que ce triangle est quilatéral.
Jack LeSage lui répond.
Un angle solide 1998-05-06
Giol pose la question :
Qu'est ce qu'un angle solide ?? J'ai beau chercher dans mes documents, je ne trouve rien de bien convaincant sur le sujet si ce n'est une définition qui me semble bien vague et creuse ... En vous remerciant de votre attention ( puissiez vous illustrer votre réponse par un exemple, s.v.p...)

Bonjour,

Chris Fisher lui répond.

Question de trigonométrie 1997-12-11
Jean-Pierre Quesnel pose la question :
Je suis dans le désert et je parcours 1000 km à partir du point "A" jusqu'au point "B". Si je reviens au point "A" et fais une rotation de 8 degrés en faisant un autre 1000 km, quelle sera la distance en km entre les points "B" et "C".
Diane Hanson et Penny Nom lui répond.
The height of a right triangle 2021-12-15
Slash pose la question :
hi how can i find the height of a triangle from the hypotenuse and base. please go into some details with some steps because it is hard to understand. thanks
Penny Nom lui répond.
The dimensions of a rectangle 2021-09-09
Ash pose la question :
I need to know how to figure out the dimensions of a rectangle. I know that the area is 30ft and the perimeter is 26ft. I'm 15 and I have no idea what I'm supposed to do.
Harley Weston lui répond.
The dimensions of a rectangle 2020-11-02
Chie pose la question :
A rectangular piece of property has an area of 987m² and a perimeter of 136m. Find its LENGTH and WIDTH.
Penny Nom lui répond.
4sin(2x)cos(2x)+1=0 2020-07-02
Sheila pose la question :
4sin(2x)cos(2x)+1=0
Penny Nom lui répond.
A hexagonal planter 2020-05-28
Callie pose la question :
Hello, I'm having trouble cutting my angles for my 2 x 4 planter. I did exactly as I've watched and read and mine is just not turning out! Ok, so I want 6 or 8 piece hexagon. With an interior of about 10". So I understand that the boards can be any length, but its the 30 degree or 22 1/2" angles that matter. So. How do I figure how long my 2 x 4 is with 6. and another 8 pieces of 2 x 4 to make this planter and at what angle. So does the length determine the angle? Thank you in advance! i don't want to waste anymore wood! And how to figure length, angle and center circumference?
Harley Weston lui répond.
An angle i a triangle 2020-05-16
Ogunjobi pose la question :
Two goal post are 8m apart a footballer is 34 m from one post and 38m from the other within what angle must he kick the ball if he is to score
Penny Nom lui répond.
The width of a rectangle of land 2020-04-26
Teresa pose la question :
If a rectangle shape piece of land is 813 feet long. How many feet width would be 2 acres?
Penny Nom lui répond.
An isosceles triangle 2020-04-16
jo pose la question :
Find the angle between the two sides of length 5 in an isosceles triangle that has one side of length 9 and two sides of length 5.
Penny Nom lui répond.
Why is a square a rectangle but a rectangle is not a square? 2020-04-15
A parent pose la question :
Why is a square a rectangle but a rectangle is not a square? Explain your answer.
Penny Nom lui répond.
A hexagon constructed from two triangles 2020-04-13
sunny pose la question :
Triangles AEC and FDB are equilateral triangles and EA= DF= 12cm. The polygon at the center of the star is a regular hexagon. What is the area of the hexagon?
Penny Nom lui répond.
The height of an isosceles triangle 2020-02-23
Reagan pose la question :
I need to find the height of an isosceles triangle with a base of 6 and sides of 4 units. How do I find it?
Penny Nom lui répond.
Degrees, minutes and seconds 2020-02-21
Jonathan pose la question :
If a cone has an angle of 22 degrees, when i place it flat on a surface, the new resulting central angle is now at 68.69123834, but how come when i saw it on my friend it say 68 degree and 40 minutes, what is this minute?
Penny Nom lui répond.
The area of a triangle from two angles and a side 2020-02-10
Chinmoy pose la question :
How to measure the area of a triangle with two angles and length of the included side known?
Harley Weston lui répond.
The angle of a countersunk screw 2020-01-19
Barbie pose la question :
I need to be able to verify the angle used for the head of a countersunk screw.
I have the diameter of the head, diameter of the shank and height between the two. I assume it would be considered a frustum.
For example:
A standard 90 degree metric flat head screw in an M2 diameter has a head diameter of 3.65mm, the actual thread diameter is 1.98mm and the height of the head is 1.20mm.
How can I prove that it is a 90 degree angle?

Harley Weston lui répond.
What do you call a 43-sided polygon? 2020-01-06
Alniko pose la question :
What do you call a 43-sided polygon and what is its interior angle's total measure?
Penny Nom lui répond.
An exterior angle of a parallelogram 2019-11-29
A student pose la question :
one angle of a parallelogram is four times the measure of the exterior angle adjacent to it. find the interior angles
Penny Nom lui répond.
The length and width of 1 1/3 acres 2019-11-22
Ben pose la question :
I’m trading to figure out what the length and width of 1 1/3 of an acre would be in a rectangle or in a perfect square
Penny Nom lui répond.
cos (3pi/2 -x) 2019-11-20
sam pose la question :
1. express the following as a trigonometric function of angle x for cos (3pi/2 -x) the answer in the textbook is -sin x but i am unsure of the process to get the answer.
Penny Nom lui répond.
How do you find the length and the width if all you had was area? 2019-10-16
Ali pose la question :
So, How do you find the length and the width if all you had was area?
Penny Nom lui répond.
The area of a triangle in acres 2019-06-23
Karen pose la question :
if I have a triangle that measures 78 ft x 54 ft x 50 ft, how do I figure out what part of an acre that is??
Harley Weston lui répond.
A tube through a board at 45 degrees 2019-06-15
Guy pose la question :
I need to insert a tube 5/8" diameter into a board at a 45 degree angle. What size hole must I drill for the tube to fit snugly?
Harley Weston lui répond.
Do all angles have to be equal to a number? 2019-05-15
Malik pose la question :
Do all angles have to be equal to a number? By all angles I mean adjacent, vertical, supplementary, complementary.
Penny Nom lui répond.
Is every rectangle a rhombus? 2019-04-25
Danny pose la question :
Is a rectangle a rhombus??? True or False
Penny Nom lui répond.
The area of a triangle 2019-04-25
troy pose la question :
The base and the height of a right triangle are represented by the expressions below. The area of the triangle is 28 square units. Find the lengths of the base and the height of the triangle

base= x+4
height=x-6

Penny Nom lui répond.
(3d) Whether line segment intersects triangle 2019-04-13
rj pose la question :
Well hello,

Lets say I have a triangle,

Not any ordinary triangle, but a triangle that is represented in 3 dimensions. I also have a line segment.

Lets say that this triangle has points A(0,0,0), B(1,0,0), C(0,1,1)

And the line segment has points D(0,1/2,0), E(1,1/2,1)

In what way can I find if the line segment and triangle intersect? Their intersection point?

I don't prefer matrices, because they don't make any sense, but ... if that's the only way to solve this, then do what's necessary.

Thank you in advance.

Penny Nom lui répond.
An equilateral triangle 2019-04-10
ethan pose la question :
In equilateral triangle RST, R has coordinates (0, 0) and T has coordinates of (2a, 0). Find the coordinates of S in terms of a.
Penny Nom lui répond.
Internal acute angles 2019-04-02
karan pose la question :
Prove that a convex polygon cannot have more than three acute internal angles.
Penny Nom lui répond.
The area of a triangle 2019-03-19
Dylan pose la question :
The area of a triangle is 108in squared. A base and corresponding height are in the ratio 3:2. Find the length of the base and the corresponding height
Penny Nom lui répond.
A triangle of maximum area 2019-03-07
Tom pose la question :
Triangle ABC is such that AB=3cm and AC=4cm.
What is the maximum possible area of triangle ABC?

Penny Nom lui répond.
An equilateral triangle inscribed in a circle 2019-01-29
Penny pose la question :
What is the length of each side of the largest equilateral triangle that fits inside a 3 inch diameter circle?
Penny Nom lui répond.
The supplement of a complement 2019-01-25
Andrei pose la question :
The value of the supplement of the complement of 8° is ?°.
Penny Nom lui répond.
What angle s its own supplement? 2019-01-25
Ciara pose la question :
What is the angle of it's own supplement?
Penny Nom lui répond.
A median of a triangle 2019-01-24
Anna pose la question :
For triangle J(1,4) K(-3,-1) L(3,4), find an equation for the median from vertex J
Penny Nom lui répond.
A triangle made from 12 matchsticks 2019-01-20
Palesa pose la question :
How many different types of triangles can be made with a perimeter of 12 matches?
Penny Nom lui répond.
A rectangle problem 2019-01-01
ahamed pose la question :
In rectangle ABCD, angle BCD is trisected. CE,CF meet the sides AB,AD at E,F. BE=6cm, AF=2cm so, find the area of the rectangle ABCD?
Penny Nom lui répond.
cos(2x) = sin(x) 2018-12-18
Liz pose la question :
Determine all values of x in [0,2pi] that solve the equation Cos(2x)=sinx
Penny Nom lui répond.
The height of a tower 2018-12-12
Pandey pose la question :
Hello sir plz solve this question. If the shadow of a tower is found to be 10.5m longer when the sun's altitude is 45°AND60°. Find the height of the tower.
Penny Nom lui répond.
Two expressions for the perimeter of a a rectangle 2018-05-13
Slayde pose la question :
A rectangular house has one side 4 metres longer than the other. Create two different expressions for its perimeter. The expressions should be given in factorised and expanded form.
Penny Nom lui répond.
How to find the area of triangle? 2018-04-19
g.a pose la question :
how to find the area of triangle
Penny Nom lui répond.
A triangle in a cube 2018-04-16
Sohel pose la question :
A cube has an edge of 4 cm. A triangle is formed by joining the middle point of one face of the cube to the ends of a diagonal of the opposite face . What will be the area of the triangle ?
Penny Nom lui répond.
An equilateral triangle inscribed in a circle 2018-03-15
Olatundun pose la question :
An Equilateral Triangle Of Side 20cm Is Inscribed In A Circle.Calculate The Distance Of A Side Of The Triangle From the Centre Of The Circle.
Penny Nom lui répond.
The dimensions of a rectangle 2018-03-03
Mia pose la question :
A wire of length 36cm is bent to form a rectangle. Find the dimensions of the rectangle.
Penny Nom lui répond.
Is a rectangle a parallelogram? 2018-02-28
ellie pose la question :
is a rectangle a parallelogram?
Penny Nom lui répond.
A footballer angle 2018-02-14
Kim pose la question :
Two goal posts are 8m apart. A footballer is 34m from one post and 38m from the other. Within what angle must he kick the ball if he is to score a goal.
Penny Nom lui répond.
Related rates 2018-02-11
angelo pose la question :
hi admin please help me answer this question. thank you! At a certain instant of time, the angle A of a triangle ABC is 60 degrees and increasing at the rate of 5degrees per second, the side AB is 10cm and increasing at the rate of 1cm per second, and side AC is 16cm and decreasing at the rate of 1/2 cm per second. Find the rate of change of side AB?
Penny Nom lui répond.
A parallelogram, a rhombus and a rectangle 2018-02-07
Sambo pose la question :
what do parallelogram, rhombus and rectangle have in common?
Penny Nom lui répond.
Similar rectangles 2018-01-31
Kathy pose la question :
A rectangular picture frame is 14 inches long and 4 inches wide. Which dimensions could a similar picture frame have.
8 L X 21 W
35 L X 15 W
49 L X 14 W
7 L X 3 W

Penny Nom lui répond.
A range hood 2017-12-04
Chuck pose la question :
I'm building a custom range hood and can't seem to get the angles correct where the front and side panels intersect.
I saw a similar post but there's no way I can do the calculations for a Wolfram Alpha!

Here are the dimensions that I have (in inches)...

Base - Front 29 7/16" x side 19 3/16"
Top - Front 10" x side 9"
Front Panel Length (from base to top on an angle) 21"
Side Panel Length (from base to top on an angle) - 17 9/16" Vertical distance from the base to the top 14 1/4 inches.

Any help finding the bevel/miter of the two front corners where the sides meet the front panel would be greatly appreciated.

I will need the angle for the saw in degrees.

Thank you in advance!

Regards, Chuck

Harley Weston lui répond.
An expression for the area of a triangle 2017-11-19
hari pose la question :
why is area of a triangle drawn from centre of a circle i.e radii forming the sides is 1/2 *r^2 sin theta where r is radius
Penny Nom lui répond.
A triangle with sides measuring 6, 8 and 10 inches 2017-11-11
rukhsar pose la question :
A certain triangle has sides, which are, respectively, 6 inches, 8 inches, and 10 inches long. A rectangle equal in area to that of the triangle has a width of 3 inches. The Perimeter of the rectangle, expressed in inches, is
Penny Nom lui répond.
math 2017-11-11
rukhsar pose la question :
A certain triangle has sides, which are, respectively, 6 inches, 8 inches, and 10 inches long. A rectangle equal in area to that of the triangle has a width of 3 inches. The Perimeter of the rectangle, expressed in inches, is
Penny Nom lui répond.
11% of a central angle 2017-10-22
Hope pose la question :
How do you turn a percentage in to a central angle eg. 11%
Penny Nom lui répond.
An equilateral triangle inscribed in a circle 2017-09-15
sumit pose la question :
what is the area (in sq. unit) of an equilateral triangle inscribed in the circle x^2+y^2-4x-6y-23=0.
Penny Nom lui répond.
Complementary angles 2017-09-15
SM pose la question :
The measure of angle A is 60 degrees more than its complement. Find the measure of angle A.
Penny Nom lui répond.
The angle between two planes 2017-09-12
Arthi pose la question :
Hello. hope you are having a good day.

I am stuck on a maths question in which it is asking, how to find the angle between two given planes. These planes are in a 3D shape such as a prism.
How can I do this?
Please reply as soon as possible

Thank You

Penny Nom lui répond.
A rectangle and a circle 2017-08-12
Ramzan pose la question :
a diagram shows the shape of rectangular framework with length (2x+20)cm and with (y+10)cm . The outline is made of wire . If a circle with diameter 56 cm is to be made from the wire and the area of the rectangle is 420 square centimeter ,find the possible lengths and width of the rectangle
Penny Nom lui répond.
Squares and rectangles 2017-07-15
Tront pose la question :
So, there's a general rule that all squares are rectangles but not all rectangles are squares. Im trying to find a term that would describe this relationship. I've found that if all of A is B but not all B is A then I'd say that A is a subset of B, but is there a term that describes the relationship as a whole? I don't want to describe the components, I want to describe the relationship as a whole.
Penny Nom lui répond.
The maximum area of a rectangle with a given perimeter 2017-06-02
Bob pose la question :
How would I go about finding the maximum area of a rectangle given its perimeter (20m, for example)?
Penny Nom lui répond.
A circle inscribed in a 30-6-90 triangle 2017-04-07
Kameron pose la question :
i have been given a problem with a 30-60-90 triangle and a circle inscribed with a radius of 2 and was told to find the perimeter of the triangle
Penny Nom lui répond.
A circle inscribed in an isosceles triangle 2017-01-14
Sal pose la question :
There is a right isosceles triangle. Inscribed inside of it, is the largest possible circle. Ho do you find the value of the radius?

I want to find out a way of only using the rules/laws of geometry, or is that not possible.

Penny Nom lui répond.
A rectangle with a given perimeter and area 2017-01-11
Joe pose la question :
i have a math question for homework that i don't understand. can you draw a rectangle that has 18cm in perimeters and 20cm in areas. i tried but i can't.
Penny Nom lui répond.
An angle and its complement 2017-01-10
Ysa pose la question :
If the measure of an angle is twice the measure of its complement, what is the measure of the angle?
Penny Nom lui répond.
The area of a city block 2016-12-08
Miura pose la question :
A city block is three times as long as long as it is wide.if the distance around the block is 0.48 kilometers, what is the area of the block in square meters?
Penny Nom lui répond.
Is every rectangle a rhombus? 2016-11-28
Robert pose la question :
not every rectangle is a rhombus.true or false? if false please leave a short explanation. thank you:)
Penny Nom lui répond.
A circle inscribed in an equilateral triangle 2016-11-27
jo pose la question :
what is the radius of the inscribed circle of an equilateral triangle with altitude 12 units?
Penny Nom lui répond.
Radius angle and arc length 2016-11-24
pavidthra pose la question :
Length or arc 11 and angle of subtended 45.need to find a radius
Penny Nom lui répond.
The area of a rectangle 2016-11-05
Ivan pose la question :
If the vertices of a rectangle are a(1,3),b(3,-4),and c(1,-4),find it area.
Penny Nom lui répond.
Hexagonal pyramid bevel angles 2016-09-30
Peter pose la question :
I have seen your response to a similar question from Steve which Chris and Harley responded to, however I am not familiar with some of the terms. Is there a formula that I can enter my details in which will give me the specific angles I require. The item I am constructing is much larger than the one you responded to previously. Thank you for any assistance you are able to provide.
Penny Nom lui répond.
The dimensions of a rectangle 2016-08-31
Hazym pose la question :
How do I find the length(which is 15m longer than the breadth) and the breadth of a rectangle just by its perimeter which is 70 m?
Penny Nom lui répond.
The circumcentre of a triangle 2016-07-28
Raj pose la question :
If the vertices of triangle ABC are A(-4,-1) B(1,2) C(4,-3) then find coordinates of circumcentre of triangle
Penny Nom lui répond.
The dimensions of a rectangle 2016-07-25
Krunox pose la question :
A rectangle is twice as long as it is wide. Its perimeter (the sum of the lengths of its sides) is 60 cm. Find its length and width.
Penny Nom lui répond.
The number of sides of a polygon 2016-07-23
Shriya pose la question :
All the angles of a polygon are either 155° or 140°.
There are twice as many angles of 155 °or 140°.
Find the number of sides of the polygon

Penny Nom lui répond.
A congruence theorem for two right angled triangles 2016-07-17
Sayari pose la question :
Hello. Is it possible for two right angled triangles to have the same length of base and height and a different hypotenuse? If not so, then why in the congruence criteria RHS the hypotenuse is given more importance? It can also be like- 'two triangles are congruent if they are right angled and have the same base and height.' Thank You.
Penny Nom lui répond.
The difference between the ares of two rectangles 2016-06-09
Ingrid pose la question :
I am trying to help my son with an area question.
I have the answer, from the solutions, but I cannot figure out how to teach him.

Question:

Two rectangles have lengths 13cm and 19cm respectively.
Their total area is 376cm squared.
If both their widths are whole numbers, what is the difference in their areas?

I know that this is solvable once I determine the widths of the rectangles ,
but how do I go about finding that?

Thanks for your help

Chris Fisher and Harley Weston lui répond.
A trough with a triangular cross section 2016-05-21
Clarice pose la question :
A trough having an equilateral triangle end sections has sides equal to 0.4 m and 7m long.what is the volume of the liquid in the container if the depth of the water is one half the depth of the trough?
Penny Nom lui répond.
sin 2x - sin x = 0 2016-04-24
lilly pose la question :
sin 2x - sin x = 0 0
Penny Nom lui répond.
A triangle inscribed in a circle 2016-04-22
Olyana pose la question :
I am struggling with this question! Help!

So, there is a circle. In the circle, there is an equilateral triangle inscribed. Each side of the triangle is 20. There is no other info given, other than the triangle is inscribed in the circle and the sides of the triangle are 20. I am supposed to find the radius of the circle! Please help!

Penny Nom lui répond.
External and interior angles of a regular polygon 2016-04-19
pearl pose la question :
a polygon has n sides.The exterior angle is 8 times the interior angle
find the value of the interior angle
find the value of n

Penny Nom lui répond.
The area and the perimeter of a rectangle 2016-04-12
kimberly pose la question :
is there a relationship between the are and the perimeter of a rectangle? explain.
Penny Nom lui répond.
An isosceles triangle inscribed in a circle 2016-03-25
NIHAL pose la question :
A isosceles triangle is inscribed in a circle having sides 20cm,20cm,30cm. find the radius of circle
Penny Nom lui répond.
The perimeter of a rectangle 2016-03-17
Linda pose la question :
Noel correctly adds the length of three sides of a rectangle and gets 88 cm her brother Ryan correctly adds the lengths of three sides of the same rectangle and gets 80cm. What is the number of cm in the perimeter of the rectangle
Penny Nom lui répond.
The sides of a triangle 2016-03-15
Hitarth pose la question :
Why we cannot construct a triangle ofngiven sides 5cm,5cm and 10cm?
Penny Nom lui répond.
Angles 2016-03-12
Laurynn pose la question :
What are angles in general (please include the 'angle of incline')

Thank you
Laurynn

Penny Nom lui répond.
The area of a rectangle 2016-03-03
Lucas pose la question :
The area of a rectangle is given by A=x2+18x+72
-use factoring to find an expression for the dimensions of the rectangle.
-if the area of the rectangle is 7 square feet, what are the possible values of x?
-what are the dimensions of a rectangle?

Penny Nom lui répond.
The sum of the angles of a triangle 2016-02-24
Sophia pose la question :
Does every triangle add up to 180 degrees? (Such as a unique triangle)
Penny Nom lui répond.
The interior and external angles of a regular polygon 2016-02-17
percy pose la question :
a regular polygon has n sides .The size of each interior angle is eight times the size of each exterior angle .
1.find the size of each exterior angle
2.calculate the value of n

Penny Nom lui répond.
The height of a triangle 2016-02-03
Brooklyn pose la question :
How do I figure out the height of a triangle when it only tells me the base of he triangle? And vice versa.
Penny Nom lui répond.
Consecutive angles of a parallelogram 2016-01-28
Hanna pose la question :
The consecutive angles of a parallelogram measures
Penny Nom lui répond.
The perimeter of a triangular plot of land 2016-01-11
Janelle pose la question :
I need to fence off 6.5 acres. The plot is triangular shape. How many feet would I be fencing off?
Penny Nom lui répond.
The area of a rectangle 2016-01-08
ranjan pose la question :
if we make a rectangle with a 100cm of wire, then its area will be?
Penny Nom lui répond.
The angles of a triangle 2015-12-17
Faith pose la question :
Does the measure of angle determine the length of its side? For example two angles are congruent then the sides are also congruent because from my understanding the angle determine the shape of triangle.
Penny Nom lui répond.
The dimensions of a rectangle given its area and aspect ratio 2015-12-13
colum pose la question :
Can you work out the length and width of a rectangle where you know only the area and aspect ratio
Penny Nom lui répond.
The sweep angle of a video camera 2015-11-25
Franc pose la question :
a television camera is 30ft from the sideline of a basketball court 94 ft long. The camera is located 7 ft from the mid court. through what angle must it sweep in order to cover all action on the court?
Penny Nom lui répond.
The angle at the vertex of an isosceles triangle 2015-11-25
Karan pose la question :
I have been given an isosceles triangle. the Top angle is what i have to find out and the two sides adjacent to it are both 4.9 cm. i have been told that the area of the triangle is 4 cm^2. i have no idea how to work this out, any ideas?
Penny Nom lui répond.
The measure of an angle in terms of its complement 2015-11-22
Pam pose la question :
Can you please help me so I can help my daughter the equation is the measure of angle v is 4 time the measure of its complement what is the measure of angle v when the equation is 4x+x=90
Penny Nom lui répond.
The length of a rectangle 2015-09-26
Tris pose la question :
Hi,
I'm trying to figure out the length of a rectangle but i forgot how. So, the width is 5cm and the perimeter is 34cm.

Penny Nom lui répond.
The dimensions of a rectangle 2015-09-25
Erick pose la question :
A rectangular strip of land has a perimeter of 124meters and an area of 385 square meters. Find its possible dimensions
Penny Nom lui répond.
The length of a shadow 2015-08-01
maaz pose la question :
Hello
I am having trouble with this question:

Lizzie, who is 6 feet tall, stands in her driveway at night, exactly 24 feet from the base of a spotlight, and casts a shadow that is 12 feet long if her friend Hannah who is 5 feet tall decides to stand next to lizzie how long will her shadow be?

Penny Nom lui répond.
The area of an isosceles triangle 2015-06-11
Pradipta pose la question :
Find the area of an isosceles triangle having two equal sides of 20 cm each and angle between them 45 degree ? ( solve without trigonometric function).
Chris Fisher lui répond.
An equilateral triangle inscribed in a circle 2015-06-11
Casey pose la question :
I have an equilateral triangle inscribed in a circle - this triangle has been bisected to give me 2 right triangles. I know the length of the line bisecting the equilateral triangle is 36 inches. How do I figure out the circumference of the circle and the length of the sides of the equilateral triangle?
Penny Nom lui répond.
The perimeter of a rectangle 2015-05-28
Imran pose la question :
The length of a rectangle is x+3 centimetres.
The width of the rectangle is x-1 centimetres.

Find an expression in terms of x for the perimeter of the rectangle.
The perimeter of a rectangle The perimeter of a rectangle
Give your expressions in it's simplest form.
The perimeter of a rectangle The perimeter of a rectangle
Thank You.

Penny Nom lui répond.
An isosceles right triangle 2015-05-17
Ari pose la question :
In a 45-45-90 triangle find the ratio of a leg to the hypotenuse
Penny Nom lui répond.
An octagonal frame around a pool 2015-05-17
James pose la question :
I have a 20' pool and need to put a frame around it using 2by 4 what r the lengths and angle cuts
Penny Nom lui répond.
An angle of depression problem 2015-05-14
haxir pose la question :
find the height of the balloon directly above a town if the measure of angle of depression of another town 5km from the first town is 20°!6`
Penny Nom lui répond.
Folding a rectangle to preserve the aspect ratio 2015-05-09
Anna pose la question :
If you fold a rectangular piece of paper in half and the resulting rectangles have the same aspect ratio as the original rectangle, then what is the aspect ratio of the rectangles?
Penny Nom lui répond.
A triangle inscribed in a circle 2015-05-07
R2D2 pose la question :
A triangle is inscribed in a circle of radius 10. If two angles are 70 degrees and 50 degrees find the length of the side opposite the third angle?
Chris Fisher lui répond.
A square and a rectangle 2015-05-01
nick pose la question :
why is the area of a square of a given circumference greater than the area of a rectangle with the same circumference ?
Penny Nom lui répond.
An isosceles triangle inscribed in a circle 2015-03-23
Rachel pose la question :
Triangle ABC is an isosceles triangle inscribed in circle O. If each leg of the triangle is 13cm and the altitude to the base of triangle ABC is 5cm, find the radius of the circle.
Penny Nom lui répond.
The height of an equilateral triangle 2015-03-12
anna pose la question :
I am anna and i am in 7th grade.
i am trying to find the height of and equilateral triangle, all sides equaling 4 inches

Penny Nom lui répond.
Angles of elevation and depression 2015-03-08
Timmy pose la question :
Joel is walking down a street and sees a 115 ft tall building in front of him. He stops 190 feet from the base of the building at the tip of the building's shadow. Round answers to three decimal places.

A. If there was a piece of rope from the top of the building to Joel, how long would it be?
B. What is the angle of elevation from Joel to the top of the building?
C. Margaret says that she could find the angle of depression from the top of the building to Joel by subtracting the angle of elevation from 90°. Is she correct? Explain.

Penny Nom lui répond.
A man is standing on top of a building 2015-03-06
kiki pose la question :
a man of height 1.5m standing on top of a building of height 48.5m views another building across the square. he observe that the angle of depression of the bottom of the building is 40 degrees and the angle of depression of the top of the building is 5 degrees. Both buildings stand on the same level
a) calculate the distance of the man from the base of the building across the square measured along the line of sight

Penny Nom lui répond.
An isosceles triangle and an arc 2015-02-18
Sreeharsha pose la question :
The diagram shows an isosceles triangle ABC in which BC = AC = 20 cm, and angle BAC = 0.7 radians. DC is an arc of a circle, centre A. Find, correct to 1 decimal place,
(i) the area of the shaded region, [4]
(ii) the perimeter of the shaded region. [4]

Penny Nom lui répond.
The height of a building 2015-01-20
Emily pose la question :
A man 2m high observes the angle of elevation to the top of the building to be 70 degrees. And the angle of depression to the bottom of the building to be 19 degrees. How tall is the building?
Robert Dawson lui répond.
The length of a roll of paper 2015-01-12
Peggy pose la question :
roll of paper 3ft wide and roll equals 500 sq ft how long would the roll be?
Penny Nom lui répond.
An isosceles triangle 2014-12-31
ahmadmahmoud pose la question :
an isoseles triangle is such that each of the base angles is twice the vertical angle.Find the angles of the triangle
Penny Nom lui répond.
A right triangle 2014-12-13
Katie pose la question :
the shorter leg of a right triangle measures 2 feet less than the longer leg. the hypotenuse measure 10 feet. find all three sides
Penny Nom lui répond.
Cutting a hexagon with a sliding miter saw 2014-11-23
Joseph pose la question :
I need the angles in degrees for a hexagon one side is 18inches on the outside the problem I am having is in the angle of the cut and I am using a sliding miter saw which only goes from0 degrees to 45 degrees so I need help in getting the angle of the cut
Harley Weston lui répond.
Building an 8 sided box 2014-10-14
Michael pose la question :
I want to build an 8 sided box. The North, East, South, and West sides I want them to be 4 feet in length.
The NE, SE, SW, and NW sides I want to be 2 feet in length. What angles do I need to cut my angles.
Thanks Michael

Penny Nom lui répond.
A rectangle and a triangle 2014-09-27
rajesh pose la question :
The perimeter of a rectangle is equal to the perimeter of a right-angled triangle of height 12 cm if the base of the triangle is equal to the breadth of the rectangle, what is the length of the rectangle?
Penny Nom lui répond.
A wire is bent to form a right triangle 2014-09-25
Lorena pose la question :
A wire 24 dm is bent to form a right triangle. The ratio of the shorter leg to the hypotenuse is 3:5. Find the length of the longer leg?
Penny Nom lui répond.
The perimeter and diagonal of a rectangle 2014-09-10
niniha pose la question :
the perimeter of a rectangle is 68cm.if the diagonal is 26, find its dimensions
Penny Nom lui répond.
Interior angles in a parallelogram 2014-08-30
xavier pose la question :
so, one of my math homework questions requires me to know how to find out how to find the interior angles in a parallelogram, the question is, "how many interior angles does a parallelogram have?"
Penny Nom lui répond.
The perimeter of a triangle 2014-08-21
Phyllis pose la question :
What is the perimeter of a five acre triangle with one side being 250 feet and the second side would be at a 90 degree angle.
Thank you.

Penny Nom lui répond.
A barge of triangular cross section 2014-08-18
tushar pose la question :
a barge of triangular cross section is 20m long 12 m wide and 6m deep.its floats in SW at a draft if 4m find its displacement
Penny Nom lui répond.
The dimensions of a rectangle 2014-08-03
Jamaica pose la question :
if the perimeter of a rectangle is 30m and its area is 56 m2.find its length and width?
Penny Nom lui répond.
A ladder against a wall 2014-07-09
thabo pose la question :
A ladder 6.5m long,leans against a wall so that the top of the ladder is 4.8m from the ground.what is the angle of elevation of the ladder to the top of the wall
Penny Nom lui répond.
The width of a rectangle 2014-07-02
john pose la question :
if the area of a rectangular field is 90 sq. m and its length its is 19 m find its width
Penny Nom lui répond.
A triangle on the surface of the Earth 2014-06-26
Christine pose la question :
A, B and C are three towns, the bearing of B and C from A being 310 degrees and 220 degrees, and their distances from A are 510km and 700km respectively. Find the bearing of B from C to the nearest minute.
Robert Dawson lui répond.
An equilateral triangle with vertices in 3 parallel planes 2014-05-31
prajay pose la question :
How to construct an Equilateral Triangle whose vertices lie on three parallel lines, if the distances of two lines are 'a' and 'b' units from the middle line.What is the length of the side of the Equilateral Triangle ?
Chris Fisher lui répond.
A circle a square and a rectngle 2014-05-12
mazhar pose la question :
suppose the length and breadth of the rectangle are 5 cm and 10 cm respectively and M is a point along the corner of the circle. what is the radius of the circle?(diagram is given..but i didn't mention it..actually the diagram looks like a circle inscribed in a square and the right bottom corner one rectangle will be given ,it is touches to circle at a point M that I've already mentioned and the dimensions of that rectangle also I've mentioned) please help me out..
Penny Nom lui répond.
The length of a shadow 2014-05-09
vijay pose la question :
A girl of height 80 cm is running away from the base of a lamp-post at a speed of 2 m/s. If the lamp is 4 m above the ground, find the length of her shadow after 5 seconds.
Penny Nom lui répond.
Solve for theta 2014-04-25
ALASTAIR pose la question :
Hi, The question asks Solve for 'theta' cos2theta=sintheta x costheta. Substituting for cos2th either 1-2sin^2th or 2cos^2th does not give an equation in either sine or cosine alone how do I solve this please?
Penny Nom lui répond.
Two cones 2014-04-09
c.j pose la question :
what is the length of the radius of the LARGER cone(the LARGER cone has a slant height of 15) when the SMALLER cone has a radius of 8 and a slant height of 12ft ,please help.
Penny Nom lui répond.
The sides of a triangle 2014-04-06
Michael pose la question :
I am supposed to sove for the length of side "b" of an irregular triangle. I am given the following:
Side a: 65'
Side b: Find this length
Side c: 50'
Angle A: unknown
Angle B: unknown
Angle C: 52 degrees
I am supposed to use the law of cosines to solve for side "b" and my teacher says there is no mistake in the "givens" for the problem. I do not see how this can be done using the law of cosines and i have not figured out how to sove for angle B to use the law of cosines.

Penny Nom lui répond.
Shadows of a father and son 2014-04-02
esada pose la question :
the father and son cast a shadow of 11 feet and 8 feet,respectively if the son is 4'8'' in height,how tall is the father?
Penny Nom lui répond.
Is this a right triangle? 2014-04-01
amina pose la question :
decide whether or not the following points are the vertices of right triangle (-9,2),(-1,-2),(-9,11)
Penny Nom lui répond.
The faces of a triangle 2014-03-24
Westun pose la question :
Your answer says that a triangle has 0 faces but isn't the part of the shape you see the face!? Because i am having trouble with that so could you help me!
thanks! :)

Penny Nom lui répond.
A circle inscribed in a right triangle 2014-03-16
akshaya pose la question :
A circle with centre O and radius r is inscribed in a right angled triangle ABC. If AB=5 cm, BC=12 cm and < B=90*, then find the value of r.
Penny Nom lui répond.
A chord length of 2 cm 2014-03-13
prema pose la question :
I am having only chord length as 2cm. how to find arc length and degree
Penny Nom lui répond.
The angle of depression 2014-03-08
Ranger_minor pose la question :
A woman of height 1.4 metres standing on the top of a building 34.6 metres high views a tree some distance away. she observes that the angle of depression of the bottom of the tree is 35 degrees and the angle of depression of the top of the tree is 29 degrees.

assume that the building and the tree are on level ground :
1). calculate the distance of the woman from the top of the tree measured along her line of sight.

Penny Nom lui répond.
What are the possible lengths of the hypotenuse? 2014-03-07
audrey pose la question :
The three sides of a right angles triangle measure x-2, x+5, and 2x-1 in length. What are the possible lengths of the hypotenuse?

... I'm doing the equation c2=a2+b2 and subbing in the numbers but nothing makes sense

Penny Nom lui répond.
A circle graph 2014-03-06
Caitlyn pose la question :
I need to find the measure of the central angle that represents the amount of time spent on each activity.
My question includes a circle graph with the following: Sleep 31%, Other 15%, Entertainment 18%, Errands 7%, Work 20%, and Food 9%.

Penny Nom lui répond.
A triangle has angels in the extended ratio of 2:5:8 2014-02-06
Rubina pose la question :
a triangle has angels in the extended ratio of 2:5:8. find the measure of all three angles?
Penny Nom lui répond.
A rectangle inscribed in a circle 2014-01-10
Marian pose la question :
A 16 cm by 12 cm rectangle is inscribed in a circle. Find the radius of the circle.
Penny Nom lui répond.
A scalene triangle 2014-01-07
cherry pose la question :
Hi
I have been given a scalene triangle of sides -
8m, 6m, and 10m ..
Please help me to find out the area and help me to find out the height ..

Penny Nom lui répond.
An equilateral triangle inscribed in a circle 2014-01-06
Anonymous pose la question :
An equilateral triangle with sides 6 inches is inscribed in a circle. What is the diameter of the circle?
Penny Nom lui répond.
The perimeter of a rectangle 2013-10-24
sana pose la question :
if the length is given 70 m and the breadth is given as 40 m find the perimeter of the rectangle
Penny Nom lui répond.
A triangle problem 2013-10-02
raneem pose la question :
ABC is a triangle in which : BC=20cm. M(<B) =29 and m(<C)=73 . D is the midpoint of BC Find the length Of AB and AD approximated to 2 decimal places
Penny Nom lui répond.
Constructing a triangle 2013-08-22
Nazrul pose la question :
The base, the difference of the angles adjoining the base and the sum of the other two sides are given. How can I draw the triangle?
Please help me.

Chris Fisher lui répond.
Similar rectangles 2013-08-17
Mattie pose la question :
The dimensions of a rectangular-shaped picture frame are 14 inches long and four inches wide. Which dimensions below represent another frame that i geometrically similar>

a. l=49 in. and w=14in.
b. l=7in and w=3in
c. l=21in and w=8in
d. l=24in and w=14in

thanks

Penny Nom lui répond.
Using trig to find the height of a hill 2013-08-14
Anna pose la question :
From the top of a hill, the angles of depression of two successive milestones on a level road, which leads straight away from the hill, are 5degrees and 15degrees respectively. Fine the height of the hill.
Suggestion: BE is drawn perpendicular to AD. Find BE, then BD, finally CD.

Thanks :)

Penny Nom lui répond.
A triangle construction 2013-07-27
Nazrul pose la question :
Two angles and the difference of the lengths of their opposite sides of a triangle are given. How can I draw the triangle. Please help me.
Chris Fisher lui répond.
The angle of elevation of the sun 2013-07-03
Maurice pose la question :
A vertical pole with a length of 7m cast a shadow with a length of 5m. Calculate the angle of elevation of the sun and include a diagram.
Penny Nom lui répond.
An isosceles tiangle 2013-06-16
Izzy pose la question :
what's the height of an isosceles triangle which has a base of 50 m, and both of the other sides are 25 m?
Penny Nom lui répond.
A kennel for a beagle 2013-06-03
david pose la question :
Hi, I'm building some beagle kennels and I am in need of help with an angle problem. I need to place a roof on my kennel with a drop of 2inches across 3ft 10inches. the posts on the right side will be 5ft and the post on the left will be 4ft 10in. the posts are 4x4 and the space to be covered is 3ft 10in from the outside of the 4x4. Please help, thanks.
Penny Nom lui répond.
siin (A) and sin (A/2) 2013-05-09
shanaia pose la question :
given that sin A=4/5 and A is obtuse.find sin (A/2)
Penny Nom lui répond.
A triangle and an incircle 2013-05-09
Max pose la question :
On my Geometry Test about tangent, chord, and secant lengths, my teacher gave an extremely difficult problem.
It was a Circle inscribed in a Triangle with all triangle sides being tangents and lengths were given. My class was told to find the length of each segment of the line. The points on each line were the vertexes of the triangle, and the point where the line hits the circle.
Please explain how someone could do this.

Chris Fisher lui répond.
Drawing a pentagon 2013-05-06
jacob pose la question :
hello. i am trying to draw a template for a project. but i don't have a protractor. i'm trying to draw a normal pentagon with 2cm sides. what i'm trying to figure out is the distance in centimeters from point a, to c, in a straight line. this is needed so that i can measure and create the shape without a protractor, and keep the angles and sides correct. if you can help me with the measurement, or help me another way, any help would be greatly appreciated. thank you.
Penny Nom lui répond.
The area of a right triangle 2013-04-10
derek pose la question :
The legs of a right triangle measure 4 and 6 inches. The area of the triangle is
Penny Nom lui répond.
The area of an isosceles right triangle 2013-03-30
Michelle pose la question :
We have a triangle it is a right isosceles. The sides touching the right angle are 10 cm. They hypotenuse is 14.1 (the base) we are trying to find the height of this so we can figure out the area of a shaded sector of a circle.
Chris Fisher lui répond.
A shortest distance problem 2013-03-28
LYNDELL pose la question :
I have a right triangle and know the length of all sides. How do I calculate the shortest distance from the vertex of the 90 degree angle to the hypotenuse?
Penny Nom lui répond.
Trigonometry 2013-03-23
Tizoc pose la question :
I am in a trig class and I have a conflict. When solving the length of a side, I know what trig function to use, but I do not know what angle to use in a calculator. To make this a little more understandable, if I have all the angles available in a right triangle and I use the tangent function, how do I know what to use?
Heres what I do not know what to put in my calculator: Tan(?)
Thanks in Advance!

Penny Nom lui répond.
The dimensions of a rectangle 2013-03-19
stephanie pose la question :
the length of a rectangle is 5m more than twice its width, and the area of the rectangle is 88m^2 find the demensions of the rectangle
Penny Nom lui répond.
Fencing a parcel of land 2013-03-11
Koleen pose la question :
How much fencing do I need for an 56,850 parcel of land for a 15 stable horse barn
Penny Nom lui répond.
The area of a rectangle 2013-01-29
Bali pose la question :
if the length of the rectangular field is decreased by 5m, its area is decreased by 70 sqm. if the width is increased by 6m, the area is increased by 180 sqm. determine the length and width of the rectangular field?
Penny Nom lui répond.
An equilateral triangle inscribed in a circle 2013-01-17
Nicole pose la question :
How do you find the shaded region of a circle if an unshaded equilateral triangle in inscribed in it. The only other things I know about the problem are that the side lengths of the equilateral triangle are 14 inches.
Penny Nom lui répond.
The side lengths of a triangle 2013-01-07
Katie pose la question :
How do I find the side lengths of a triangle knowing the base and its height?

Thank you,

Penny Nom lui répond.
The perimeter of an equilateral triangle 2012-12-14
shreyaarora pose la question :
if the area of on equilateral triangle is 24/3cm,then what is its perimeter
Penny Nom lui répond.
The angles of elevation and depression 2012-12-03
Chelsey pose la question :
a person on a balcony of one building looks towards a second building. if the angle of elevation to the top of the second building is 25 degrees, the angle of depression to the bottom of the second building is 17 degrees, and the balcony of the first building is 22 feet above the ground, what is the height of the second building?
Penny Nom lui répond.
A circle inscribed in a triangle 2012-11-28
Angie pose la question :
Circle O is inscribed in triangle ABC. Angle A = 50 and angle B = 60. Find arc XY in dgrees
Penny Nom lui répond.
The area of a triangle given its vertices 2012-11-27
elizabeth pose la question :
how to calculate the area of a triangle using points ABC a (2,5) b ( -4,-3) c ( 6, -1)
Penny Nom lui répond.
Two sides of a triangle 2012-11-25
Nazrul pose la question :
In the triangle ABC, AB>AC and the bisector AD of angle BAC intersects BC at D. How can I prove that angle ADB is an obtuse angle.
Penny Nom lui répond.
Two congruent circles in a rectangle 2012-10-20
Alexander pose la question :
Have you ever solved a problem, in which you have a rectangle, from which you need to cut the largest two circles of equivalent diameter? I bisected a rectangle diagonally, but the circles, while tangent to two of the sides, are not tangent to eachother. Can you devise a method for two equivalent circles, that are tangent to two sides, are also to eachother?

Take for example a piece of paper, Each if the two largest circles has a diameter that is greater than the distance to the midpoint of the diagonal bisector of the rectangle.

Chris Fisher lui répond.
The degree measure of the central arc of a circle 2012-10-17
Crystal pose la question :
On a circle with radius of 12 cm is an arc of length 20 cm. What is the degree measure of the central angle used to make this arc?
Penny Nom lui répond.
The height of a triangle 2012-10-17
Brian pose la question :
the base of my triangle is 12 metres and the two sides are 8.45 metres can you help me find the height of the triangle
Penny Nom lui répond.
The perimeter of a rectangle 2012-09-19
aretha pose la question :
calculate the perimeter of a rectangle with the long side 462ft x 9inches and the short side 50% less.
Penny Nom lui répond.
The dimensions of a rectangle 2012-09-12
Becky pose la question :
A rectangle has an area of 48 square inches and perimeter of 32 inches. Find the dimensions of the rectangle.
Penny Nom lui répond.
An equilateral triangle and a regular hexagon in a circle 2012-09-11
Heidemarie pose la question :
The vertices of an equilateral triangle with side length of 10 sqrt 3 cm lie on a circle. Find the side length of the regular hexagon whose vertices lie on the same circle.
Penny Nom lui répond.
The hypotenuse 2012-09-06
Jeevan pose la question :
how can i find the height and base of a right angle triangle if i have the hypotenuse only ?
Penny Nom lui répond.
The area of a rectangle 2012-08-31
mario pose la question :
The area of a triangle when on side is x and the other is x+6
Harley Weston lui répond.
Problem solving 2012-08-15
Sugavanas pose la question :
If length of a rectangle exceeds its width by 5 m. if the width is increased by 1m and the length is decreased by 2 m, the area of the new rectangle is4 sq m less than the area of the original rectangle. Find the dimensions of the original rectangle.
Penny Nom lui répond.
Two altitudes of a scalene triangle 2012-08-13
grace pose la question :
Two of the altitudes of a scalene triangle ABC have length 4 and 12. If the length of the third altitude is also an integer, what is the biggest that it can be? Justify all of your conclusions.
Chris Fisher lui répond.
Three right triangles 2012-07-26
Jora pose la question :
I am having a lot of difficulties with this question.

Name: Jora
Subject: Math
Who are you: Student

Chris Fisher lui répond.
The area of a rectangle 2012-07-24
michael pose la question :
the dimension of the rectangle are (x+5) and (x-3) find its area
Penny Nom lui répond.
A truncated cone: the central angle 2012-07-17
Tom pose la question :
I have researched several sites, including this one and am fairly confident I can do the calculations required to produce the two radii and the slant height for a truncated cone. I a somewhat confused by the central angle. Some sites indicate that it should never exceed 180 degrees while others do not. Different examples on your site seem to use this in two different ways in constructing a truncated cone. Sometimes the angle seems to indicate the section to be removed, while in other examples the angle seems to indicate the section to be saved. Since the two angles will always total 360 I am confused about how to use the info to calculate the minimum rectangle required to contain the pattern. I'm guessing that in some cases I may need to use the chord or sagitta to determine the desired height and width of the material. Any help is greatly appreciated. Thank you. Tom
Penny Nom lui répond.
The angular elevation of the sun 2012-07-14
VINEET pose la question :
WHAT IS ANGULAR ELEVATION OF THE SUM
Penny Nom lui répond.
The height of an isosceles triangle 2012-07-10
ken pose la question :
I am trying to determine the various heights of an isosceles triangle, if each has the same base dimension and varies in the degree of the base (equal) angles. What is the method to do this? As an example, of the base is 10, and the two equal angles are each 45 degrees, what is the height? With the same base (10), but with the two equal angles at 60 degrees, what is the height? And with the same base (10) and the two equal angles at 75 degrees, what would be the height?
I know how to calculate the degrees of the third angle (add the degrees of the known angles, and subtract from 180); but am unsure if that is needed for figuring the overall height. And to be clear; I am not looking for the length of the sides of the triangle, but the height from the base to the top point.
Thank you!

Chris Fisher lui répond.
An octagonal sandbox around a pool 2012-06-12
Linda pose la question :
My pool is 15 feet across and it is round.

How do I measure to cut wood to build a sandbox around it? was thinking it will look like an octagon.

Thanks

Harley Weston lui répond.
Finding the circumcentre 2012-05-24
Glen pose la question :
Hi, I'm having trouble finding the circumcentre of these 3 co-ordinates- A(1,1) B(10.5,1) and C(6,6). I understand what circumcentre is but I cannot work out where it is. Thanks very Much, Glen
Chris Fisher lui répond.
The area of a triangle 2012-05-17
Emilee pose la question :
hi. How do i find the area of a triangle if the vertices are (-3,0) (-2,2) and (3,0). I graphed it and it came out a triangle. How do I find the area of it in an easy way?
Penny Nom lui répond.
Drawing an isosceles triangle 2012-05-10
Nazrul pose la question :
How can I draw an isosceles triangle whose each angle adjacent to the base is twice the vertex angle?
Chris Fisher lui répond.
The maximum distance from the vertex of a triangle 2012-05-02
David pose la question :
There are three towns A,B,and C, equi-distant apart. A car is 3 miles from town A, and 4 miles from town B. (ie, somehwere outside of the triangle which the three towns form) What is the maximum distance that the car can be from town A?
This was asked as quiz question in my local pub last Sunday.
The answer is 7. How do I prove it?
Best regards. David in Denton.

Robert Dawson and Chris Fisher lui répond.
The dimensions of a rectangle 2012-04-30
Nikki pose la question :
Name the dimensions of a rectangle that has the same area as this triangle? I found the area of the triangle, which is 30. What do I have to do in order to complete this math problem?
Penny Nom lui répond.
Third side of a irregular triangle. 2012-04-27
Darya. pose la question :
How to find third side of irregular triangle if c = 17m, b = 28m and area S = 210m^2.
Penny Nom lui répond.
A circle inscribed in a triangle inscribed in a circle 2012-04-26
Maty pose la question :
How do i find the area of a triangle inscribed a circle while another smaller circle is circumscribed by the same triangle and the radius is 8.
Chris Fisher lui répond.
The circumcentre and orthocentre of a triangle 2012-04-25
Nazrul pose la question :
The circumcentre and orthocentre of a triangle is given. A point on a side of the triangle given. A vertex of the triangle on the circumcircle is also given. How can I draw the triangle? Please Help me. Thank you for answering my previous questions.
Chris Fisher lui répond.
How to find the base length of a isosceles triangle if only the sides are given? 2012-04-25
aqilah pose la question :
how to find the base length of a isosceles triangle if only the sides are given?
Chris Fisher lui répond.
A puzzle with a triangle and 57 2012-04-25
sharon pose la question :

Question from sharon, a parent:

We have been battling with this for days and cant find any solution -

Using each of the following numbers once, fill all the spaces in the triangles so that each side of the triangle adds up to 57

numbers are 11,12,13,14,15,16,17,18,19

diagram is -

      0      
    0   0    
  0   57   0  
0   0   0   0

 

Any ideas ? this has to be in in just over 24 hrs ......help!


Chris Fisher lui répond.
The area of a right triangle 2012-04-03
Shelli pose la question :
How do you find the area of a right triangle if you only know that the hypotenuse is 10 and that the base and height are unknown but are equal?
Penny Nom lui répond.
A circle drawn around a equilateral triangle 2012-04-01
BIMAL pose la question :
what is the diameter of a circle drawn around a equilateral triangle of size 6 cm
Penny Nom lui répond.
Angle measures 2012-03-21
jogiboy pose la question :
how can i get the sine theta if the given is 3.14/3
Penny Nom lui répond.
Gravel, sand, cement and an equilateral triangle 2012-03-21
saheed pose la question :
how many tonnes of gravel and sand,bags of cements will be required to concrete 5feet depth/deep of 4feet equilateral triangle?
Harley Weston lui répond.
Forming a triangle from 3 line segments 2012-03-15
rustom pose la question :
A point X is selected at random from a line segment AB with midpoint 0. Find the probability that the line segments AX, XB, and A0 can form a triangle.
Penny Nom lui répond.
The dimensions of a rectangle 2012-03-13
irvyn pose la question :
if you are only given the area and the perimeter how do you find the dimension?
Penny Nom lui répond.
The area and perimeter of a rectangle 2012-03-01
RAJIB pose la question :
PEREMETER OF A RECTANGLE IS 24. FIND THE AREA ?
Penny Nom lui répond.
The height of an equilateral triangle 2012-02-29
Carley pose la question :
Hi my name is Carley. I am an 8th garder. What us the height of an equilateral triangle si the sides are 18 cm?
Penny Nom lui répond.
A road around a garden 2012-02-07
Manu pose la question :
Inside a square garden of side 158m, a road 4m wide is built all around. What is the area of the remaining part of the garden?
Penny Nom lui répond.
The dimensions of a rectangle 2012-01-31
Dazz pose la question :
Hi I was wondering how to find the length of a rectangle if you know the area is 484 square inches And the length is 10 square inches or 22 square inches?
Thanks~Dazz

Penny Nom lui répond.
The area of a triangle 2012-01-27
mary pose la question :
I have a scalene triangle and the base is 33m, side is 52m and the unparallel side is 25m but i drew a dotted line down and that s 20m

how do i figure the area?

thanks

Penny Nom lui répond.
1 + 3 + 3^2 ...+3^(n-1) = 3^n - 1/2 2012-01-27
Vicki pose la question :
I am trying to find out how to do show how this proof was worked.
Here is the end result 1 + 3 + 3^2 ...+3^(n-1) = 3^n - 1/2

This equation was used to find the number of white triangles in the Sierpinski Triangle

Walter Whiteley lui répond.
What is the height of the triangle? 2012-01-12
Hailey pose la question :
The area of a triangle is 77ft2. if the base is 11ft. how tall is the height?
Penny Nom lui répond.
The dimensions of a rectangle 2012-01-06
joseph pose la question :
The perimeter of a rectangle is 72 inches. The length of the rectangle is three more than 2 times the width of the rectangle. What is the length of the rectangle? Write and solve an equation to determine the dimension of the rectangle.
Penny Nom lui répond.
Triangles, area and perimeter 2012-01-01
Panny pose la question :
I saw a question asked on your board without an answer.
How do I find the area of a triangle any triangle if I only know the perimeter..

Penny Nom lui répond.
Three sides of a triangle 2011-12-24
saba pose la question :
the three sides of a triangular lot have lengths 10,11and 13cm,respectively. find the measure of its largest angle and the area of the lot?
Penny Nom lui répond.
Jennifer wants to fence in her backyard 2011-12-09
Val pose la question :
Jennifer wants to fence in her backyard. A yard is rectangular and is 15 feet by 12 feet. One 12-foot side is against the house so it does not need fencing. How many feet of fencing does Jennifer need?
Penny Nom lui répond.
Squares and triangles 2011-12-06
Liaqath pose la question :
You have squares and triangles.
Altogether there are 33 sides.
How many squares do you have?
How many triangles do you have?

Penny Nom lui répond.
Solve for theta if 8cos^2 theta-3=1 2011-12-02
Katherine pose la question :
Hi, I have just learned to solve trigonometric problems for theta and have one specific question in order to find the solutions to my homework. I will use one example for this question. If I have 8cos^2 theta-3=1 I first divide by 8 and get cos^2theta=3/8 then I have cos theta= plus or minus the square root of 3/8 Then I assume that I plug in inverse cos (the square root of 3/8) to my calculator. How do I find the four solutions (we are typically supposed to find four, I believe?) Can you help me with finding the solution to this problem? Thank you!
Penny Nom lui répond.
A triangle problem 2011-11-20
May pose la question :
In triangle ABC, AB=AC, angle A=20,
D lie on AB making DC=AD,
E lie on AC making angle EBC=70.
Find angle DEB.

Chris Fisher lui répond.
The 3, 4, 5 right triangle 2011-11-15
rishil pose la question :
to the nearest degree,what is the measure of the smalles angle in a rightt triangle with sides of length 3,4 and 5?
Penny Nom lui répond.
A right angled triangle 2011-10-31
bijo pose la question :
how can i find the tangent point at a circle with origin as center with radius r and the tangent pass through a given point P? I also want to find out the third point of a right angled triangle given other two points?
Chris Fisher lui répond.
The area of a triangular garden 2011-10-31
tasha pose la question :
a garden has sides of 6m,8m and 10m find the area
Chris Fisher lui répond.
Building a custom range hood 2011-10-08
Bill pose la question :
I'm building a custom range hood for a customer with special order material that matches their newly installed cabinets and I need it to be perfect. The hood is basically a pyramid but the 4th side is the flat wall at the back and a flat, rectangular top. I need to calculate the bevel and miter of the three sides but I never was very good with geometry functions (although I am fairly good with other math fields). I either need the calculations from you at least (shudder) a formula or set of formulas so that I can calculate them myself.
Harley Weston lui répond.
A rectangle of largest possible area 2011-09-16
mary pose la question :
Steven has 100 feet of fencing and wants to build a fence in a shape of a rectangle to enclose the largest possible area what should be the dimensions of the rectangle
Penny Nom lui répond.
A rectangle is inscribed in a circle 2011-07-17
Alexea pose la question :
A rectangle is inscribed in a circle of diameter 15in. Express the perimeter as a function of the width x.
Penny Nom lui répond.
Construct a 30 degree angle 2011-06-10
Ella pose la question :
they give you a simple straight line and ask you to construct an angle of 30 degrees at one end of the line using only a ruler and a compass?
Penny Nom lui répond.
What is the angle of decent? 2011-05-20
Taten pose la question :
So you have an airplane with an altitude of 5.8KM. It has 46KM till the run way. What is the angle of decent?
Penny Nom lui répond.
The dimensions of a rectangle 2011-05-08
yolanda pose la question :
If the length of a rectangle is 3ft less than twice its width and the area of the rectangle is 54ft^2,what are the dimensions of the rectangle?
Penny Nom lui répond.
The volume of a triangular shaped aquarium 2011-05-07
Jeff pose la question :
HI, I searched the site but really couldn't find my answer, at least that I could understand! I have a triangular shaped aquarium ( think of a cube, except its an equal triangle shape). all 3 sides are 48 inches, and it is 34" tall. Can you please tell me what the volume is in gallons? Thanks!
Penny Nom lui répond.
A circle inscribed in a triangle 2011-05-07
Aishwarya pose la question :
The angles of a triangle are 50, 60, and 70 degrees, and a circle is touches the sides at A, B, C. Calculate the angles of the triangle ABC.
Penny Nom lui répond.
An exclusion zone around a triangle II 2011-05-03
Aishwarya pose la question :
A long time ago Mr gibson found an island shaped as a triangle with three straight shores of length 3km, 4km and 5km. He declared an exclusion zone around his island and forbade anyone to come within 1km of his shore. What was the area of his exclusion zone? This question was answered but did not understand the explanation.
Robert Dawson lui répond.
The area of a rectangle 2011-04-30
Hayley pose la question :
A rectangles width is exactly twice its length. The area of the rectangle is 450 square feet. What are the length and width of the rectangle?
Penny Nom lui répond.
cos(x) = -1/(square root of 2) 2011-04-27
Shelby pose la question :
Find exact value of x for 1 <(or equal to) x < 2pi a) cos(x) = -1/(square root of 2)
Penny Nom lui répond.
A right angled triangle 2011-04-22
jeremy pose la question :
Solve the right triangle with the given sides and angles. a=3.0 cm and b=1.6 cm
Penny Nom lui répond.
Dividing a polygon into triangles 2011-04-16
Foxie pose la question :
You have a given regular polygon with n vertices and you divide it into triangles(using the vertices of the polygon) which each share at least one side with the polygon. How many distinct ways can you divide the polygon if its vertices are numbered? For n=3 it's 1 way, for n=4 it's 2 ways for n=5 5 ways, I'm not quite sure but think that for n=6 it's 12 ways... thanks in advance!
Claude Tardif lui répond.
A concrete walk around a pool 2011-04-15
Skipper pose la question :
I want to know the answer if you were to want a pool to be 20 ft by 30 ft and wanted a 5 ft border around it what would be the dimension of the whole area plus concrete walk
Penny Nom lui répond.
The perimeter of a rectangle 2011-04-04
aspen pose la question :
the perimeter of a rectangle is twice the sum of its length and its width. the perimeter is 40 meters and its length is 2 meters more then twice its width. what is the length?
Penny Nom lui répond.
The rectangle is reduced to 90% of its original size 2011-03-31
stephon pose la question :
A rectangle has a length of 4cm and a width of 8cm.

If this rectangle is reduced to 90% of its original size what will be the new length

Penny Nom lui répond.
sin x = -0.25 2011-03-29
Wayne pose la question :
How do you solve for x in the equation sin x = -0.25

the answer is 3.394 and 6.030 but I don't know the steps they used to calculate this

Penny Nom lui répond.
The interior angles of a pentagon 2011-03-10
Daima pose la question :
I need help!
The interior angle of a pentagon i s5 ( I hope ) or 540 degrees.
The interior angle of a pentagon is what? Explain to me please.

Daima

Penny Nom lui répond.
Find the dimension of the original rectangle 2011-02-28
sandra pose la question :
the perimeter of a rectangle is 40 cm. if the length were doubled and the width halved, the perimeter would be increased by 16cm. Find the dimension of the original rectangle
Robert Dawson lui répond.
Triangles with perimeter 16 cm 2011-02-22
Chong pose la question :
How many triangles (up yo congruence) with perimeter 16 cm and whose lengths of its side are integers?
Chris Fisher lui répond.
Prove sin x = sin (pi - x) 2011-02-15
Janet pose la question :
Prove sin x = sin (pi - x)
Penny Nom lui répond.
Calibrating a conical tank 2011-02-05
Bill pose la question :
Hi, I have a round tank with tapered sides where I know the diameter at the top and bottom. Is there a formula I can use to calculate the volume by measuring from the bottom up the side (at the angle of the side) to any given point? Thanks, Bill
Stephen La Rocque and Penny Nom lui répond.
A cone with a specific angle 2011-02-01
John pose la question :
Hi my daughter came with a seemingly easy question (which to me it was not) How to make/calculate a cone of a specific angle from top to bottom radius.
Penny Nom lui répond.
The area of a rectangle 2011-01-31
T.S. pose la question :
for which value of w does the rectangle below have an area of 864 square units if sides are width=w+15 and length=w-15
Penny Nom lui répond.
The sides of an equilateral triangle 2011-01-27
Cristal pose la question :
Hello :) How can i find the sides of an equilateral triangle when only given the height 10m?
Penny Nom lui répond.
The area of isosceles triangle 2011-01-24
imraan pose la question :
how to find the area of isosceles triangle by knowing only its sides
Penny Nom lui répond.
A rectangle 2011-01-16
meg pose la question :
if a rectangle has a length that is 3cm less than 4 times its width and its perimeter is 19cm, what are the dimension?
Penny Nom lui répond.
The height of a lamp 2011-01-13
Dorothy pose la question :
I need to find the lamp height with casting shadow (base line of triangle where a boy 1.6m tall stands 3m from base of street lamp and has a 2m shadow. In other words, think of a right angle triangle with zero height starting at left, then 2m to right stands boy (1.6m high). Angle (hypotenuse) increases up to top of street lamp with 'x' height and 3m base.
Penny Nom lui répond.
The angle between a line and a plane 2011-01-12
tom pose la question :
what are the angles of the diagonal of a rectangular parallelepiped 2in by 3 in by 4 in makes with the faces...
You know this is a problem that I can't figure out ...I don't know where the angle and the diagonals where? can you help me with this one?

Penny Nom lui répond.
Tan(x+pi)-tan(pi-x)=2tan(x). 2011-01-09
Steven pose la question :
Verify the identity:
Tan(x+pi)-tan(pi-x)=2tan(x)

Penny Nom lui répond.
The third side of a triangle 2010-12-28
andrea pose la question :
in triangle abc, a-b=5 and the area is 50cm2.If angle c- 90degrees, find the length of c.
Penny Nom lui répond.
Draw a figure with 16 triangles using only 6 line segements 2010-12-16
Jill pose la question :
How can you draw a figure with 16 triangles using only 6 line segements?
Stephen La Rocque lui répond.
Angle of elevation 2010-12-10
PANKAJ pose la question :
angle of elevation of the sun perpendicular72 and base 88 find angle
Penny Nom lui répond.
A television camera at a basketball court 2010-12-06
ryan pose la question :
Can you just please give me an idea on how the diagram would look like. I'm really confused.

A television camera is 30 ft. from the sideline of a basketball court 94 ft long. The camera is located 7 ft from the midcourt. Through what angle must it sweep in order to cover all action on the court?

Penny Nom lui répond.
Can determine if it is scalene, isosceles, or equilateral 2010-12-01
Jessie pose la question :
find the measures of the sides of triangle KPL and classify each triangle by its sides. my first problem would be K(-3,2) P(2,1) L(-2,-3) ...The three points they give you are the vertices of the triangle and you need to match them up. Draw the triangle and write in the vertices and the related point with the vertex. You will then do the distance formula three times to find the distance of all three sides. Once you have the three sides you can determine if it is scalene, isosceles, or equilateral...using the distance formula how do i solve this?
Penny Nom lui répond.
A rectangular piece of plywood 2010-11-30
Austin pose la question :
A rectangular piece of plywood is trimmed to make it square by cutting a 4cm strip off the top and a 2cm strip off one side. If the area of the original piece is 74cm2 greater than the area of the square, find the dimensions of the rectangle.
Penny Nom lui répond.
The perimeter of a rectangle 2010-11-24
shericka pose la question :
a rectangle has a perimeter of 60cm. its length is 5 times greater than its width. what is its length?
Penny Nom lui répond.
The area of an equilateral triangle 2010-11-11
ginny pose la question :
If a equilateral triangle has side lengths of 41.33m how would i calculate the area? if i don't know the height of the triangle?
thank you.

Penny Nom lui répond.
The base of an isosceles triangle 2010-10-24
Brian pose la question :
how can I find the base of an isosceles triangle from the height and the perimeter?
Penny Nom lui répond.
A rectangle with rounded corners 2010-10-21
Connie pose la question :
How do you find surface area if the rectangle is 100ft. by 200ft. with quarter circles in the corners with a radius of 20ft. ?
Penny Nom lui répond.
The angles in an m-gon and genrealizations 2010-10-16
Michael pose la question :
Hello: In answer to a student's question, someone named Penny from your organization provided a proof that the sum of the interior angles of a triangle in the plane is pi radians (or 180 degrees).

I am interested (and I'm sure many other people would be too) in 3 potential generalizations of this basic fact in plane geometry:

Walter Whiteley lui répond.
The third vertex of a triangle 2010-09-23
nikos pose la question :
Hi, I would really appreciate any help in this:
"Suppose you have a triangle (any triangle, no hypotheses allowed), let's call it ABC. You know the coordinates of the two vertexes, A(x1,y1), B(x2,y2) and the area of the triagnle, let's call it D. Can you find the third vertex of this triangle based on these facts?"

Thanks a lot!

Harley Weston lui répond.
Limiting Cases in Geometry 2010-09-22
Niki pose la question :
Consider a rectangle inscribed in a circle with a radius or R. What are the possible perimeters for the rectangle?
Stephen La Rocque lui répond.
A building and a flag pole 2010-09-09
paul pose la question :
A flag pole and a building stand on the same horizontal level. From the point p at the bottom of the building,the angle of elevation of the top t of the flag pole is 65 degrees. From the top q of the building the angle of elevation of the point t is 25 degrees.If the building is20 meters high. Calculate the distance pt
Penny Nom lui répond.
The height of a triangle 2010-08-05
Ann pose la question :
The height of a triangle is 4cm less than the base. The area is 198cm^2. Find the height.
Stephen La Rocque lui répond.
A spaceship playhouse 2010-07-26
Dave pose la question :
I would like to build a spaceship playhouse for my grandson I want it to be about 36" around and I want to use 5/4 decking boards that measure 5 1/2" wide how do I figure out how to lay out a base pattern to nail to (what angle do I need to cut and how many boards will it take to go around the circle.
Penny Nom lui répond.
An octagon shaped bench 2010-07-09
rob pose la question :
i am trying to build a octagon shaped bench to fit inside a 69 inch round hot tub so that the tip of each point touches the edge of the circle where it will be fastened.
Stephen La Rocque lui répond.
Rays and angles 2010-06-24
cristina pose la question :
what is the formula for finding the number of angels that can be named by a given number of rays with the same endpoint?
Robert Dawson lui répond.
The height of a triangle 2010-06-21
Taylor pose la question :
how do i find the height of a triangle with a base of 3 and a side of 5 the book says 4 but i cant get that
Penny Nom lui répond.
How wide a strip has he cut when the lawn is half mowed? 2010-05-24
Arib pose la question :
rectangular lawn has length 40 m and width 30 m. David starts cutting from the outside in. How wide a strip has he cut when the lawn is half mowed?
Penny Nom lui répond.
The area of a triangle 2010-05-22
jose pose la question :
haw many square feet are in a triangle 30 feet high with a 100 ft base?
Tyler Wood lui répond.
The area of a triangle 2010-05-10
Saeed pose la question :
Show that:
Area of triangle = half of the perimeter of the triangle times the radius of the inscribed circle !!!!!!!!!!!!!!!

Penny Nom lui répond.
The altitude of a triangle 2010-05-08
kylie pose la question :
the vertex angle of an isosceles triangle is 57 degrees 24 minutes and each of its equal sides is 375.5 feet long. find the altitude of the triangle
Penny Nom lui répond.
An angle in a triangle 2010-05-06
Morgan pose la question :
Question from Morgan, a student:

                 t
        8   t      t 8
          t    78   t
       t               t
  t                        t
t   x                       t
ttttttttttttttttttttttttt
              10

I'm having trouble solving for x I'm not sure where to start ( the ones in the middle of the triangle are both degrees) thanks in advance for your help i really do appreciate it
Penny Nom lui répond.

A wishing well 2010-05-04
Cassie pose la question :
I'm trying to construct a Wishing Well made of treated 2x4 wood, so the actual measurement is 3 1/2 by 1 1/2. The well is going to be three feet in diameter. I'm trying to construct this as circular as possible, what angle should the wood panels be cut at?
Penny Nom lui répond.
A trig equation 2010-04-28
Steve pose la question :
2cos^2(X)-2sin^2(x)+1=0
Harley Weston lui répond.
cos(x) = sin(x - 1) 2010-04-28
alex pose la question :
In the equation cos x = sin x-1 for -pi/2 A: solve for x graphically
B: solve algebraically and prove the solution is correct.

Alex

Penny Nom lui répond.
The height of a flag shaft 2010-04-25
Sarah pose la question :
A man standing 20metres away from a tower observes the angles of elevation to the top and bottom of a flag shaft standing on the tower as 62degrees and 60degrees respectively. Calculate the height of the flag shaft.'
Penny Nom lui répond.
A rectangle problem 2010-04-12
Charlene pose la question :
The ratio of the width to the length of a certain rectangle is equal to the ratio of its length to the difference between its length and twice its width. If the area of the rectangle is 25cm^2, then what are its dimensions? Present a non- algebraic solution?
Tyler Wood lui répond.
The perimeter of a rectangle 2010-04-12
Charlene pose la question :
The area of a rectangle is 390m^2. If its length is increased by 10m and its width is decreased by 6m, then its area does not change. Find the perimeter of the original rectangle.
Tyler Wood lui répond.
A regular hexagon and an equilateral triangle in a circle 2010-04-05
Beth pose la question :
A regular hexagon and an equilateral triangle are both inscribed in the same circle so that the hexagon and the triangle share three vertices. The radius of the circle is 10cm. What is the difference between the area of the hexagon and the area of the triangle?
Chris Fisher lui répond.
The diagonals of a rectangle 2010-03-30
Tabatha pose la question :
The diagonals of a rectangle are 8 units long and intersect at a 60 degree angle. Find the dimensions of the rectangle.
Penny Nom lui répond.
The sides of a 30-60-90 triangle 2010-03-29
maris pose la question :
Find the sides of a 30-60-90 triangle whose perimeter is 6. The solution must use systems of equations using 3 variables.
Penny Nom lui répond.
The angle of intersection between two lines 2010-03-25
Madhumitha pose la question :
How to find the angle of intersection between a line x=249 and another line which is of the form y=m*x+c; Line1 has a slope of infinity so what does the angle of intersection turn out to be? Or how can i find it?
Chris Fisher lui répond.
A rectangle inscribed in a circle 2010-03-24
sadiq pose la question :
here is the question, in my mathematics book there is equation of the area of the rectangle inscribed in a circle having equation x^2+y^2=a^2 and the area of rectangle is 4xy=4x(a^2-b^2)^1/2 i don't know what is b but a is surely the radius (i want the derivation for the area of rectangle).
Harley Weston lui répond.
A quadrilateral with 4 known sides and 1 known angle 2010-03-19
samuel pose la question :
Name: Samuel
Status: Student

I have a quadrilateral with 4 known sides and 1 known angle, and I'm trying to evaluate the other angles of my quadrilateral.

By the law of cosines, I can easily find my opposite angle (using the diagonal as a basis for the equation).

However, to find the two remaining angles, I have found no other way so far than to use the other diagonal, which can be found with the equation attached (from geometry atlas).

Is there any simpler way?

Robert Dawson and Harley Weston lui répond.
The perimeter of a rectangle 2010-03-12
makiah pose la question :
A rectangle has a perimeter of 12 meters.If each side is a whole number of meters, what are the possible dimensions for the length and width?
Penny Nom lui répond.
An irregular octagon 2010-03-09
Gayle pose la question :
Question from Gayle:

I am building an irregular shaped octagon wooden box.
The measurements are 291/2 inches by 211/2 inches.
Sides are 12 inches.
It will be 36 inches high.

What would the cutting angles degrees be?

Harley Weston lui répond.
The dimensions of a rectangle 2010-03-03
Kirsten pose la question :
A rectangle is twice as long as it is wide. Its area is 128 square meters. What are the dimensions of the rectangle?
Tyler Wood lui répond.
The hypotenuse 2010-02-27
Dannielle pose la question :
how do you find the hypotenuse if a=8 and b=6?
Penny Nom lui répond.
How many triangles...? 2010-02-22
deciree pose la question :
Given 12 points, no 3 of them on a line with 6 red, 4 blue and 2 green points.
a) How many triangles have vertices all the same color?
b)How many triangles have vertices with each vertex a different color?
c)How many triangles have at least one green vertex?

thanks to lorraine!

Penny Nom lui répond.
Triangles 2010-02-19
Wiliam pose la question :
A box contains one 2 inch rod, one 3 inch rod, one 4 inch, and one 5 inch rod. What is the maximum number of different triangles that can be made using these rods as sides?
Penny Nom lui répond.
The vertices of an equilateral triangle 2010-02-19
Emma pose la question :
If two of the vertices of an equilateral triangle are (2,1) and (6,5), what are the two possible coordinates of the third side of the triangle? Answer in radical form.
Tyler Wood lui répond.
An isosceles triangle 2010-02-11
Kim pose la question :
I am given the length of the two legs of an isosceles triangle (8), and the base angles are 30 degrees...I am asked to find the area of the triangle with only this information
Tyler Wood lui répond.
The angle at which the road is inclined 2010-02-10
abeth pose la question :
a mountain road drops 5 m for every 22 m of road. Calculate the angle at which the road is inclined to the horizontal to the nearest degree.
Penny Nom lui répond.
The other two sides of a right angle triangle 2010-02-09
ayesha pose la question :
how to find the other two sides of right angle triangle when length of one side and angle of other side is given i.e 45
Penny Nom lui répond.
A dirt berm 2010-02-05
casey pose la question :
My question is two fold. I want to build a dirt berm that is 95 feet long and 8 feet high which has a 45 degree angle on each side of the slope. My first question is how wide will the berm end up being? Second question is how many cubic yards of dirt will I need to get? Thanks!
Harley Weston lui répond.
Fencing a garden 2010-02-04
kris pose la question :
How many meters of fencing do i need to enclose a garden if the yard is 26 meters long and half as wide?
Penny Nom lui répond.
An impossible isosceles triangle 2010-01-31
Hailey pose la question :
An isosceles triangle has one angle that measures 50 degrees and another that measures 70 degrees. Why can't this triangle be drawn?
Penny Nom lui répond.
An isosceles triangle 2010-01-28
Jazzy pose la question :
An isosceles triangle is a 2 congruent (equal) sides. Of the third side is three times the length of the congruent side(s), and the perimeter is 75 cm, find the length of all three sides of the triangle
Penny Nom lui répond.
The perimeter of a rectangle 2010-01-27
debbie pose la question :
how many dimensions are there to get a perimeter of 400 feet for a rectangle?
Penny Nom lui répond.
A triangle in a rectangle 2010-01-25
Sangya pose la question :
The perimeter of the rectangle is 21 inches. The perimeter of the inscribed triangle is 21 inches. Find the dimensions of the rectangle
Robert Dawson lui répond.
Solving a triangle 2010-01-25
Paige pose la question :
how do i solve a triangle with one angle of 73 degrees, one angle of 32 degrees, and one side of 23cm?
Harley Weston lui répond.
An altitude of the triangle intersect a side of the triangle 2010-01-23
Sarah pose la question :
In what type of triangle does an altitude of the triangle intersect a side of the triangle at its midpoint??
-Sarah

Walter Whiteley lui répond.
Angle of incline 2010-01-20
Alan pose la question :
how do I fnd the angle of an incline with a measurement of 0.042 with an adjacent of 1.2mtrs?. Thank You
Penny Nom lui répond.
Trigonometry and picture hanging 2010-01-13
george pose la question :
The top of a picture 1m high 0.8m from the ceiling. At a point on the ceiling directly in front of the picture, we wish to install a light so that the angle subtended by the picture equals to the angle of depression of the top of the picture. How far out from the wall should the light be installed?
Penny Nom lui répond.
The triangle inequality 2010-01-11
Nazrul pose la question :
If A and B are any two vectors, then how can I prove that magnitude of (A+B)<=magnitude of A+magnitude of B. Please show me the process in details. Thank you.
Robert Dawson lui répond.
The maximum area of a rectangle 2010-01-03
Mohammad pose la question :
determine the maximum area of a rectangle with each perimeter to one decimal place?
a)100 cm b)72 m c)169 km d)143 mm

Penny Nom lui répond.
Write a function for the area in terms of x 2010-01-01
Almarie pose la question :
Lauren has 40 feet of metal fencing material to fence three sides of a rectangular garden. A tall wooden fence serves as her fourth side. Write a function for the area of the garden A in terms of x, the width in feet.
Penny Nom lui répond.
Triangles on a base of 2.4 meters 2009-12-26
Allan pose la question :
Please,How do I calculate the height of a triangle when I only know the width of the base line,It is 2.4 mtrs.
Thankyou very much.

Penny Nom lui répond.
A triangle with two equal medians 2009-12-19
Nazrul pose la question :
If two medians of a triangle are equal , how can I prove that the triangle is isosceles.
Chris Fisher lui répond.
Houses on a triangular field 2009-12-18
Beth pose la question :
Hi, this is the question i have to answer. Planning permission has been granted to build houses on the triangular field in the diagram. The housing density permitted is 1 house for each 200m². Calculate how many houses can be built on this

Beth

Robert Dawson lui répond.
Polygons, diagonals and the sum of the measures of the angles 2009-12-18
jason pose la question :
find the set of polygons in which the number of diagonals is greater than the sum of the measures of the angles
Robert Dawson lui répond.
The acute angle formed by intersecting lines 2009-12-17
Katie pose la question :
To the nearest tenth of a degree, find the size of the acute angle formed by the intersecting lines 3x + 2y = 12 and x - 2y = -2. Was is necessary to find the intersection point?
Robert Dawson lui répond.
An equilateral triangle is inscribed in a circle 2009-12-06
anna pose la question :
An equilateral triangle is inscribed in a circle of radius 6. Find x and the length of one side of the equilateral triangle. The picture is a triangle where the corners touch the sides of a circle and there is a line drawn down the middle of the triangle. A point labeled D which is in the triangle but im pretty sure that its marking the radius of the circle. From that point D is a line going from that point to the bottom left corner of the triangle. So this line shall make another mini triangle. The bottom of the big triangle is then split into 2 segments and the left segment is labeled x. Please help for I am stuck!
Penny Nom lui répond.
The height of an isosceles triangle 2009-12-06
Carl pose la question :
What is the height of an isosceles triangle if its base is 12cm, and its base angle is 72degrees?
Penny Nom lui répond.
The dimensions of a rectangular room 2009-11-26
crystal pose la question :
. The length of a rectangular room is 7 feet longer than its width. The area of the room is 60 feet. What are the dimensions of the room?
Penny Nom lui répond.
The triangle formed by the tangent and the coordinates axes 2009-11-23
Nirmala pose la question :
Given that y=1/x, x is not equal to zero. Prove that the area of the triangle formed by the tangent and the coordinates axes is 2.
Harley Weston lui répond.
The three medians of a triangle 2009-11-18
Nazrul pose la question :
If the three medians of a triangle are equal, how can I prove (without using the property that the three medians of a triangle cut one another at a point of trisection) that the triangle is equilateral.
Chris Fisher lui répond.
The base and height of a triangle are in the ratio of 3:2 2009-11-18
Mike pose la question :
Find the base and height of a triangle using the ratio 3:2 and an area of 108 inches square. Is there a formula we need to use?

Thanks

Mike

Penny Nom lui répond.
Three angles and one side of a triangle 2009-11-16
Esther pose la question :
How do i find the sides of an acute triangle if i know the angels are 60,45,75 and i only know one side which is 10? Thanks!
Penny Nom lui répond.
The line through D(-4, 0) and E(2, 6) 2009-11-16
Rogerson pose la question :
The point F is on the line through D(-4, 0) and E(2, 6) so that DF=4DE. Find the coordinates of F.
Penny Nom lui répond.
The fourth vertex of a rectangle 2009-11-11
Ellena pose la question :
The points A(2,-1), B(5,-3), and C(7,0) are the three vertices of a rectangle. Determine he coordinates of the fourth vertex.
Penny Nom lui répond.
sinA + cosA = 1 2009-11-06
Nazrul pose la question :
What is the solution of sinA+cosA=1 where A is an acute angle? Will A=0 degree be included in the solution? Please explain.
Harley Weston lui répond.
Is it a rectangle? 2009-10-29
sefora pose la question :
The coordinates of the vertices of quadrilaterals are given. Draw each quadrilateral on a grid. Determine whether it is a rectangle (show your work)

P(5,1), Q(-4,4), R(-6,-2), S(3,-5)

Penny Nom lui répond.
A triangle problem 2009-10-28
Nazrul pose la question :
ABC is a triangle , D is the mid point of AB and DE is parallel to BC where E lies on AC. How can I prove that E is the mid point of AC.
Penny Nom lui répond.
Cutting a pipe at an arbitrary angle 2009-10-24
Carol pose la question :
I have read your article on cutting pipe,etc. at a 45 degree angle. I need to develop an equation and pattern for cutting any size pipe (3" to 7") to any degree. I don't understand how to transfer the wave pattern to graph paper.
Thanks.

Harley Weston lui répond.
The hypotenuse of a right triangle 2009-10-18
steven pose la question :
the perimeter of a right triangle is 20 cm. its area is 15 sq cm. find its hypotenuse.
Penny Nom lui répond.
The three angles of a triangle 2009-10-13
Michelle pose la question :
The second angle of a triangular garden is four times as large as the first. The third angle is 45 less than the sum of the other two angles. Find the measure of the other two angles?
Penny Nom lui répond.
The radius of an arc 2009-10-13
AYMAN pose la question :
Question from ayman:

i would like to know the radius of an arc , i do not know the angle , can u please explian it with an example ,

all i know , or i think i know is a draw line , the reason i asked yu guys this Q , is i am a boilermaker apprentice , sometimes i do deal with a bended pipes or flate plates of metal .

when i try to do the same bend as these pipes , it is quite hard so for me to find out the radius of the pipe already bended & find out the angle i will be able to know the full length of the material thats all , thank you

Robert Dawson lui répond.
A right triangle 2009-10-12
Josseph pose la question :
A right triangle has an area of 84ft sq. and a hypotenuse 25ft long. What are the lengths of the other two sides?
Penny Nom lui répond.
The area of a rectangle 2009-10-10
Ann pose la question :
given a rectangle with sides a and 2a and diagonal d, find the area of the rectangle as a function of the diagonal,d
Penny Nom lui répond.
Two rectangles 2009-10-08
Lillian pose la question :
A rectangle is 5cm longer than twice its width. The width of another rectangle is 3cm less than the width of the first rectangle and its length is 6cm more than 3 times its width. If the perimeters are equal, find the dimensions of both rectangles
Penny Nom lui répond.
The maximum number of right angles in a polygon 2009-10-05
Bruce pose la question :
Is there way other than by trial and error drawing to determine the maximum number of right angles in a polygon? Secondary question would be maximum number of right angles in a CONVEX polygon. Is there a mathematical way to look at this for both convex and concave polygons? Or are we limited to trial and error drawing?
Chris Fisher lui répond.
A right-angled triangle 2009-10-02
Hunter pose la question :
The three sides of a right-angled triangle measure x-1, x+6, and 2x+1 in length. What are the possible lengths of the hypotenuse?
Penny Nom and Melanie Tyrer lui répond.
A triangle on a sphere 2009-09-07
Rohit pose la question :
How do I find the angles of a triangle drawn on a sphere (spherical triangle)?
Chris Fisher lui répond.
Is -100 degree angle an acute angle? 2009-09-02
Nazrul pose la question :
Is -100 degree angle an acute angle? Please explain. Here -100<90.
Robert Dawson lui répond.
Zero Angle 2009-09-01
Nazrul pose la question :
Is zero degree an acute angle? Please explain.
Janice Cotcher lui répond.
The area of a rectangle 2009-08-21
roxanne pose la question :
the dimensions of the rectangle: 3x length and 4x width. What is the area of the rectangle in square units?
Penny Nom lui répond.
A paper towel roll 2009-08-19
Jeff pose la question :
I am making a spiral tube with paper that is 2" in dia. and 102" long I will be using paper that is slit 3" wide how many lineal feet of paper will I need to to cover the 102" I will be using 3 rolls of paper that will over lap the other by half to make a hard tube (paper core) in a roll of paper towels Thanks Jeff
Penny Nom lui répond.
Find the central angle 2009-08-18
Larissa pose la question :
In a circle, the length of a chord AB is 4 cm and the length of the arc AB is 5 cm. Find the central angle theta, in radians, correct to four decimal places. Then give the answer to the nearest degree. I think I'm supposed to use Newton's method, but am not sure how to start with this problem.
Harley Weston lui répond.
Solve for s 2009-08-12
Michelle pose la question :
My daughter and I are having trouble with this problem...All it says i solve thr formula for the indicated variable.
Height of an Equilateral Triangle.

Solve for s:

H= square root of 3/2 * S

Penny Nom lui répond.
An isosceles triangle 2009-08-09
Megan pose la question :
Find the perimeter of an isosceles triangle with a verticle angle of 100 degrees and a base of 25 cm.

I think I could answer this if I knew what a verticle angle was.
Thanks

Stephen La Rocque lui répond.
Eye Height Level Found from Visual Angle 2009-08-07
Jolie pose la question :
Hello, I am trying to figure out what height from the ground a person's eyes are if they have a 12 degree visual angle to a screen that is 58cm away from their eyes? Thank you.
Janice Cotcher lui répond.
Constructing an isosceles triangle 2009-07-27
Sanjay pose la question :
How to construct an isosceles triangle ABC with a base BC = 5.4 centimetre and altitude AD = 4.7 centimetre? Please help.
Janice Cotcher lui répond.
How can I trisect an angle? 2009-07-27
Nazrul pose la question :
How can I trisect an angle?
Robert Dawson lui répond.
Finding Supplementary Angles 2009-07-24
emir pose la question :

Robert J. Dawson lui répond.
The sides of a triangle 2009-07-08
Susan pose la question :
The second side of a triangle is 5ft more than the length of the first side. The third side of the triangle is 10ft less than the length of the first side. The perimeter of the triangle is 61ft. Find the lengths of each side of the triangle.
Thank you so much!

Penny Nom lui répond.
The area of a triangle 2009-07-02
Rory pose la question :
I'm trying to find the area of a triangle with points -2, 1; 4,1 and 7,4
Penny Nom lui répond.
A diagonal of a rectangle 2009-07-01
Ruben pose la question :
Please assist me with the following problem: A rectangle has a diagonal that is 3.6 feet longer than the length and 7.1 feet longer than the width. What are the dimensions of the rectangle? Thanks
Harley Weston lui répond.
Finding an Acute Angle using Trigonometric Identities 2009-06-29
Nazrul pose la question :
How can I find the value of A if sinA-cosA=1 , where A is an acute angle.
Stephen La Rocque lui répond.
Two questions from math class 2009-06-18
Con pose la question :
Hello,

My name is Con and my son is required to answer the following questions for his maths class.

He has attempted Q1 through trial and error and has found the answer to 72453. Is this correct?

He has attempted to draw the triangles described in Q2 in a number of ways and has found that BE can not equal ED and is dependent of angle BAC. Therefore, he claims that the triangle can not be drawn/practical. Is this correct or is there a slolution?

Q1.
Digits 2, 3, 4, 5 and 7 are each used once to compose a 5-digit number abcde such that 4 divides a 3-digit number abc, 5 divides a 3-digit number bcd and 3 divides a 3-digit number cde. Find the 5-digit number abcde.

Q2.
Let ABC be a triangle with AB=AC. D is a point on AC such that BC=BD. E is a point on AB such that BE = ED = AD. Find the size of the angle EAD. Con

Chris Fisher lui répond.
Stones around a pool 2009-06-01
Jessica pose la question :
I am putting a yard pond in and I am trying to find out how many stones I would need for the perimeter.

The pond is 4ft (W) by ft (L) by 1.5ft (D)

Thank You,

Jessica

Penny Nom lui répond.
The dimensions of a rectangle 2009-05-30
Linda pose la question :
A rectangle has a perimeter of 38 feet & an area of 48 square feet. What are the dimensions of the rectangle?
Stephen La Rocque lui répond.
I'm building a fence at my house. 2009-05-24
dylan pose la question :
I'm building a fence at my house. If my hypotenuse is 50ft, what are the lengths of my other two legs?
Penny Nom lui répond.
The position of the fulcrum 2009-05-23
jim pose la question :
I think I need a formula. I need to know how far an object will be lifted. A beam is 246 inches long on one side of the fulcrum, and 41 inches on the other side, if I push down 36 inches on the long side of the beam, how much will the short side move up?
Stephen La Rocque lui répond.
Application of Derivatives of Trig Functions 2009-05-21
Alannah pose la question :
I have a word problem from my Calculus textbook that I can't figure out. Triangle ABC is inscribed in a semicircle with diameter BC=10cm. Find the value of angle B that produces the triangle of maximum area. I am supposed to set up an equation for the area of the triangle A=b x h/2 using Trig functions based on angle B to represent the base and height but I'm not sure how to do this when the side length given is not the hypotenuse.
Janice Cotcher lui répond.
The width of a rectangle 2009-05-19
Stacey pose la question :
If the length of rectangle is fixed at 29 cm, what widths will make the perimeter greater than 82 cm?
Penny Nom lui répond.
The sum of the angles of a triangle 2009-05-18
mary pose la question :
prove that the sum of the three angles inside any triangle always add up to 180 degrees?
Harley Weston lui répond.
Angles in a triangle 2009-05-16
Robert pose la question :
The second angle of a triangle is twice the sizee of the first angle, the third angle is 48 degrees less than the sum of the other two angles. What are the measurements of all 3 angles? PS: please help
Stephen La Rocque lui répond.
Finding Height of a Triangle Given Area and Base 2009-05-14
theodore pose la question :
area of a triangle is 40 in ^2 and base is 4 in what is the height
Robert Dawson lui répond.
The hypotenuse of a right angled triangle 2009-05-11
Deb pose la question :
Find the length of the hypotenuse of a right angled triangle with one leg 7 cm longer than the other and the hypotenuse 2 cm longer than the longer leg. I've ended up with the hypotenuse = x+9, another side = x and the other side = x+7. what do i do next?
Penny Nom lui répond.
A hexagon inside a triangle 2009-05-03
Brian pose la question :
How do you figure out the area of an equilateral triangle with only the clue of a regular hexagon perfectly inside it (no boarders) and the only thing you know about the hexagon is it has an area of 12cm^2?
Stephen La Rocque lui répond.
A max-min problem 2009-04-20
Charlene pose la question :
A fixed circle lies in the plane. A triangle is drawn inside the circle with all three vertices on the circle and two of the vertices at the ends of a diameter. Where should the third vertex lie to maximize the perimeter of the triangle?
Penny Nom lui répond.
The interior angles of a pentagon 2009-04-20
Mary pose la question :
If four interior angles of a five-sided figure pentagon measure 100 degrees each, what will the fifth angle measure?
Harley Weston lui répond.
Two similar rectangles 2009-04-19
Alyssa pose la question :
The ratio of the lengths of corresponding sides of two similar rectangles is 3:5. the small rectangle has an area of 36 square centimeters. What is the are of the large rectangle?
Penny Nom lui répond.
The height of a right triangle 2009-04-17
drakkar pose la question :
how do i find the height of a right triangle that in 176 feet in area
Penny Nom lui répond.
The central angle of a chord 2009-04-06
Dale pose la question :
How do I find the central angle if I only have the cord length and radius.
Harley Weston lui répond.
An isosceles triangle 2009-03-26
sela pose la question :
An isosceles triangle has two equal sides of length 10 cm. Theta is the angle between two equal sides.
a) Express area of a triangle as a function of theta
b) If theta is increasing at a rate of 10 degrees/minute, how fast is area changing at the instant theta=pi/3?
c) at what value of theta will the triangle have the maximum area?

Penny Nom lui répond.
Depth to height ratio 2009-03-26
Janet pose la question :
Is there a formula to determine how deep something (a cabinet) should be based on how tall it is?
Robert Dawson lui répond.
The base of a triangle 2009-03-25
yinna pose la question :
how do I find the base of a triangle who's height is 2 times the base and the area is 9?
Penny Nom lui répond.
A 1.5:1 rectangle 2009-03-23
melody pose la question :
I have an origami model that says to use a 1.5:1 rectangle. I don't understand how to calculate the size of paper I need to use. Surely there is a formula to calculate this ratio? Thanks for your help.
Penny Nom lui répond.
The base and height of an isosceles triangle 2009-03-23
Chris pose la question :
How do you find the base and height of an isosceles triangle that has 2cm legs?
Harley Weston lui répond.
The height of a triangle 2009-03-20
jon pose la question :
hi,
my daughter has to find the height of a triangle her side are 174ft and 184ft and the area is11,500 ft squared.
thanks for the help
jon

Penny Nom lui répond.
The perimeter of a rectangle 2009-03-18
marylyne pose la question :
the perimeter of the rectangular is 70 meters. if the width is 40% of the length, find the dimensions?
Penny Nom lui répond.
The midpoints of two sides of a triangle 2009-03-17
Manis pose la question :
Prove that the line joining the midpoint of two sides of a triangle is parallel to the third and half of it.
Robert Dawson lui répond.
Related rates 2009-03-14
Jeevitha pose la question :
The side of an equilateral triangle decreases at the rate of 2 cm/s. At what rate is the area decreasing when the area is 100cm^2?
Stephen La Rocque lui répond.
The angles of a triangle 2009-03-11
Marissa pose la question :
The angles in a triangle measure 7x-1, 18x+2, and 5x+10. Determine whether the triangle is acute, obtuse, or right. State your reasons clearly.
Robert Dawson lui répond.
A right angle 2009-03-09
bill pose la question :
I am a cotractor & i am laying out some piers in an L shape i need to know how to tell if they are square one side of my L is 18 feet the other is 11 feet 6 inches what is the formula to find out what the distance from point to point should be for square . I appreciate any help u can give me thank you
Penny Nom lui répond.
The height of a right triangle 2009-03-08
Anah pose la question :
Could you please tell me the formula for me to calculate the height of a triangle. I have the angles and side lengths. I am trying to calculate the height of an right triangle, the dimensions are as follows:

The Base/Hypotenuse is: 8x

Side A is 2x. Side B is 2x.

Harley Weston lui répond.
The sides of a triangle 2009-03-07
felicia pose la question :
ona map, tannersville, chadwick, and barkersvile for a triangle. Chadwick is 70 miles from Tannersville and Barkersville is 90 miles from Tannersville. Which is a possible distance between Chadwick and Barkersville?
a. 5miles
b. 10 miles
c. 150miles
d. 200 miles

Harley Weston lui répond.
Is ACB a right angle? 2009-03-02
racheal pose la question :
how to i prove angle ACB is not a rt. angle when given AT=BT=5;CT=4 in a indirect proof?
Robert Dawson lui répond.
Triangles within triangles 2009-02-24
Mari pose la question :
a large shaded triangle changes each day with a white triangle appearing in the center of each shaded one. If this pattern continues, how many white triangles will be there on the 6th day? On the 6th day, what fraction of the large outer triangle will be white?THANKS!
Robert Dawson lui répond.
Trig functions without geometric data 2009-02-24
bob pose la question :
I do not understand how it is possible to find the sine, cosine, or tangent of an angle if there is no hypotenuse, opposite or adjacent side?!
Robert Dawson lui répond.
Angle of depression 2009-02-18
Meeka pose la question :
An aircraft flying at an altitude of 2000m is approaching an airport. If the angle of depression of the airport is 5 degrees, what is the distance from the plane to the airport measured along the ground? Round your answer to the nearest tenth of a kilometer.
Robert Dawson lui répond.
The height of a triangle 2009-02-14
GARIMA pose la question :
IF THE SIDES OF TRIANGLE ARE GIVEN . HOW WE WILL CALCULATE THE HEIGHT OF TRIANGLE.
Penny Nom lui répond.
Forming an arc with 2 inch steel 2009-02-11
Craige pose la question :
I need to calculate the bending radius of 2" wide steel to achieve given inside and outside arc lengths
Harley Weston lui répond.
A rectangular park 2009-02-06
Debbie pose la question :
A park named Writer's Rectangle opened in town. When asked about the dimensions of the rectangle, the city planner, responded with these clues: ---The diagonals of the rectangular park plus its longer sides together measure seven times one of the shorter sides. ---The length of one diagonal is 250 m longer than one of the shorter sides. Use this information to find the area of the park.
Stephen La Rocque lui répond.
The height of a triangle 2009-02-06
lisa pose la question :
find the height of a triangle, which can be split into two right triangles, but the base (4') is not split equally in half. No interior angles are given.
side a=3', side b=5'

Penny Nom lui répond.
The height of a triangle 2009-02-05
Theresa pose la question :
Find the area of a triangle by first finding the height. base30ft hypotenuse50ft
Penny Nom lui répond.
A 30-60-90 triangle 2009-02-03
Inez pose la question :
If you have a 30-60-90 triangle and the only side you get is 73 and a 90 degree box, how do you find the area?
Penny Nom lui répond.
The height of an isosceles triangle 2009-01-29
Mariah pose la question :
An isosceles triangle has sides 10cm, 10 cm and the base 4cm. How do you find out the height of the triangle?
Penny Nom lui répond.
Find area of the rectangle in terms of L. 2009-01-27
Arlen pose la question :
The length of a rectangle is L. The width is 8 less than the length. Find area of the rectangle in terms of L.
Penny Nom lui répond.
The area of a triangle 2009-01-25
black pose la question :
Explain how the formula for the area of a triangle can be determined by using the formula for the area of a parallelogram.
Penny Nom lui répond.
1 foot drop every 25 feet 2009-01-22
jerry pose la question :
1 foot drop every 25 feet what is the angle of the degree
Penny Nom lui répond.
The height of a triangle 2009-01-20
ELEY pose la question :
WHAT IS THE HEIGHT OF A TRIANGLE IF THE AREA IS 32.4 AND A BASE OF 6.75?
Robert Dawson and Penny Nom lui répond.
A triangle with sides of length 6 and 7 2009-01-20
bill pose la question :
If I have a triangle with sides 6 and 7, which of the following is a possible area? The answers are 13 and 21, but not 24. I know this is because of a possible range of heights, but how do I calculate the range of heights? This is an SAT question. Thanks
Chris Fisher lui répond.
The area of an isosceles triangle 2009-01-19
faris pose la question :
how to calculate area of isosceles triangle with two angles of 45 degree and base of 6
Penny Nom lui répond.
The ratio of the measures of the three angles of a triangle 2009-01-18
mary pose la question :
The ratio of the measures of the three angles of a triangle is 2:5:8. What is the measure of each angle of the triangle?
Robert Dawson and Harley Weston lui répond.
The dimensions of a pool 2009-01-18
denise pose la question :
a rectangular swimming pool is three times as long as it is wide. if the perimeter of the pool is 320 feet, what are its dimensions
Penny Nom lui répond.
A stack of rectangles 2009-01-11
ashwani pose la question :
Could you please advise the area of a rectangles pyramid. There are 5 rectangles placed one above the other. The top most rectangle has length 2 cm and height 2 cm. The subsequent rectangles length is increased by 1 cm on both the sides, while the height of the 2nd and 3rd rectangle is 2cm each, while the 4th & 5th has a height of 3 cm each. Could you please let me know the the area of the figure and the formula to calculate
Penny Nom lui répond.
A flat-topped pyramid 2009-01-10
Tom pose la question :
I am planning to build an open ended, flat-topped pyramid with a rectangular base 28 x 36 in.; a square top 6 x6 in.; and a vertical height of 16 in.

I know how to calculate the dimensions of each of the pyramid sides but I don't know how to calculate the interior angles when I bend the sheet metal. I will cut the four sides separately and add a one inch flange to each "vertical" edge of the two larger sides to allow for a bonding surface for the assembly. How do I calculate that angle? I know it will be greater than 90 degrees but by how much?

Harley Weston lui répond.
Two angles are supplementary 2009-01-08
Stephanie pose la question :
two angles are supplementary, one of the angles is 30 degrees more than double the other angle. find the first angle, the second angle the complement of the given angle.
Robert Dawson lui répond.
Two rectangular plots have the same area 2008-12-23
Jason pose la question :
Two rectangular plots have the same area. The first has a perimeter of 40m while the second has a length 2m less than the first and a width 1m greater. Find the length and breadth of the first rectangle
Stephen La Rocque lui répond.
The area of a triangle 2008-12-23
ashiya pose la question :
Hi,

I have attached figure of the triangle, Can you please help to find the area of Triangle ABC.

Thanks.

Ashiya

Stephen La Rocque lui répond.
Pouring angles for a crucible 2008-12-20
Richard pose la question :
I am trying to work at pouring angles and volume left in during pouring a crucible, The crucible is cylindrical and flat bottomed.

I know the diameter, radius and volume of the crucibles. and the volume of liquid going into it.

So lets say the crucible is only half full firstly I need to work out the angle just before its going to pour. ( I can work this out as long as there is a certain volume of liquid if its not enough I cant do it)

Now the problem I also need to work out how much I should tilt the crucible to allow a certain amount out and be able to do this untill the volume reaches 0 at 90' turn. This is where I am stuck.

The reason for needing to be able to work this out is so i can develop a constant flow for example 10Kg of metal per second. Thank you very much for you time

Harley Weston lui répond.
A standard bill on a highway 2008-12-19
Stephanie pose la question :
A standard bill on a highway has a perimeter of 14 feet. the length of the bill board is 6 feet greater than triple the width. What is the length? what is the width?
Robert Dawson and Penny Nom lui répond.
The volume of a feed hopper 2008-12-18
John pose la question :
I need to calculate the volume of a feed hopper, and I'm not sure how to break it down. The top of the hopper is 36" x 36", it is 30" deep, and ends at a 6" x 6" plate. One side of the hopper is straight top to bottom, of course tapering on two sides to meet at the plate. The other three sides angle down at about 75 degrees. I need to determine the cubic foot volume of this hopper (it is used for ground coffee) so I can configure a vibrator to knock down residual grounds. Thanks.
Robert Dawson lui répond.
The angle between two lines 2008-12-17
abhi pose la question :
how to calculate the angle between two lines, given the length of the lines.. angle should vary from 0 - 360 in the counterclockwise direction
Robert Dawson and Harley Weston lui répond.
An equilateral triangle 2008-12-17
lorraine pose la question :
an equilateral triangle has side lenghts of10.the length of its altitude is?
Penny lui répond.
A rectangle with diagonal 8.5 inches 2008-12-14
nekasia pose la question :
what is the area of a rectangle with a diagonal of 8.5 inches PLEASE SHOW WORK
Penny Nom lui répond.
The third vertex of a right triangle 2008-12-10
prashant pose la question :
how to calculate coordinates of third vertex in a right angle triangle given the coordinates of hypotenuse as (1,3) and (-4,1)
Robert Dawson lui répond.
The centroid and circumcircle of a triangle 2008-12-09
prateet pose la question :
in an equilateral triangle prove that the centroid and centre of the circumcircle coincide

here i am not clear about the concept of centroid and circumcircle i cant understand how AGis 2/3 AD. please help in details about the topic mentioned.

Harley Weston lui répond.
The sides of a rectangle 2008-12-08
Mitchell pose la question :
The perimeter of rectangle ABCD (represented on a Cartesian plane) is 52cm. The equation associated with segment AD is 3x+4y-60=0 Point C is at (18, 14). What is the length of segment AD?
Penny Nom lui répond.
The area of an isosceles triangle 2008-12-02
prateet pose la question :
the area of an isosceles triangle is 60 sq cm and one of its equal sides is 13 cm. Find the base of the triangle.
Penny lui répond.
3 equidistant points 2008-12-01
Damien pose la question :
How do you find 3 equidistant points (C,D,E) on a line between point A(Xa, Ya) and point B(Xb, Yb) so that AC, CD, DE and EB are all equal?
Chris Fisher lui répond.
The height, base and perimeter of a triangle 2008-11-17
Zack pose la question :
How would i calculate the height of a triangle if the base is 10cm and the area is 50cm²? If possible how would i calculate the length around the the triangle.
Penny Nom lui répond.
A triangle inscribed in a circle 2008-11-17
Wanda pose la question :
I have the same question that you guys answered 2007-03-02. I need more clarification. I UNDERSTAND how to get the radius=3, I get that it is an equilateral triangle so each vertex is 60 degrees, I get that area of triangle is 1/2 bh. I DO NOT understand why we multiply area X 3 , or how to calculate the values of base and height. Please explain a little further. Thanks.
Wanda

Penny Nom lui répond.
The midpoint o a line segment 2008-11-15
Jane pose la question :
The vertices of a triangle are at (1,7), (6,-1) and (0,3). Find the coordinates of the midpoints of the sides.
Penny Nom lui répond.
The angles and sides of a triangle 2008-11-13
JAMIE pose la question :
a triangle with a side(b)37m an angle(C)70degrees and (a)79m find values of angles A and B and length of side c
Stephen La Rocque lui répond.
Supplementary angles 2008-11-10
Anna pose la question :
<A and <B are supplementary angles. Twice the measure of <B is one-fourth the measure of <A. Find the measure of both angles.
Penny Nom lui répond.
A flagpole and a yardstick 2008-11-07
Wanda pose la question :
One boy holds a yardstick vertically at a point 40 feet from the base of the flagpole. The other boy backs away from the pole to a point where he sights the top of the pole over the top of the yardstick. If his position is 1 ft 9 in from the yardstick and his eye level is 2 ft above the ground, find the height of the flagpole.
Penny Nom lui répond.
The dimensions of a rectangle 2008-11-07
laela pose la question :
A rectangle is 8 feet long and 6 feet wide.If the dimension is increased by the same number of feet,the area of the new rectangle formed is 32 square feet more than the area of the original rectangle.By how many feet was the each dimension increased?
Penny Nom lui répond.
The height of an equilateral triangle 2008-11-06
touqeer pose la question :
My question is that how can we find the height of an equilateral triangle without using pythagoras theorem?
Penny Nom lui répond.
The angles of a triangle 2008-10-31
kyla pose la question :
an isosceles triangle has two 50 degrees angles.what is the measure of the third angle? explain how you found your answer.
Penny Nom lui répond.
Three congruent rectangles 2008-10-27
Meagan pose la question :
Here is my problem: Three congruent (non-square) rectangles are placed to form a larger rectangle. [Two are oriented the same way and the "stacked" while the third is rotated 90 degrees and placed next to the other two.] The total area is 1350 square cetimeters. Square ABCD is then created that has the same perimeter as the large rectangle that was created. E is the midpoint of line CD and F is the midpoint of BC. Find the area of triangle AEF.
Penny Nom lui répond.
The length of a rectangle is 6 inches less than 3 times its width. 2008-10-26
Angela pose la question :
The length of a rectangle is 6 inches less than 3 times its width. Find the dimensions of the rectangle if its area is 45 square inches.
Penny Nom lui répond.
Area of a triangle 2008-10-22
Jia pose la question :
I need to find 3 ways of finding the area of a triangle. I've already had one method down which is b x h and divided by 2. What other 2 ways can i find the area of a triangle?
Chris Fisher lui répond.
A geometric construction 2008-10-17
M pose la question :
Given any 3 parallel lines on a plane, how to construct an equilateral triangle with each vertex on each line?
Chris Fisher lui répond.
The dimensions of a rectangle 2008-10-16
cristine pose la question :
the area of a rectangle is 675sq.cm. if the width is 1/3 of the length. find the dimensions of the rectangle....
Penny Nom lui répond.
Octagon angles 2008-10-06
John pose la question :
My daughter wants to build an octagon using PVC pipe fittings. There are several pipe "elbows" available, including 45 degree. I'm trying to determine if if an octagon can be constructed by using 8 pairs of 45 degree elbows, connected to each other by 8 sections of straight pipe of some equal legnth. This makes sense to me intiutively, but I do not know how this can be properly expressed in geometric terms.
Chris Fisher, Victoria West and Harley Weston lui répond.
The height of a right triangle 2008-10-06
David pose la question :
Could you please tell me the height of a right angle triangle when the base measurement is 4930mm and the base/hypotenuse angle is 10 degrees. this will help a lot.
Harley Weston lui répond.
An arc and an angle 2008-10-05
Cory pose la question :
I have a chord length of 150'. From the left starting point, I know that 30' right of starting point is 9' in height. This would be the top of the arc.

What is the arc in degrees and can anyone display an image to help me understand?

Penny Nom lui répond.
Finding the Distance Between Two Latitudes 2008-10-02
Samua pose la question :
Assuming that the Earth is a sphere of radius 4000 miles and that the cities are on the same longitude (one city is due north to the other). Find the distance between the cities with the latitudes of 37 degrees 47'36'' and another city with 47 degrees 37'18''. Heeeeeeeeeeelp!
Janice Cotcher lui répond.
The perimeter of a triangle 2008-09-29
Debbie pose la question :
How do I calculate the perimeter of a triangle if the lengths of two sides are equal and all the angles are known?
Harley Weston lui répond.
How tall is the wall? 2008-09-29
ash pose la question :
you and bob are separated by a tall wall you stand 10 feet further from the wall than bob your angle of elevation is 37 degrees and his 44 degrees how tall is the wall?
Penny Nom lui répond.
What is the perimeter of the rectangle? 2008-09-27
Pauline pose la question :
A rectangle has an area of 27 squared inches. the height is 3 times the base What is the perimeter of the rectangle?
Penny Nom lui répond.
The vertices of a triangle 2008-09-27
T. pose la question :
how do you find the coordinates of the vertices of the triangle with the sides determined by the graphs of the following equations:4x+3y+1=0,4x-3y-17=0,4x-9y+13=0?
Harley Weston lui répond.
The base of a triangle is twice its height 2008-09-26
carolyn pose la question :
The length of the base of a triangle is twice its height. If the area od the triangle is 100 square kilometers, find the height.
Penny Nom lui répond.
A right triangle 2008-09-25
john pose la question :
a right triangle has hypotenuse wich measures 20cm and a perimeter 47cm. find the measure of the remaining two sides
Harley Weston lui répond.
Consecutive Even Integer Sides of a Triangle 2008-09-24
Brad pose la question :
I am having a problem coming up with a formula for my son's eighth grade math problem. We have found the answer by guess and check but have a mental block on the equation. Any help would be appreciated. The problem is: A right triangle has sides whose lengths in feet are consecutive even intergers. Determine the length of each side. Thanks
Janice Cotcher lui répond.
The height and base of a triangle 2008-09-12
Bailey pose la question :
The length of the base of a triangle is 1cm less than 5 times the height of the triangle. The area of the triangle is 21 square cm. What is the the height of the triangle and the length of the base of the triangle?
Stephen La Rocque lui répond.
Cutting a pipe on a 45 degree angle 2008-09-12
Bakshani pose la question :
how do you a mark a pipe 5 inch diameter and cut it to form a 45 degree angel
Stephen La Rocque lui répond.
A garden in the shape of an isosceles triangle 2008-09-12
Rita pose la question :
Gregory wants to build a garden in the shape of an isosceles triangle with one of the congruent sides equal to 12 yards. If the area of his garden will be 55 square yards, find, to the nearest tenth of a degree, the three angles of the triangle.
Penny Nom lui répond.
The perimeter of an equilateral triangle 2008-09-11
Gerry pose la question :
How can I find the perimeter (length of side) of an equilateral triangle if the only information I have is the altitude?
Penny Nom lui répond.
Supplementary angles 2008-09-11
Shanaz pose la question :
Find an angle such that 3 times its supplement equals 450.
Penny Nom lui répond.
Angle of elevation 2008-09-09
kristy pose la question :
A man on the tenth floor of a building shouts down to a person on the street. If the angle of elevation from the street to the man in the building is 35° and the man in the building is 40 feet up, about how far away from the building is the person on the street?
Penny Nom lui répond.
The height of a tree 2008-09-09
danice pose la question :
At a certain time of day, the angle of elevation of the sun is 30°. A tree has a shadow that is 25 feet long. Find the height of the tree to the nearest foot.
Penny Nom lui répond.
The perimeter of a right triangle 2008-09-09
Shanon pose la question :
I know that A=1/2bh, but for the life of me cannot figure out my 6th graders math problem.

What is the perimeter of a right triangle whose area = 600 sq. ft.

Victoria West lui répond.
Finding the Speed of a Truck 2008-09-08
Rita pose la question :
A state trooper is hidden 30 feet from a highway. One second after a truck passes, the angle theta between the highway and the line of observation from the patrol car to the truck is measured. (a) If the angle measures 15 degrees, how fast is the truck traveling? Express the answer in feet per second and miles per hour.
Janice Cotcher lui répond.
An exclusion zone around a triangle 2008-09-07
Awrongo pose la question :
A long time ago Mr Gibson found an island shaped as a triangle with three straight shores of length 3 km,4 km and 5 km. He declared an 'exclusion zone' around his island and forbade anyone to come within 1 km of his shore. What was the area of his exclusion zone?
Stephen La Rocque and Penny Nom lui répond.
A security camera 2008-09-07
Rita pose la question :
A security camera in a neighborhhod carnival is mounted on a wall 9 feet above the floor inside a video gallery. What angle of depression should be used if the camera is to be directed to a spot 6 feet above the floor and 12 feet from the wall?
Stephen La Rocque and Harley Weston lui répond.
The length of an arc 2008-09-04
Angie pose la question :
Segment PR is a diameter of circle S. If angle P =3D 25, find minor arc QR. This circle has an isosceles triangle in it, it is connected to the diameter,
Harley Weston lui répond.
Largest Inscribed Rectangle 2008-09-03
astrogirl pose la question :
find the shape and area of the largest rectangle that can be inscribed in a circle of a diameter a=2
Janice Cotcher lui répond.
A triangle and an inscribed circle 2008-09-01
Nancy pose la question :
I'm a computer programming student, and I'm supposed to figure out how to find the area of a circle inside a triangle if someone types in the length of each side of the triangle.

So, a user can type in any three numbers they want into the three "side length" boxes, and I have to find the area of the circle that would fit inside the triangle they create from those values.

So the circle can be any size, depending on the size and shape of the triangle the user creates. The circle has to touch all three sides of the triangle somewhere. Then, my program calculates the area of the triangle and thus the area of the circle. I just need to know how the circle would change depending on the length of each side of the triangle that the user puts in. Is there a way to find out how the circle's area is related to the triangle around it?

Chris Fisher lui répond.
A roof angle 2008-08-30
carla pose la question :
Use vectors to find the angle in the attached diagram
Chris Fisher and Stephen La Rocque lui répond.
Number of angles formed by rays 2008-08-29
Shon pose la question :
what is the formula for finding the number of angels that can be named by a given number of rays with the same endpoint?
Stephen La Rocque lui répond.
The lengths of the sides of a rectangle are in ratio 5:3 2008-08-29
todd pose la question :
the lengths of the sides of a rectangle are in ratio 5:3 the perimeter of the rectangle is 32 cm find the length of each side
Stephen La Rocque lui répond.
Angles in an octagon 2008-08-26
Arvie pose la question :
I have seen your answer to Kay on how she can find the length for the sides of an octagon for a 4 foot window, but what is the angle that will need to be cut to get to 4 feet
Penny Nom lui répond.
The area of a triangle 2008-08-25
Beverly pose la question :
What is the area of a triangle on a graph with the following points:
18,110
6, 110
12, 100

Penny Nom lui répond.
Radii and Chords Create a Non-Right Triangle 2008-08-22
Beary pose la question :
AOC is a diameter of circle O. Line AB is 12, and lines OA and OC (the radii) are 10. Find the length of line BO and chord BC.
Janice Cotcher lui répond.
Joining vertices in a polygon 2008-08-18
Megan pose la question :
I'm trying to develop a relationship between the number of points of a regular polygon and the
(a) number of lines you could draw between those points,
(b) number of triangles you could draw,
(c) number of quadrilaterals,
etc.

Penny Nom lui répond.
Solving for Shared Height of Two Right Triangles 2008-08-17
Heidi pose la question :
find the height of a triangle, which can be split into two right triangles, but the base (50m) is not split equally in half. one end of the base is 40 degrees, while the other is 30 degrees.
Janice Cotcher lui répond.
TRIG IDENTITIES 2008-08-17
VINCE pose la question :
HOW DO YOU SIMPLIFY THE FOLLOWING:- 2sin3(theta)cos3(theta)? Regards
Harley Weston lui répond.
Similar Triangles Given a Median 2008-08-17
RAM pose la question :
Sides AB , AC and median AD of a triangle ABC are respectively proportional to PQ PR and median PM of another triangle PQR. Show that ABC is similar to PQR
Janice Cotcher lui répond.
Similar triangles 2008-08-12
ramarao pose la question :
D is a point on side BC of a triangle ABC such that BD/CD=AB/AC. prove that AD is the bisector AD is the bisector of BAC
Chris Fisher lui répond.
The base of a triangle 2008-08-10
Mark pose la question :
What is the formula I need to use to find the base of a triangle sail that is 16 feet high and has an area of 64 square feet.
Stephen La Rocque and Penny Nom lui répond.
Angles in a regular tetrahedron 2008-08-07
Carla pose la question :
Hi guys, A regular tetrahedron has all its edges 8cm in length. Find the angles which an edge makes with the base. Thanks. Carla
Penny Nom lui répond.
Arc-length and sector-angle 2008-08-06
Benson pose la question :
If chord length, radius are given, How to find the sector angle and arc-length
Janice Cotcher lui répond.
The angle between two faces of a pyramid 2008-08-06
Carla pose la question :
A pyramid has its vertex directly above the centre of its square base. The edges of the base are each 6cm, and the vertical height is 8cm. Find the angle between 2 adjacent slant faces.
Penny Nom lui répond.
Some angles in a pyramid 2008-08-04
Carla pose la question :
A symmetrical pyramid stands on a square base of side 8cm. The slant height of the pyramidis 20cm. Find the angle between the slant edge and the base, and the angle between a slant face and the base.
Penny Nom lui répond.
Area of triangle formed by three tangent circles 2008-07-31
brian pose la question :
Three circles with radii 3,4 and 5 touch each other. The circles are tangent to each other. What is the area of the triangle formed by the centers of the circles?
Stephen La Rocque lui répond.
Finding Angles in a Pyramid 2008-07-30
Carla pose la question :
A pyramid has its vertex directly above the centre of its square base. The edges of the base are each 8cm, and the vertical height is 10cm. Find the angle between the slant face and the base, and the angle between the slant edge and the base.
Janice Cotcher lui répond.
Inscribed Rectangle 2008-07-30
Felicia pose la question :
A rectangle whose base is twice its altitude is inscribed in a circle whose radius is 5 mm. Find the area of the rectangle.
Penny Nom lui répond.
An equilateral triangle 2008-07-21
Meagan pose la question :
An equilateral triangle has vertices at (0,0) and (6,0) in a coordinate plane. What are the coordinates of the third vertex? You may want to sketch it out. Note: The sides of an equilateral triangle are identical in length.
Harley Weston lui répond.
The height of a triangle 2008-07-18
Becc pose la question :
I have to find the height of a triangle.
The base is 3.1cm say ab
One of the top sides is 4cm say ca
The other side is 2cmsay cb

Penny Nom lui répond.
Graphing Using Double Angle Identities 2008-07-16
Hodan pose la question :
the Question is: Describe how you could use your knowledge of Double angle formulas to sketch the graph of each function. Include a sketch with your description.A) F(x)=sin x cos x B)F(x)=2 cos(squared)x C) F(x)= tan(x) (divided) by 1-tan(squared) x
Janice Cotcher lui répond.
An isosceles triangle inscribed in a circle 2008-07-15
Anne pose la question :
Here is the math problem quoted from book:
"An isosceles triangle is inscribed in a circle of radius R, where R is a constant. Express the area within the circle but outside the triangle as a function of h, where h denotes the height of the triangle."

Penny Nom lui répond.
The area of a triangle - no height given 2008-07-12
phoenix pose la question :
given triangle with angles 30,105 45 degrees. base = 2radical 2, no height given
Penny Nom lui répond.
Hexadecagon 2008-07-09
Dave pose la question :
I would like to know at what length and angle I would cut wood to make a Hexadecagon.
Penny Nom lui répond.
Two triangles and a circle 2008-07-03
Anita pose la question :
An equilateral triangle with side of length 1 cm is inscribed in a circle. A second equilateral triangle is circumscribed about the circle with all sides tangent to the circle. Find the length of a side of the second triangle.
Harley Weston lui répond.
Fit 7 different rectangles on grid paper 2008-07-02
anonomous pose la question :
I need to fit 7 different rectangles on grid paper of 1cm squared. Each squared centimetre equals 4 squared metres, and it needs to end up as area 64 square metres.
Janice Cotcher lui répond.
The exterior angles of a polygon 2008-06-26
evelina pose la question :
how many sides has a regular polygon if the measures of the exterior angle is given
Walter Whiteley lui répond.
A rectangle 2008-06-26
SEBASTIAN pose la question :
How to arrange quadrilateral in the form of a rectangle?
Penny Nom lui répond.
The height of an isosceles triangle 2008-06-22
Evelyn pose la question :
Hi! I am facing problem finding the height of the isosceles triangle.
Penny Nom lui répond.
sin(2x)/sin(3x) 2008-06-19
matt pose la question :
how does sin2x break down (not with identities) and how would sin3x be created. My prob. is sin 2x/ sin 3x and I want to know how the double(or triple angle) would break down. I want to be able to cancel out sins. Thanks!
Harley Weston lui répond.
A space camera circles the Earth 2008-06-16
Rita pose la question :
A space camera circles the Earth at a height of h miles above the surface. Suppose that d distance, IN MILES, on the surface of the Earth can be seen from the camera.

(a) Find an equation that relates the central angle theta to the height h.

(b) Find an equation that relates the observable distance d and theta.

(c) Find an equation that relates d and h.

(d) If d is to be 3500 miles, how high must the camera orbit above Earth?

(e) If the camera orbits at a height of 400 miles, what distance d on the surface can be seen?

Penny Nom lui répond.
The length and width of a rectangle 2008-06-11
gregg pose la question :
the perimeter of a rectangle is 70 feet. the ratio of length to width is 4:3. how would i calculate the length of the height and width?
Penny Nom lui répond.
The angles of a parallelogram 2008-06-11
light pose la question :
ABCD is a parallelogram. If angle A. = (4x=17)degrees and angle B. = (3x-5)degrees , what is angle c?
Penny Nom lui répond.
What are the dimensions of the rectangle? 2008-06-11
Denise pose la question :
A rectangle has a diagonal that is 3.6 feet longer than the length and 7.1 feet longer than the width. What are the dimensions of the rectangle?
Claude Tardif and Victoria West lui répond.
The two acute angles in a right triangle 2008-06-05
Crystal pose la question :
In the right triangle the ratio of the measure of the two acute angles is 4:1. What is the measure in degrees of the larger acute angle?
Penny Nom lui répond.
The interior angles of an 18-sided polygon 2008-06-04
Alfred pose la question :
How do I find the sum of the interior angles of an 18-sided polygon?
Janice Cotcher lui répond.
The area of a rectangle 2008-05-27
Becky pose la question :
the area of a rectangular lot is represented by 6t^2+13t-5. What are the length and the width of the rectangular lot?
Harley Weston lui répond.
A triangle and a square 2008-05-24
s pose la question :
how to construct a triangle whose area is equal to the area of a given square?
Penny Nom lui répond.
What is the length of the 3rd side? 2008-05-23
lee pose la question :
If you have a triangle 12ft across the bottom then on the right is a 90 degree angle and that side is 18ft tall. What is the length of the 3rd side?
Penny Nom lui répond.
The angles of an irregular pentagon 2008-05-20
victoria pose la question :
The sum of the measure of two angles is 240. If the remaining angles are congruent, what is the measure of each angle?
Penny Nom lui répond.
A triangle inscribed in a semicircle 2008-05-19
Larissa pose la question :
Find the area of the shaded region outside of a triangle inscribed (meaning the all three points of the triangle are on the circle ) in a half circle of diameter 10 inches, if one side of the triangle is the diameter and the other side is 8 inches long. (A triangle that is inscribed in a triangle is a right triangle by definition.)
Penny Nom lui répond.
The wedges in a circle graph 2008-05-18
Libby pose la question :
I don't understand how to do this word problem: The cafeteria workers distributed a survey to the student body asking students to pick 1 from a list of 5 choices for their favorite lunch. The circle graph below gives the results of the survey for the students who responded. What is the measure, in degrees, of the central angle for each choice?

And there is a pie chart that has food choices with these percentages: 25%, 20%, 15%, 10%, and 30%

Leeanne Boehm and Penny Nom lui répond.
A right angled triangle 2008-05-16
bryan pose la question :
The sides of a right angled triangle are x, x+1, x+2 cm. Find the perimeter.
Penny Nom lui répond.
The sides of a rectangle 2008-05-16
bill pose la question :
The area of the rectangle is 170 sq.ft. the width is unknown and the length is the width plus 7 ft. I set it up like this w(w+7)=170 and this is where I get lost. Thanks, Bill
Penny Nom lui répond.
Fencing around a playground 2008-05-15
Vanessa pose la question :
If the playground is a rectangle with it's width 10ft shorter than it's length, and it can be enclosed by exactly 140ft of fence, what is the length and width of the playground?
Penny Nom lui répond.
An isosceles triangle 2008-05-07
Jay pose la question :
Given: An Isosceles Triangle with the area=16m squared. What would be the length of each leg.
Stephen La Rocque lui répond.
Fencing a triangular field 2008-04-29
Julieta pose la question :
The area of a field shaped like a triangle is 750yards squared. One leg of the field is fenced with wood costing $5/yd. The remainder of the perimeter of the field is fenced with steel costing $10/yd. The perimeter of the field is 150yards. the total cost of the fencing is $1200.
a. What is the length of the leg fenced with wood?
b. what is the length of the leg fenced with steel?

Penny Nom lui répond.
The Pythagorean theorem with triangles rather than squares 2008-04-29
Zachary pose la question :
I need to figure out how to prove the pythagorean theoorem using equilateral triangles instead of using square. I know that A^2+B^2=C^2, but how do you get that by using equilateral triangles. I know the area of a triangle is BH1/2=Area. So what i need to know is how to derieve the formula of a triangle to get the pythagorean theorem
Penny Nom lui répond.
A rectangle inscribed in a circle 2008-04-27
sridhar pose la question :
A rectangle with perimeter 28 cm inscribed in a circle of radius 5 cm find the area ?
Penny Nom lui répond.
A circle inscribed in a triangle 2008-04-26
Amar pose la question :
I have been given a circle inscribed in a triangle and have been told to prove that the ratio of the perimeter of the triangle to the circumference of the circle is the same as the ratio of the area of the triangle to the area of the circle. How would this be done?
Stephen La Rocque and Walter Whiteley lui répond.
Find the length and the width of the rectangle 2008-04-24
Vickie pose la question :
A rectangle is 2 times longer than it is wide. It has a diagonal length of 50 centimeters. Find the length and the width of the rectangle. Round your answers to the nearest tenth. Show your work.
Penny Nom lui répond.
The dimensions of a rectangle 2008-04-20
Brandy pose la question :
Find the dimensions of a rectangle with a primeter of 54 cm if the length is 5 less than three times the width.
Penny Nom lui répond.
The height of a triangle 2008-04-17
Trent pose la question :
I need to know the area of a triangle. One side is 5 the base is 12 and the angle between them is 52 degrees. No height is specified. How do I find the height to get the area?
Penny Nom lui répond.
A 30-60-90 triangle 2008-04-16
Ron pose la question :
30-60-90 triangle. Base is 2.75 how do I find the length of the other two sides?
Penny Nom lui répond.
The three sides of a triangle 2008-04-16
Bridgett pose la question :
The sum of the lengths of any 2 sides of a triangle must be greater than the third side. If the triangle has one side that is 11 CM, and the second side of the triangle is 4 cm less than twice the third side, what lengths do the 2nd and 3rd side have to be?

between 0000-00-00 and 9999-99-99


Stephen La Rocque lui répond.
Chord, radius, arc length and central angle 2008-04-15
Cindy pose la question :
There is a railroad curve with a chord length of 2000 ft. and a central angle of 35 degrees. What is the radius and arc length of the circular arc?
Stephen La Rocque lui répond.
A rectangle word problem 2008-04-10
Nick pose la question :
A rectangle is twice as long as it is wide. If the length and width are both increased by 5cm, the resulting rectangle has an area of 50cm^2. Find the dimensions of the original.
Penny Nom lui répond.
An isosceles right triangl 2008-04-09
Stephanie pose la question :
An isosceles right triangle has an area of 35 sq cm. What are the lengths of the sides?
Walter Whiteley lui répond.
Side lengths and angles in a regular octagon 2008-04-09
Lori pose la question :
I am a builder and need to find the length of the sides of an octagon. I know the length between the parallel sides (26 feet). What is the length of each side? What is the angle measurement?
Harley Weston lui répond.
The angle between the hands on a clock 2008-04-04
Kristin pose la question :
The question I need help with is : Bobo looked at the clock at 1:25. What was the precise measure of the angle formed by the hour and minute hands of the clock? Assume that each of the clock hands moves at a constant rate continuously around the face of the clock. Thus, in any given fractional part of an hour, each hand will move that same fractional part of an hour's worth of movement.
Stephen La Rocque lui répond.
Perimeter, area and diagonals of a rectangle 2008-04-03
jenea pose la question :
I have to find the perimeter as well as the area of a rectangle when given the diagonals are 12 units long and they intersect at a 60 degree angle. how do i find both the area and perimeter; do i use the 30 60 90 triangle rule?
Stephen La Rocque and Penny Nom lui répond.
The rectangle with smallest perimeter 2008-04-01
nicole pose la question :
when given an area,squares are the rectangles with the smallest perimeter write the length and width of the rectangle with the smallest perimeter
Penny Nom lui répond.
A fish tank in the shape of an irregular pentagon 2008-03-29
richie pose la question :
i am building a fish tank. it is going to be an irregular pentagon. the sides are going to be
24"
24"
8"
8"
32"(approximately)

there will 3 right angles A, B, E

my question is how to figure out the degree of the angles that are not right angles (C,D)?

Chris Fisher lui répond.
The area of an equilateral triangle 2008-03-28
Elizabeth-Morgan pose la question :
I have a problem that reads: Find the area of a triangle with sides of 4 cm. Find the height first.

This means that I have an equilateral triangle, and I need a formula to calculate the height using the base, and the sides.

Stephen La Rocque lui répond.
Similar triangles 2008-03-26
Nisha pose la question :
In triangle ABC, LM is parallel to AB. If AL=x-3,AC=2x,BM=x-2,BC=2x=3. Find x
Penny Nom lui répond.
The area of a rectangle 2008-03-25
Raphael pose la question :
What is the area of rectangle that measures 7 yards by 23 yards
Penny Nom lui répond.
A word problem about a triangle 2008-03-23
kathy pose la question :
The measure of the largest angle of a triangle is twice the measure of the second largest. The measure of the second largest is 20 degrees more than the measure of the smallest. Find the measure of each angle.
Melanie Tyrer lui répond.
The area of the triangle with vertices (-1,-8), (-1,1), and (5,1) 2008-03-23
jimmy pose la question :
find the area of a triangle whose vertices are (-1,-8), (-1,1), and (5,1)
Penny Nom lui répond.
Angles subtended by the same arc 2008-03-23
Reid pose la question :
Prove that two inscribed angles subtended by the same arc are equal.
Stephen La Rocque lui répond.
The area of a right triangle 2008-03-19
Tate pose la question :
I need to find the area of a right triangle. I already know that the formula would be a squared + b squared = c squared. However, I only have the length of the hypotenuse and the base.
Victoria West lui répond.
The area of an equilateral triangle 2008-03-16
michael pose la question :
my daughter has been given a task and I'm stuck too. if an area of an equilateral triangle = 85cm squared there are no other dimensions provided. how do I calculate the length of each side ? please
Penny Nom lui répond.
The area of a triangle 2008-03-14
Tara pose la question :
Area of triangle is 4,018 square yards. Its base is 82 yards. What is its height? Thanks
Stephen La Rocque lui répond.
Side lengths and area of a triangle 2008-03-08
angela pose la question :
In the diagram of ABC the angle A is 65 the angle B is 70 and the side opposite vertex B is 7. Find the length of the side opposite vertex A and find the area of ABC
Penny Nom lui répond.
Two circles and a triangle 2008-03-07
Adrian pose la question :
The vertices of a right-angled triangle are on a circle of radius R and the sides of the triangle are tangent to another circle of radius r. If the lengths of the sides about the right angle are 16 and 30, determine the value of R+r
Penny Nom lui répond.
A fence around a rectangular region 2008-03-06
sharon pose la question :
a farmer want to make a rectangular fence he has 750 feet of material . if the width is 95 feet what is the area of the rectangle
Penny Nom lui répond.
The dimensions of a rectangle 2008-03-06
Julie pose la question :
Find the dimensions of a rectangle with an area of 400. Width x ft. Length (2x-7)ft.
Stephen La Rocque and Penny Nom lui répond.
Is every rectangle a square? 2008-03-04
Odette pose la question :
if every square is a rectangle, is every rectangle aslo a square?
Penny Nom lui répond.
A 6 pointed star 2008-03-04
Siddharth pose la question :
When 2 congruent equilateral triangles share a common center, their union can be a star If their overlap is a regular hexagon with an area of 60, what is the area of one of the original equilateral triangles?

a) 60 b) 70 c) 80 d)90 e)100

Stephen La Rocque lui répond.
The angles of a triangle 2008-03-01
Allen pose la question :
How to find the other two angles of a scalene triangle if one angle is given
Steve La Rocque and Penny Nom lui répond.
Is a square a rectangle? 2008-02-29
Jaime pose la question :
Is a square a rectangle?
Penny Nom lui répond.
A triangle problem 2008-02-28
Jessica pose la question :
Triangle ABC Angle C is 3 times angle A angle B is 30 degrees less than angle C what are the measurements of the angles I have the measurements and I have the Equation but I can't figure out quite how to solve it the proper way
Stephen La Rocque lui répond.
A proof in geometry 2008-02-27
Kimberly pose la question :
I'm trying to write a proof for the following: If all altitudes are equal in an equilateral triangle then all sides are equal.
Stephen La Rocque and Penny Nom lui répond.
A right triangle 2008-02-25
Ali pose la question :
The length of a hypotenuse of a right triangle is 2 metre more than the longer leg. The length of the longer leg is 7 metre more than the length of the shorter leg. Find the number of metre in length of each side of the right triangle.
Stephen La Rocque lui répond.
The base of an isosceles triangle 2008-02-24
tahrima pose la question :
find the base of an isosceles triangle whose area is 12sq.cm and the length of 1 of the equal sides is 5 cm.
Stephen La Rocque lui répond.
The diagonal of a rectangle 2008-02-21
tim pose la question :
is there a formula for finding the diagonal of a rectangle?
Penny Nom lui répond.
Two right triangles 2008-02-18
Amy pose la question :
Hi, I have a problem that has me stumped.
There are two right triangles that share the same adjacent side (AB) which is 240'. The right angles of each triangle are at the opposite ends of the shared side.
The Opposite side (AC) of Triangle ABC is 180'
and the Hypotenuse (CB) is 300'.
The Opposite side (BD) of Triangle ABD is 100'
and the Hypotenuse (AD) is 260'.
How do I figure out where the hypotenuses intersect?

Stephen La Rocque and Harley Weston lui répond.
The area of a right triangle 2008-02-18
ASHLEY pose la question :
WHAT IS THE AREA OF A RIGHT TRIANGLE? WITH THE SHORT SIDE BEING 16 AND THE HYPOTENUSE 20?
Penny Nom lui répond.
The length of the third side of a triangle 2008-02-16
mary pose la question :
I have an angle of 72 degrees and each of the sides are 5' long. What is the distance from each of the ends of the 5 feet to form a triangle.
Stephen La Rocque lui répond.
The area of a rectangle 2008-02-12
mark pose la question :
The diagonal of a rectangle is 15cm, and the perimeter is 38cm. What is the area? It is possible to find the answer without finding the dimensions of the rectangle. Please help me and show as much of the problem as possible. Thank You
Penny Nom lui répond.
A triangle and its incenter 2008-02-11
Elle pose la question :
In the figure, P is the incenter of triangle ABC, the radius of the inscribed circle is 6cm, and the perimeter of triange ABC is 38cm. What is the area of triangle ABC?
Penny Nom lui répond.
A flagpole and a statue 2008-02-09
Krista pose la question :
A flagpole casts a ten meter (m) shadow at the same time as a six metre (m) statute beside it casts a two metre shadow. What is the height of the flagpole??
Steve La Rocque and Penny Nom lui répond.
The width of a rectangle is 4 less than twice its length. 2008-02-09
Shawn pose la question :
The width of a rectangle is 4 less than twice its length. If the area of the rectangle is 72 cm, what is the length of the diagonal
Stephen La Rocque lui répond.
A three sided shape 2008-02-09
Tim pose la question :
Question from Tim:

How would I calculate the area in square feet of a "triangle" with sides:

Base is 36 feet; height is 70 feet; and the thirs side is an arch (curve) of 77 feet.

Harley Weston lui répond.
Classifying a triangle 2008-02-07
kevin pose la question :
scalene triangle 8 ft base right side 9.5 left side 12 ft what is the angles
Penny Nom lui répond.
The radius of a circular arc 2008-02-04
Bill pose la question :
Hi,the Central Angle of a sector of a circle is 40 degrees. The circular arc of the curve has a chord length of 3000 ft. Find the radius(r) of the circular arc.
Penny Nom lui répond.
The dimensions of a rectangle 2008-02-01
Evan pose la question :
you are given that the area of a rectangle is 21.6in sq. find the base and the height of the rectangle.
Penny Nom lui répond.
The interior angles of a parallelogram 2008-01-29
amber pose la question :
the measure of one interior angle of a parallelogram is 42 degrees more than twice the measure of another angle. Find the measure of each angle.
Penny Nom lui répond.
The interior angles of a parallelogram 2008-01-28
steffie pose la question :
How do you calculate the interior angle sof a parallelogram?
Penny Nom lui répond.
Inscribed square in a triangle 2008-01-27
Adrian pose la question :
Consider a right-angled triangle PQR, where QR is the base and PQ is the height. QR=4cm and PQ=3cm. A square is inscribed in this triangle.Determine the length of one side of the square.
Stephen La Rocque lui répond.
Finding the area of an isosceles triangle given one angle and the inradius 2008-01-24
Saurabh pose la question :
Given an isosceles Triangle, whose one angle is 120 and inradius is √3. So area of triangle is?
Stephen La Rocque lui répond.
The smallest possible perimeter 2008-01-23
RS pose la question :
If two points of a triangle are fixed, then how can the third point be placed in order to get the smallest possible perimeter of the triangle.
Chris Fisher and Penny Nom lui répond.
Number of points with integer coordinates which lie inside the triangle 2008-01-22
Saurabh pose la question :
Number of points with integer coordinates which lie inside the triange whose verticies are (0,0) , (0,21) and (21,0) ???
Stephen La Rocque lui répond.
The three angles of a triangle 2008-01-21
val pose la question :
two angle in a triangle equal 120 degree. what is the measure of the third angle?
Penny Nom lui répond.
Two circles in a rectangle 2008-01-18
Alex pose la question :
One side of a rectangle is 10 and the other is x.

Two circles with equal radii are inscribed in the rectangle and like a Venn Diagram, the circles overlap.

Each circle touches the top and bottom of the rectangle and the left circle touches the left side of the rectangle and the right circle touches the right side of the triangle.

The distance between the centre of each circle is 2x/3.

Find x

Penny Nom lui répond.
The angles of a triangle given the three sides 2008-01-17
Lucy pose la question :
Is there a way to find the angles of a triangle just by knowing the lengths of it's sides? It seems like the would be a relationship between the two, but I'm not sure.
Stephen La Rocque and Harley Weston lui répond.
How tall is the pole? 2008-01-13
Michael pose la question :
If a 25 foot cable is attached to the top of a pole, and that cable attaches to the ground ten feet away from that pole, how tall is the pole? The 25 foot cable is the hypotenuse of a right triangle with a base of ten feet.
Leeanne Boehm and Penny Nom lui répond.
Forces on an inclined plane 2008-01-10
Ron pose la question :
A body that weighs 540lbs is caused to slide up an inclined plane with a uniform velocity by a force that acts parallel to the plane. For each foot of horizontal distance, there is a vertical rise of 2in. If the coefficient of sliding friction is 0.16, what force is required to move the body?
Stephen La Rocque lui répond.
The height of a triangle 2008-01-04
Dara pose la question :
The area of a triangle is 54m2. If its base is 5m, find its height.
Penny Nom lui répond.
Lining up coins visually using geometry and trigonometry 2007-12-31
Jessica pose la question :
a) In what order would you arrange a penny, a nickel, a dime, a quarter, and a half-dollar so that they all have the same apparent size? The diameters of the coins, in thousandths of inches, are as follows: penny, 750; nickel, 835; dime, 705; quarter, 955; half-dollar, 1205.
b) How should the coins be placed, if the distance between the dime and the half-dollar is 100 units? How far from thw dime should your eye be to see that all the coins have the same apparent size?
c) What angle do the coins subtend when they have the same apparent size?

Stephen La Rocque lui répond.
A rectangle is three times as long as it is wide. 2007-12-31
Robert pose la question :
A rectangle is three times as long as it is wide. If the length is decreased by 2 cm and the width is increased by 3 cm, the new rectangle has an area of 42 cm. What are the dimensions of the original rectangle?
Penny Nom lui répond.
Calculate the two possible lengths of the third side. 2007-12-28
Lisa pose la question :
An isosceles triangle has an area of 25.6 sq cm. The two equal sides are 8.4 cm long. Calculate the two possible lengths of the third side.
Penny Nom lui répond.
The dimensions of a rectangle 2007-12-22
Justin pose la question :
I'm trying to find the width of a rectangle. I have its area and its height. The area is 32 inches square and the width is 4 feet (48 inches). Can someone help explain this to me?
Penny Nom lui répond.
How many triangles can be formed? 2007-12-21
pankaj pose la question :
Q.There is 2 lines parallel to each other. If 1st has 5 points in it and 2nd 3 points in it,how many triangles can be formed?
Penny Nom lui répond.
Find the lengths, and explain in a 'let statement' 2007-12-20
Cordilla pose la question :
the length of the sides of a triangle are 3 consecutive whole numbers. The perimeter of the triangle is 102 meters. Find the lengths, and explain in a' let statement'.
Stephen La Rocque and Harley Weston lui répond.
Smallest cone containing a 4cm radius inscribed sphere 2007-12-19
Eva pose la question :
A sphere with a radius of 4cm is inscribed into a cone. Find the minimum volume of the cone.
Stephen La Rocque lui répond.
A triangle and a rectangle 2007-12-14
Someone pose la question :
A certain triangle has sides that are, respectively, 6inches, 8inches, and 10 inches. A rectangle with a n area equal to that of the triangle has a width of 3 inches. What is the perimeter of the rectangle, in inches.
Penny Nom lui répond.
Finding all the angles 2007-12-13
Rajesh pose la question :
An eqilateral triangle is drawn in a square with one of the side as its base and draw the lines from the other angular sides such that there are four triangles formed inside the square which includes the equilateral triangle.I want to know all the angles of all the triangles formed inside the square.
Stephen La Rocque and Penny Nom lui répond.
A 45-45-90 triangle 2007-12-13
Aaditya pose la question :
explain to me please how to do the 45-45-90 theorem when one of the legs(not the hypotenuse) is 3. How do you find the remaining two sides? please help me out.
Leeanne Boehm lui répond.
The dimensions of a rectangle 2007-12-12
tara pose la question :
The length if a rectangle is 5 feet less than twice its width. The perimeter of the rectangle is 38 feet. let w represent the width. write an equation for the perimeter of the rectangle in terms of w. Then solve the equation to find the length and width o the rectangle.
Penny Nom lui répond.
A right triangle 2007-12-06
Shubhomoy pose la question :
The co-ordinates of a hypotenuse are (1,3) and (-4,1). Find the equations of the perpendicular sides.
Harley Weston lui répond.
A circle inscribed in a triangle 2007-12-06
Linnea pose la question :
I have a tringle with a circle inscribed in it. My teacher wants me to find the radius of the circle. This is what she gave me to work with. The triangle is ABC, AB = AC = 6, and BC = 4. She also told us to use A(squared) + B(squared) = C(squared). and that there are altitudes and and incenter. I have no idea how to do this.
Harley Weston lui répond.
Third side of a triangle 2007-12-05
Sudhir pose la question :
How can we find the all positive integral values for the third side of a triangle, whose other two sides measure 15 and 20 respectively. (No other information viz. the type of triangle, area, perimeter or angles etc. is given.)
Stephen La Rocque lui répond.
Area = 1/2 ab SinC 2007-12-03
Eileen pose la question :
Given: Acute triangle ABC, with a, b, c, being the respective opposite sides to angle A, angle B, angle C, and altitude, h, drawn from angle B to b. Prove: The area of trianlge ABC=1/2abSin C
Stephen La Rocque lui répond.
What angle should he turn? 2007-12-01
Jasmine pose la question :
Bob is traveling due north he then turns left 45 degrees followed by four 20 degree turns to the right

What is his new heading if he needed to turn so his heading was SSW what direction and what angle should he turn?

Walter Whiteley lui répond.
The area of a triangle 2007-11-30
Kevin pose la question :
if the base of the triangle is 13ft and the altitude is 6ft then what is the area, and how do we calculate this?
Penny Nom lui répond.
Find the centroid 2007-11-26
josh pose la question :
vertices X(-2,6) Y(4,10) Z(14,6) find the coordinates of the centroid
Stephen La Rocque lui répond.
An isosceles triangle 2007-11-26
Nancy pose la question :
I needed to help my 9th grade daughter regarding a geometry problem. After a while I realized I am not getting anywhere. I saw that in 2005 someone asked the same question and you gave them a hint. Unfortunately it still did not help. Because I had figured that much!Can you kindly help me proof this problem?
PX and QY are attitudes of acute triangle PQR, and Z is the midpoint of PQ. Can you write a proof that triangle XYZ is isosceles?
I am sure it is something simple I am missing, but I just can not seem to be able to see it. Thank you much.

Harley Weston lui répond.
The maximum area of a rectangle 2007-11-23
Christy pose la question :
Question from Christy, a student:

Show that among all rectangles with an 8m perimeter, the one with the largest area is a square.

I know this is simple but I'm not sure if I'm doing it correctly. Here is what I did.

1. A = xy
2. 8 = 2x+2y
3. y = 4-x
4. A = x(4-x) = 4x-x^2


Not sure what to do from this point because I don't know if its right.

Harley Weston lui répond.
The areas of a rectangle and a triangle 2007-11-13
karen pose la question :
hoe to calculate the shaded area of a triangle in a rectangle
Penny Nom lui répond.
The third side of a triange 2007-11-12
Christine pose la question :
I need the inequality to describe the third side of a triange if two sides are known. First side is 23 and second side is 15. What is the third side?
Stephen La Rocque and Victoria West lui répond.
Related Rates (streetlamp and shadow) 2007-11-09
Casey pose la question :
A street light is mounted at the top of a 15ft pole. A man 6ft tall walks away from the pole at a rate of 5ft per second. How fast is the tip of his shadow moving when he is 40ft from the pole?
Stephen La Rocque and Penny Nom lui répond.
Two triangles 2007-11-08
Barbara pose la question :
If the base of a larger triangle is 34 inches long, what is the length of side A of the smaller triangle?

(the small triangle has on top the letter a on the right side of the triangle it has the letter b, and at the bottom of the small triangle it has the number 17)

Penny Nom lui répond.
The width of a rectangle 2007-11-07
BJ pose la question :
The perimeter of a rectangular field is yards. If the length of the field is yards, what is its width?
Penny Nom lui répond.
The height and area of a triangle 2007-11-05
Daniella pose la question :
I need the exact formulas to find the height, the area,the lenght of triangles...
Penny Nom lui répond.
The height of a triangle 2007-11-03
William pose la question :
I need to find the height of a triangle if the base is 15, one side is 14 and the other side is 13.
Stephen La Rocque and Penny Nom lui répond.
A right angle triangle 2007-11-02
PRAIMNAUTH pose la question :
In a right angle triangle , knowing one angle (90 degree ) and the length of the base, how can you calculate the hight and the hypotenuse ?
Penny Nom lui répond.
The base and height of a triangle triangle 2007-10-31
mike pose la question :
How do you find the length of the base or the height of a triangle if the area is 30 square feet?
Penny Nom lui répond.
The height of an isoceles triangle 2007-10-29
Maura pose la question :
I am trying to calculate the height of an isoceles triangle but in this case AB is not equal to AC. How do I calculate this? Thank you, Maura age 13
Stephen La Rocque lui répond.
The length of the side of a triangle 2007-10-28
devon pose la question :
hay ok i have the height of a triangle but how do i find the length od the side if all of the sides are the same length
Penny Nom lui répond.
An equalateral triangle is 30 acres 2007-10-28
mike pose la question :
If the area of an equilateral triangle is 30 acres, what is the length of each side in feet or miles?
Penny Nom and Victoria West lui répond.
Area of a triangle formed by three points on a graph 2007-10-26
Betty pose la question :
My question is 'find the are of a triangle whose vertices have coordinates (3,5),(6,-5), and (-4,10)
Stephen La Rocque lui répond.
The length of a rectangle is twice the width 2007-10-26
Tami pose la question :
The length of a rectangle is twice the width. The number of square units in it's arear is four times the number of units in it's perimeter. What are the dimensions of the rectangle?
Penny Nom lui répond.
Magic triangles 2007-10-25
joshine pose la question :
i confuse about a magic triangle.but my teacher ask me to make examples of it in 4 different magic triangle and also different pattern can you help me about it?
Penny Nom lui répond.
Two mirrors 2007-10-24
Peter pose la question :
The reflecting surfaces of two intersecting flat mirrors are at an angle θ (0° < θ < 90°). For a light ray that strikes the horizontal mirror, show that the emerging ray will intersect the incident ray at an angle β = 180° – 2θ.
Stephen La Rocque lui répond.
Given a six by six square, how rectangles are there in the square? 2007-10-24
Maria pose la question :
Given a six by six square, how rectangles are there in the square?
Stephen La Rocque lui répond.
The length of a rectangle 2007-10-24
Kharez pose la question :
What is the length of a rectangle with an area of 24 square feet and a width of 4 feet?
Penny Nom lui répond.
An equilateral triangle with height 2 inches 2007-10-23
Christine pose la question :
I'm doing an project in school about tessellations. But first, I have to construct an equilateral triangle with an altitude of exactly 2 inches. I know how to draw an equilateral triangle...but I don't know what I have to do in order for the triangle to have an altitude of 2 inches.
Penny Nom lui répond.
Calculate the dimensions of the rectangle 2007-10-16
Sara pose la question :
The perimeter of a rectangle is 62cm. The length of the diagonal is 25 cm. Calculate the dimensions of the rectangle.
Stephen La Rocque lui répond.
Surface area of an open-ended cone 2007-10-16
Lorne pose la question :
What is the surface area of an open ended cone? Measured at 10' high, 16' diameter on the bottom and 2' diameter at the top.
Stephen La Rocque lui répond.
Four triangles in a square 2007-10-15
Kristina pose la question :
A square with side lengths of 6 cm is divided into 3 right triangles and a larger isosceles triangle. If the three right triangles have equal area, find the exact area of the isosceles triangle.
Stephen La Rocque lui répond.
The dimensions of a rectangle 2007-10-14
Elizabeth pose la question :
The perimeter of a rectangle is 20m. The area is 20sqm. Calculate the dimensions of the rectangle. Give your answers correct to two decimal places.
Penny Nom lui répond.
The angles in a polygon 2007-10-11
Farzan pose la question :
Prove with induction that in a polygon( that may not be convex ) with n sides, the sum of the amounts of the angles become 180(n-2). If there is any easier methods to prove the problem, please write as well.
Stephen La Rocque lui répond.
Triangle perimeter and area 2007-10-05
divakar pose la question :
The sides of the triangle are integers. The perimeter is 8. What is the area?
Stephen La Rocque lui répond.
Arc lengths, central angles and radii 2007-10-04
Ashutosh pose la question :
Jose can remember that the length of an arc is 440cm, but he cannot remember the radius of the arc or the angle at the center. He does know that the angle was a whole number of degrees and the radius was less than 100cm. Find three possible angles and write down the size of each of the possible radii.
Stephen La Rocque lui répond.
Find the area of the traingle 2007-10-02
Indrajit pose la question :
The perimeter of a triangle is equal to the perimeter of a semicircle of radius 35 cm.The ratio of the triangles is 5:6:7. Find the area of the triangle? [pie = 22/7]
Stephen La Rocque lui répond.
Equation of a circle circumscribing a triangle with given vertices 2007-10-01
Randy pose la question :
How do I determine the equation of a circle when it is circumscribed by a triangle whose vertices are (-1, 6), (3, -2), and (2, 5)?
Stephen La Rocque lui répond.
Finding equations, intersection point of two lines at right angles 2007-09-22
Yaz pose la question :
Find the equation of the line joining A(-1,-9) to B(6,120). Another line passes through C(7,-5) and meets AB at rigth angle of D. Find the euation of CD and calculate the co-ordinates of D.
Stephen La Rocque lui répond.
An equilateral triangle 2007-09-13
Bibi pose la question :
An equilateral triangle has a point P inside it. PA, PB, PC are the perpendiculars from point P to the three sides of the triangle. Help me show that PA+PB+PC is the same no matter where the point P is..?
Harley Weston lui répond.
Composite angles 2007-09-13
Gilbert pose la question :
what is a composite angle? please help
Stephen La Rocque lui répond.
Find the area between (-4,0) (2,0) (2,6) 2007-09-11
sabrina pose la question :
Find the area between (-4,0) (2,0) (2,6)
Penny Nom lui répond.
A rectangle inscribed in a circle 2007-09-11
Sobeida pose la question :
A rectangle that is x feet wide is inscribed in a circle of radius 8 feet. What is the area of the rectangle as a function of x. Thanks!
Stephen la Rocque lui répond.
A right angles triangle 2007-09-05
Daryl pose la question :
I have a right-angled triangle. two of its sides are 12.6cm. The third/last line is missing. How can I find the length of the last line?
Stephen La Rocque and Penny Nom lui répond.
Similar triangles 2007-08-29
James pose la question :
Question: In the triangle ABC, X is a point on AC. AX = 15 m and XC = 5 m. The angle AXB is 60 degrees and the angle ABC is 120 degrees. Find the length of BC.

I am sure to an extent that this has to do with similar triangles, but I am not certain.

Harley Weston lui répond.
Reference angles 2007-08-25
Jenny pose la question :
find the reference angles for the angles given below, find the quadrants in which the angles lie
1. 0=6n/7
2. 0=3.3

Stephen La Rocque lui répond.
A geometry problem 2007-08-20
samhita pose la question :
ABC is a triangle. Let D be a point on side BC produced beyond B such that BD=BA. Let M be the mid-point of AC. The bisector of angle ABC meets DM at P. Prove that angle BAP=angle ACB.
Chris Fisher lui répond.
Area and perimeter 2007-08-16
sheryl pose la question :
a 50*70 horse pen is 240 feet and 3500 square feet a 60*60 horse pen is 240 feet and 3600 square feet why is there a 100 square feet difference?
Penny Nom lui répond.
The area and perimeter of a rectangle 2007-08-15
Brooklyn pose la question :
Is there a way to find the perimeter of a rectangle if you have the area, or vice versa? If so, what is the equation?
Leeanne Boehm and Penny Nom lui répond.
The length of the third side of a triangle 2007-08-15
Brooklyn pose la question :
What is the equation to find the length of the third side of a triangle if you have the length of A, B, and the angles(s)?
Stephen La Rocque lui répond.
A right triangle 2007-08-11
Peter pose la question :
Ok, is it possible to find the height and base of a right triangle when all the information you are given is the length of the hypotenuse?
I also know that the angle between side b and the hypotenuse is 45 degrees. please help.

Walter Whiteley lui répond.
Find the dimensions of the rectangle that will contain the greatest area 2007-08-06
Julirose pose la question :
The perimeter of a rectangle is 38 meters. Find the dimensions of the rectangle that will contain the greatest area.
Penny Nom lui répond.
The height of a pole 2007-08-02
lalaine pose la question :
Hi, this is my problem.. From a point 50.2 m to the pole, a student measured the angle of elevation to the top of the pole to be 32°. Find the height of the pole if the student's height from his feet to his eyes is about 4 ft.
Penny Nom lui répond.
Three points on the circumference of a circle 2007-07-30
Bharathi pose la question :
given a circle with radius r and a point x,y on its circumference,output two other points x1,y1 and x2,y2 on the circle so thar all 3 points form a equilateral triangle.
Stephen La Rocque lui répond.
Area of a star in a regular pentagon with side length 10cm 2007-07-24
Chetna pose la question :
A regular pentagon with side 10 cm has a star drawn within (the vertices match). What is the area of the star?
Stephen La Rocque lui répond.
Angle of depression 2007-07-23
joyce pose la question :
hello, here is my problem......
As you stand on a bridge w/c is 100 ft. above the water you are looking @ an approaching barge. If the A of top of the front of the bridge is 29.04 degrees and the angle of depression of the rear is 17.36 degrees . Find the length of the barge?

Harley Weston lui répond.
An oblique triangle 2007-07-19
fhay pose la question :
In an oblique triangle, If side a=95, side b=102 and side c = 150 find the missing angles solved by right angles...Thank's a lot......
Penny Nom lui répond.
Angle of depression (declination) between sailboat observations 2007-07-18
Joyce pose la question :
From a cliff 150 ft above a lake, we see a boat sailing directly towards us. The angle of depression of the boat is seen to be 5 degrees and 7 inches and 11 degrees and 18 inches. Find the distance sailed between observations.
Stephen La Rocque lui répond.
Angle of inclination from the horizontal 2007-07-18
Joyce pose la question :
In flying upward for 1260 yards along a straight inclined path airplane rises 156 yards. Find the climbing angle ( the angle of inclination from the horizontal) Thank you in advance
Stephen La Rocque lui répond.
The isosceles triangle of largest area with perimeter 12cm 2007-07-16
sharul pose la question :
find the dimension of isosceles triangle of largest area with perimeter 12cm
Harley Weston lui répond.
Height of a tower from two observations 2007-07-16
joyce pose la question :
An observer wishes to determine the height of a tower. He takes sight @ the top of the tower from A & B w/c are 5oft. apart @ the elevation on a direct line w/ the tower. The vertical angle @ point A is 30 degrees & the point B is 40 degrees. What is the height of the tower? Find the value of x in angle tangent 40 degrees and 30 degrees? Show the solution of the value of x?
Stephen La Rocque lui répond.
Height of an antenna (angle of elevation) 2007-07-16
Fhay pose la question :
An antenna stands on the edge of the top of a 52 story building from a point 320 ft. from the base of the building, the angle of elevation to the top of the antenna is 64 degrees in each story is 12 ft. high. Find the height of the antenna
Stephen La Rocque lui répond.
The two towers (angles of elevation trigonometry) 2007-07-14
joyce pose la question :
The angle of elevation of tower B from the top of tower A is 28 degrees and the angle of elevation of the top of tower A from the base is 46 degrees Find the height of tower A if tower B is 120 m high?
Stephen La Rocque lui répond.
Proving a quadrilateral is a rectangle 2007-07-14
Sonja pose la question :
I was having this discussion with another teacher and we need a third opinion. When you are trying to prove a quadrilateral is a rectangle which method should you use:
  1. Prove the shape is a parallelogram by doing slope 4 times by stating that parallel lines have equal slopes. Then proving a right angle by stating that perpendicular lines have negative reciprocal slopes.
  2. Doing the slope 4 times and stating that the shape is a rectangle because opposite sides are parallel because of equal slopes and it contains a right angle because of negative reciprocal slopes.
I guess the real question is do you have to first state that the shape is a parallelogram?

Stephen La Rocque lui répond.
Proof that any side of a triangle is less than half the perimeter. 2007-07-07
Omkar pose la question :
Any side of a triangle is smaller than half of its perimeter, prove this in short ?
Stephen La Rocque lui répond.
Finding the radius of an inscribed circle 2007-07-05
Maria pose la question :
I need to find the radius of a circle which is inscribed inside an obtuse triangle ABC. I know all the angles and all the lengths of the triangle.
Stephen La Rocque and Chris Fisher lui répond.
Using calculus to prove the formula for the area of a triangle 2007-07-04
Apratim pose la question :
Using calculus how can one show that the area of any triangle is 1/2 times its base times its height?
Stephen La Rocque lui répond.
A square and an equilateral triangle have the same perimeter. 2007-07-02
eve pose la question :
I am trying to help my daughter with the following problem: A square and an equilateral triangle have the same perimeter. The sum of one side of the square and one side of the triangle equals 28 cm. Find the sides of the figures.
Penny Nom lui répond.
Using Heron's Formula to help maximize the area of a triangle 2007-06-27
Claire pose la question :
Given one side of a triangle is 4 cm and the ratio 1:3 for the other 2 sides. What is the largest area of the triangle?
Stephen La Rocque and Harley Weston lui répond.
Volume in a triangular water trough 2007-06-24
David pose la question :
A water trough has sloping sides of length 500mm making it triangular in cross section, with vertical ends. The width at the top is 600mm and the length is 2.0 metres.
(i) Calculate the capacity of the trough, giving your answer accurate to the nearest litre.
(ii) Find out the depth of the water when the trough is half full.

Stephen La Rocque lui répond.
The sum of angles in a square is three hundred and sixty degrees 2007-06-21
victor pose la question :
can you please send to me a formalize proof for the assertion that, the sum of angles in a square is three hundred and sixty degrees. i will be grateful to hear from you
Stephen La Rocque lui répond.
The perimeter of a triangle 2007-06-20
chelsie pose la question :
how do i find the perimeter of a triangle given the area, base, and height?
Penny Nom and Walter Whiteley lui répond.
A square and a rectangle 2007-06-19
Clarence pose la question :
What is the difference between rectangle and a square
Penny Nom lui répond.
Using the Pythagorean Theorem 2007-06-18
cynthia pose la question :
Hi, If I have a question with a right triangle and it asks.... If ABC is say 400 miles. How much shorter will the miles be if I travel from BC? I don't exactly remember the question but, I would I solve a problem similiar to this one?
Stephen La Rocque lui répond.
Angles of depression 2007-06-13
Phonda pose la question :
The pilot of a small private plane can look forward and see the control tower for a small airstrip. Beyond that is a large factory that is 3 milies from the airstrip. The angles of depression are 12.5 degrees and 4.8 degrees respectively. Find the airplane's altitude, to the nearest ten feet.
Stephen La Rocque lui répond.
The area of equilateral triangle 2007-06-05
solomon pose la question :
describe the area of equilateral triangle in terms of its side? solve in two ways.
Stephen La Rocque lui répond.
A regular octagon is made up of 8 identical isosceles triangles 2007-06-04
Alex pose la question :
A regular octagon is made up of 8 identical isosceles triangles. If the perimeterof each isosceles triangle and the hexagon are 58 cm and 168 com respectively, find the length of the identical sides of the triangle.
Stephen La Rocque lui répond.
How many faces does a triangle have? 2007-06-04
Daniela pose la question :
how many faces does a triangle have?
Stephen La Rocque lui répond.
Area of an isosceles triangle 2007-06-01
Josh pose la question :
In a previous question answered by Sue regarding the area of a regular polygon you gave a formula for the area of an isosceles. My question is how did you get this formula? Can you please explain to mean the process that you used to get that formula? Thanks
Stephen La Rocque lui répond.
Altitudes of scalene triangles 2007-05-17
Chris pose la question :
Two of the altitudes of a scalene triangle ABC have length 4 and 12. If the length of the third altitude is also an integer, what is the biggest that it can be?
J. Chris Fisher lui répond.
Angle of Elevation 2007-05-10
Micky pose la question :
Two Buildings are on opposite sides of a street 40 feet wide. The taller of the two buildings is 580 feet tall. The angle of depression from the top of the tallest building to the shorter building across the street is 57 degrees. Find the height of the shorter building.
Stephen La Rocque lui répond.
Area of an equilateral triangle 2007-05-09
Diana pose la question :
how do i find the area of an equilateral triangle when they give us the base?
Penny Nom lui répond.
The dimensions of a rectangle 2007-05-07
Stephanie pose la question :
The perimeter of a rectangle is 54yds , and the area of the rectangle is 180yds^2 . Find the dimensions of the rectangle.
Penny Nom lui répond.
The vertices of a triangle 2007-05-02
natakie pose la question :
Find the coordinate of the vertices of a triangle whose sides lie on the following three lines. 2x+5y-16=0, 4x-3y-6=0 and 3x+y+2=0
Penny Nom lui répond.
The height of a triangle 2007-05-02
Cassie pose la question :
If you have a scalene triangle with sides measuring 8 units, 6 units, and 10 units (and 10 is the base) how do i find the height perpendicular to the base of 10?
Penny Nom lui répond.
A triangle problem 2007-04-23
Dimitri pose la question :
In the triangle ABC, M is the midpoint of AB and N is the midpoint of AC. Prove that CM and BN cannot bisect. (I can prove it by contradiction if it is a scalene triangle, but i cant seem to prove it for isosceles)
Chris Fisher lui répond.
How can I find out how many sides there are? 2007-04-23
gn pose la question :
I know the angle measurements of a convex polygon. They are congruent angles. How can I find out how many sides there are?
Steve La Rocque, Penny Nom and Walter Whiteley lui répond.
I am looking for a triangle joke 2007-04-18
deborah pose la question :
Hi there, I just found your website. Wow!
I am presenting a triangle presentation on Friday at school and I am looking for a triangle joke to open up the presentation.
Do you have one to share with me? I'm discussing types of triangles if that helps.

Stephen La Rocque lui répond.
A gardener has 140 ft of fencing to fence 2007-04-16
Johann pose la question :
A gardener has 140 ft of fencing to fence in a rectangular shaped vegetable garden. To plant all his vegetables, the gardener needs more than 825 sq ft of space. What are the possibles values for the length of the rectangle that would meet the space requirements? You may assume that the variable x represents the length of the rectangle.
Stephen La Rocque lui répond.
The angle of depression 2007-04-14
Mary-Beth pose la question :
Two towers are 30 m apart. From the 15th floor, 40 m up, find the angle of depression to the base of the taller tower? I think the answer is 53, but the answer book says 37.
Penny Nom lui répond.
Determine the probability that the triangle is acute. 2007-04-10
Greg pose la question :
Points A1, A2, . . ., AN are equally spaced around the circumference of a circle and N >=3. Three of these points are selected at random and a triangle is formed using these points as its vertices. If N = 2k for some positive integer k >= 2, determine the probability that the triangle is acute.
Penny Nom lui répond.
I have an isosceles triangle. 2007-04-10
Stephanie pose la question :
I have an isosceles triangle. The two equal sides are given to me, but not the base. The equal sides are 12. I have to find the base of the triangle. Help!!
Stephen La Rocque lui répond.
A right triangle 2007-04-05
Lee pose la question :
What would be the length of the long side of a right angle triangle if one side was 47cm and the other was 56cm. Many Thanks
Steve La Rocque and Jaymi Peterson lui répond.
What is the hypotenuse of a right traingle 2007-04-04
debbie pose la question :
what is the hypotenuse of a right triangle with sides of 38 meters and 24.2 meters.
Stephen La Rocque lui répond.
Interior and exterior angles 2007-04-02
Anuj pose la question :
If in a regular polygon, each exterior angle is twice the interior angle,find the number of sides?
Leeanne Boehm lui répond.
Find the measure of all 3 angles. 2007-03-28
Brittney pose la question :
The smallest angle in a triangle is one-third of the largest angle. The third angle is 10 more than the smallest one. Find the measure of all 3 angles.
Stephen La Rocque lui répond.
The angles in a right triangle 2007-03-28
Golaan pose la question :
I need the to understand the formula for finding either of the acute angles of a right triangle given it's hieght length and base length. I want to find the degrees of either accute angle. So for this example I have a right triangle with a height of 410 meters and a base length of 1,700 meters. I don't understand cosine, sine, and tangent or the other ones at all. So if the solution includes those (which I believe it does) could b very verbose yet in a very elementary way?

The purpose of ths is that I have to define the general incline angles (or grade) of various areas of terrain. I know the distance by map and also the the altitude at either endpoint.

Penny Nom lui répond.
The height of a pole 2007-03-22
clyde pose la question :
I am trying to determine the height of a pole by using 3 angles top 30.8 deg, center 17.3 deg, and bottom -5.8 deg. Can you point me in the right direction ?
Stephen La Rocque lui répond.
The Reuleaux Triangle 2007-03-22
Dani pose la question :
Hi! My question is: Determine how the area of the Reuleaux Triangle of width h compares to the area of a circle of width h? Thanks! Dani
Penny Nom lui répond.
Angles of depression 2007-03-21
romaine pose la question :
a woman of height 1.4m standing on top of a building of height 34.6m veiws a tree some distance away. she observes that the angle of depression of the bottom of the tree is 35 degrees, and the angle of depression of the top of the tree is 29 degrees. assume that the building and the tree is on level ground.
(a) calculate the distance of the woman from the top of the tree measured along her line of sight.
(b) determine the height of the tree.

Stephen La Rocque lui répond.
A triangle 2007-03-18
Farzana pose la question :
pointsA(1,0),B(8,0),C(3,4) are the vertices of a triangle.What is the of this triangle?
Stephen La Rocque lui répond.
A river crossing 2007-03-16
tara pose la question :
A river has a constant current of 4 kilometers per hour. At what angle to a boat dock should a motorboat, capable of maintaining a constant speed of 20 kilometers per hour, be headed on order to reach a point directly opposite the dock? If the river is 1/2 kilometer wide, how long with it take to cross?
Stephen La Rocque lui répond.
An isosceles Triangle 2007-03-15
Devon pose la question :
The length of one of the equal legs of an isosceles triangle is 8 cm less than 4 times the length of the base. If the perimeter is 29 cm, find the length of one of the equal legs.
Stephen La Rocque lui répond.
Triple angle tangent formula 2007-03-15
sam pose la question :
Hi I am trying to derive a triple angle formulae for tan. I know i need to use compound and double angle formulae but am finding it difficult to "clean" up my fraction to get the triple angle formulae can you show me a worked derivation?! thanks
Penny Nom lui répond.
The dimensions of a rectangle with least perimeter for given area 2007-03-14
Karlianna pose la question :
find dimensions of a rectangle with least perimeter for given area
A) 30cm2
b) 50cm2

Penny Nom lui répond.
Angle of elevation 2007-03-13
Joslyn pose la question :
A ship at sea sights a 12m high lighthouse on a cliff which is 80m above sea level. If the angle of elevation to the top of the lighthouse is 27 degrees, calculate the distance from the ship to the shore.
Haley Ess lui répond.
Can the triangle be called both an acute triangle and an equilateral triangle? 2007-03-10
Jane pose la question :
If all sides of a triangle are the same length and all angles are 60 degrees, can the triangle be called both an acute triangle and an equilateral triangle?
Walter Whiteley lui répond.
Finding the vertical height of a roof 2007-03-05
Zainab pose la question :
The question is: If the vertical height if a triangle is half the width of the base and the slant length is 6 metres, find the exact vertical height of this part of the roof. I'm actually confused about finding out the height of an equilateral triangle if you're only given the length or slant height. Please help! O.o
Stephen La Rocque lui répond.
The area of an inscribed triangle 2007-03-02
Caitlin pose la question :
What is the area of an equalateral triangle inscribed in a circle whose circumference is 6 pie?? PLEASE HELP
Penny Nom lui répond.
sin(3a), cos(3a) and tan(3a) 2007-02-28
mailene pose la question :
hi, i...indeed,to..need..your..help
how..cn..i..prove..this,formula???
sin3a=3sina-4sin^3a
cos3a=4cos^3-3cosa
tan3a=3tan-tan^3a /1-3tan^2a

the..symbol...^is..the..expOnent

Haley Ess and Penny Nom lui répond.
The three sides of a right triangle 2007-02-23
Sharon pose la question :
I know the hypotenuse of a right triangle is a^2 + b^2 = c^2 but when I replace this with the numbers given, I can't solve it.

Here is my question: The hypotenuse of a right triangle is 2.8 units long. The longer leg is 1.9 units longer than the shorter leg. Find the lengths of the sides of the triangle.

Pam Fowler and Penny Nom lui répond.
The three sides of a triangle 2007-02-23
Christopher pose la question :
If two sides of a triangel have lengths 4 and 9, then the length of the third side may be any number:
a) Less than 13
b) Greater than 5 but less than 13
c) Greatehr than 4 but less than 9
d) Greater than 5

Haley Ess lui répond.
Similar triangles 2007-02-21
Vivienne pose la question :
The lengths of the sides of a triangle are 8, 15, & 17. If the longest side of a simlar triangle is 51, what is the length of the shortest side?
Stephen La Rocque and Penny Nom lui répond.
Find the area of the triangle 2007-02-20
Christina pose la question :
Graph the function f(X)= x+1/x-1 and graph the tangent line to the function at the points A:(2,3) and B:(-1,0). The point of intersection of the two tangent lines is C. Find the area of the triangle ABC.
Stephen La Rocque lui répond.
Square centimeters 2007-02-12
Ivan pose la question :
how many square centimeters are in a rectangle that is 210 centimeters long by 39 centimeters wide.? thank you
Karen McIver and Melanie Tyrer lui répond.
Area of a rectangle 2007-02-07
kyle pose la question :
paul has enough material to make a fence 22 feet long. he will use this material to fence of a rectangular section of his yard. the rectangles sides must all be whole numbers. i am having a hard time finding the greatest and the least area that he can enclose his fece
Haley Ess lui répond.
The interior angles of a parallelogram 2007-02-07
jenniffeir pose la question :
Can a parallelogram have two 45 degree angles and two 75 degree angles?
Haley Ess lui répond.
A triangle inscribed in a semicircle 2007-02-06
Benneth pose la question :
Consider a triangle inscribed in a semicircle with a radius of R. What are th possible perimeters for the triagle? And the areas?
Penny Nom lui répond.
Angle of elevation 2007-02-05
Zee pose la question :
A 55 ft. flagpole casts a 25 ft. shadow. Calculate the angle of elevation to the sun to the nearest degree.
Stephen La Rocque lui répond.
A rectangle inscribed in a circle 2007-02-04
Benneth pose la question :
Consider a rectangle with radius R inscribed in a circle. What are the possible areas of the rectangle?
Steve La Rocque and Walter Whiteley lui répond.
Rectangles and squares 2007-01-31
Shelby pose la question :
I am doing homework and I need to know why squares are rectangles but not all rectangles are squares?
Penny Nom lui répond.
Angle of clock hands 2007-01-31
Sophia pose la question :
How many degrees are in the aute angle formed by the hands of a clock at 2:20 p.m.?
Stephen La Rocque lui répond.
The perimeter and area of a right angled triangle 2007-01-31
Naveen pose la question :
I need to calculate the area of a right angled triangle. However the measurements of the 3 sides is missing. All we are told is that the perimeter of the triangle is 1000cm.
Penny Nom lui répond.
A thin wire 20 centimeters long is formed into a rectangle 2007-01-29
Trisha pose la question :
a thin wire 20 centimeters long is formed into a rectangle. if the width of this rectangle is 4 centimeters, what is the length?
Penny Nom lui répond.
Height of a right triangle? 2007-01-29
Engelbert pose la question :
The base of the right triangle is 50 ft. At the angle (on the base of the triangle) across from the right angle is labeled portion (a triangle within a triangle), the base of this little triangle is 10 ft. and the height is 6 ft. What equation can i use to solve this?
Steve La Rocque and Haley Ess lui répond.
A right triangle 2007-01-29
Tayler pose la question :
One leg of a right triangle is 8 cm. less than the hypotenuse while the other leg is 1 cm. less than the hypotenuse. Find the length of the hypotenuse.
Penny Nom lui répond.
The height of an isosceles triangle 2007-01-27
Brendan pose la question :
I need to find the height of an isosceles triangle whose angles are 52, 52 and 76 degrees. The base is 100, and the two equal sides are unknown. How would I go about this?
Stephen La Rocque lui répond.
The area of a scalene triangle 2007-01-20
Kalsi pose la question :
I have a scalene triangle in which i have to find the area.
Penny Nom lui répond.
The height of a triangle 2007-01-17
Sharon pose la question :
I am needing to determine the height of a triangle with a 50 ft base and one side is 40 ft the other side being 30 ft.
Penny Nom lui répond.
The height of a triangle 2007-01-14
Shanelle pose la question :
I'm trying to find the height of a triangle but i only know the base and one of the sides as well as the angle between those two sides...but how can i find the height?
Penny Nom lui répond.
In the shadow of a building 2007-01-11
Bill pose la question :
if a building b feet cast a shadow f feet long, then, at the same time of the day, a tree t feet high will cast a shadow, how many feet long?
Stephen La Rocque lui répond.
One leg of a right triangle 2007-01-11
Eric pose la question :
I have a right triangle and the only known leg is one leg adjacent to the right angle. The question is how to find either of the other angles?
Penny Nom lui répond.
Find the area of the rectangle 2007-01-09
Mike pose la question :
The perimeter of a rectangle is 72 in. The base is 3 times the height. find the area of the rectangle?
Penny Nom lui répond.
Are all rectangles trapezoids? 2007-01-05
Sarah pose la question :
Are all rectangles trapezoids?
Chris Fisher lui répond.
Where do the medians of a triangle meet? 2007-01-02
Brittany pose la question :
Where do the medians of a triangle meet?
1. Inside the triangle
v 2. On the traingle
3. Outside the traingle

Stephen La Rocque lui répond.
How do the perimeters of the two pentagons compare? 2007-01-02
Robert pose la question :
If the area of one pentagon is eighty-one times the area of another pentagon, how do the perimeters of the two pentagons compare?
Penny Nom lui répond.
An octagonal birdhouse 2006-12-30
Verner pose la question :
I am building a octagon birdhouse,what degree would I cut each side of each piece of wood to assemble the birdhouse?
Penny Nom lui répond.
The area of a triangle 2006-12-29
Paula pose la question :
The area of a triangle is 24cm². Its height is 2cm more than its base. Determine the base and the height.
Penny Nom lui répond.
An isosceles triangle 2006-12-29
Katrina pose la question :
Is there any way to calculate the height or the length of the equal sides of an isosceles triangle given only the base length and the angles?
Penny Nom lui répond.
A quuadratic equation 2006-12-22
Harold pose la question :
L=3w-4
A=96Square
I used the quadratic equation to solve and my answer is still wrong

I convert the square of 1168 to decimal.

here is where i start
(3w-4) x w=96

could you give me an example

Stephen La Rocque lui répond.
The sum of the angles in a triangle 2006-12-12
Becky pose la question :
Why does a triangle always equal 180 degrees? We know that all the angles add up to 180 degrees but cant find a valid reason for it.
Penny Nom lui répond.
The height of a triangle 2006-12-01
Jeni pose la question :
I need to know how to figure out the height of a triangle when all I know are the sides and base. The sides are 5 and the base is 8 What do I do?
Penny Nom lui répond.
A triangle problem 2006-11-29
Ann pose la question :
Triangle FUN has A) 72.8
B) 65.2
C) 72.7
D) 65.3

Chris Fisher lui répond.
Creating a triangle in a circle 2006-11-28
Dirk pose la question :
My daughter has a school project where she must draw a circle and then draw an equilateral triangle inside the circle. She said you have to identify six points on the circle to correctly draw the triangle. How do you accomplish this?
Penny Nom lui répond.
Dimensions of a rectangle 2006-11-22
Terri pose la question :
How can you find the dimensions of a rectangle that has a perimeter of 18 feet?
Stephen La Rocque lui répond.
A right angled triangle 2006-11-22
Siddhartha pose la question :
area of a right angled triangle OAB where OA = 5 and OB = 3
Penny Nom lui répond.
The height of a building 2006-11-19
Sweetie pose la question :
I have to figure out the measure of the water towers antenna on my schools campus using up to two items created by me and a manual. I don't know far away we are going to have to be. I need to have a fool proof way to figure out the distance to the tower. I have an idea using trigonometry but its really roughly estimated. Do you have any suggestions?
Stephen La Rocque lui répond.
The name of a shape 2006-11-11
Valentina pose la question :
I have a shape of 4 sides 3 are the same length (2cm each) and the other smaller (1.5cm)
Chris Fisher lui répond.
The dimensions of a rectangle 2006-11-02
Ben pose la question :
A rectangle's length is 8 cm. greater than its width. Its perimeter is 128 cm. Find its dimensions.
Penny Nom lui répond.
The height of a triangle as a function 2006-10-19
Ryan pose la question :
Let 2s denote the length of the side of an equilateral triangle. Express the height of the triangle as a function of s
Penny Nom lui répond.
How do you find the length and width of a rectangle if you know ... 2006-10-15
Anne pose la question :
How do you find the length and width of a rectangle if you know the perimeter of the rectangle is 48 m and the area is 144 m squared?
Stephen La Rocque and Penny Nom lui répond.
A 5x2 rectangle 2006-10-03
Stanley pose la question :
Cut a 5x2 rectangle into 4 pieces, and then put them back into as a square.
Claude Tardif and Penny Nom lui répond.
The hypotenuse 2006-10-02
Ashley pose la question :
How do you find the hypotenuse of a right triangle? I don't understand how to find c.
Stephen La Rocque lui répond.
a _________________ line forms a 90 degree angle with another line 2006-09-24
Lisa pose la question :
a _________________ line forms a 90 degree angle with another line is this to it what fills in the blank? first letter is E third letter is R and has 13 letters
Stephen La Rocque lui répond.
An equilateral triangle has been wedged in between two circles. 2006-09-22
Kim pose la question :

An equilateral triangle has been wedged in between two circles. How does the diameter of the smaller circle compare to the diameter of the larger circle.

image: circle inside of an equilateral triangle touching all sides of the triangle; both the triangle and the circle inside are placed into a larger circle where the triangle vertices all touch the circle


Penny Nom lui répond.
The length of the third side of a 45 degree Isosceles triangle 2006-09-20
Rusty pose la question :
what is the formula to determine the length of the third side of a 45 degree Isosceles triangle?
Penny Nom lui répond.
How large is the property? 2006-09-20
Jahn pose la question :
a triangular lot has 750 feet on one side of a 90-degree angle and 500 feet on the other side of the angle. how large is the property?
Penny Nom lui répond.
A rectangular tank 2006-09-19
Milton pose la question :
What size rectangular tank will I need to hold 841.5 gallons of water. The water level is 3'-0" from the bottom of tank.
Penny Nom lui répond.
The length of 2 sides of a triangle 2006-09-15
Lonnie pose la question :
I need to know how to figure the length of 2 sides of a triangle, as the following example:

The length of the bottom is 12' and the angles are 45, 45 I need to know how long the other 2 sides must be to get an angle of 90 at the top.
Stephen La Rocque lui répond.

The volume of a triangular box 2006-09-12
Irene pose la question :
How do I find the volume of a triangular box?
Penny Nom lui répond.
The area of a triangle 2006-09-01
Anthony pose la question :
I WOULD LIKE TO KNOW THE AREA OF A TRIANGLE THAT IS 42" AT THE BASE WITH EQUAL SIDES OF 29" .
Stephen La Rocque lui répond.
How do i get the height of an isosceles triangle? 2006-08-29
Luis pose la question :
How do i get the height of an isosceles triangle?
Penny Nom lui répond.
How high (in feet) is the mountain? 2006-08-29
Briana pose la question :
A survey team is trying to estimate the height of a mountain above a level plain. From one point on the plain, they observe that the angle of elevation to the top of the mountain is 29 degrees. From a point 2000 feet closer to the mountain along the plain, they find that the angle of elevation is 31 degrees. How high (in feet) is the mountain?
Stephen La Rocque lui répond.
Mini Golf 2006-08-17
Sarah pose la question :
I am a sixth grade teacher in Minnesota. I want to have my students explore mini golf and calculate the reflections and angles so that they can figure out how to hit a hole in one. I know that my daughter had various problems like this in eighth grade geometry, but I can't seem to find any internet activities of the appropriate level.
Stephen La Rocque lui répond.
An angle in a parallelogram 2006-08-13
Sam pose la question :
Parallelogram ABCD has diagonal AC equal in length to side AB. CD is produced to E so that D is between E and C. If angle BAC =30 degrees find the size of angle ADE.
Stephen La Rocque lui répond.
How fast is the water level rising 2006-08-12
Erin pose la question :
Water runs into a conical tank at the rate of 9ft3/min. The tank stands point down and has a height of 10 ft. and a base radius of 5 ft. How fast is the water level rising when the water is 6 ft. deep? (V=1/3 pi r2 h).
Penny Nom lui répond.
A triangle problem 2006-08-06
A student pose la question :
A triangle is such that its medium side is twice as long as its shortest side, and its longest side is three times as long as its shortest side. The perimeter of the triangle is 54 yards. What are the lengths of the three sides?
Penny Nom lui répond.
How do I draw the arc? 2006-07-22
Dani pose la question :
How do I draw the arc (calculate the included angle) when I know only the arc length?
Stephen La Rocque lui répond.
Mini Golf geometry 2006-07-18
Sarah pose la question :
I want to have my students explore mini golf and calculate the reflections and angles so that they can figure out how to hit a hole in one. I know that my daughter had various problems like this in eighth grade geometry, but I can't seem to find any internet activities of the appropriate level. If you can steer me towards any resources, I'd be most grateful.
Natasha Glydon lui répond.
The dimensions of a rectangle 2006-07-09
Lori pose la question :
The length of a rectangle is 1 cm more than 3 times its width. If the area of the rectangle is 61 cm2, find the dimensions of the rectangle to the nearest thousandth.
Penny Nom lui répond.
Isoperimetric quotients 2006-07-07
Jessica pose la question :
How do you work out an algebraic equation for the IQ of an isosceles triangle.
Penny Nom lui répond.
Order of operations 2006-06-29
Jaden pose la question :

(1) A triangle has vertices at (-3,-3) (7,-3) and (-2,5). What is the area of the triangle?

Is there a way of doing this so that you dont have to graph it? If the x axis is horizontal can i say it is the base and the y-axis is horizontal meaning the height. Would I be able to do this...-3 to 7-->9 spaces, -3 to 5-->7 spaces-----9(7)63/2=31.5

(2) A cube has a surface area of 54 square centimeters. What is the volume of the cube in cubic centimeters?


Stephen La Rocque lui répond.
The area of a sector and a triangle 2006-06-23
Howard pose la question :
I thought of the following problem which is similar but much simpler than the tethered goat problem: What is the angle(it is more illustrative in degrees)of arc of a unit circle so that the area between the chord it subtends and the arc length is equal to the area of the triangle with opposite side the subtended chord.
Stephen La Rocque and Penny Nom lui répond.
Designing a garage 2006-06-08
A builder pose la question :
I'm currently designing a garage and came upon this interesting math problem. I've tried using various methods to solve it but have so far been unsuccessful. I've included a picture as its far easier to show you my question than explain it verbally. I realize it could be done by trial and error but i'm looking for a real solution.
Stephen La Rocque and Penny Nom lui répond.
The area of a 30, 60 90 triangle 2006-05-24
Willetta pose la question :
if there is a triangle with a 30, 60, and 90 degree angle and the shortest side is 6cm how do you find the area?
Stephen La Rocque and Penny Nom lui répond.
The area of a triangle 2006-05-21
A student pose la question :
how do you find the area of a triangle?
Penny Nom lui répond.
The interior angles of a right triangle 2006-05-20
Greg pose la question :
I am wondering if there is a way to figure out the interior angles of a right triangle if we know ONLY the side lengths, and the trick is, we CANNOT use arctangent!
Leeanne Boehm and Penny Nom lui répond.
A triangle problem 2006-05-18
Jim pose la question :
Right angle triangle with a hypotenuse of 20 units. Square inside the triangle with sides of 4 units, the square shares two sides with both legs of the triangle, and the corner touches the hypotenuse limiting the triangles size.
Penny Nom lui répond.
What is the measure of the interior angles of a decagon? 2006-05-10
Malissa pose la question :
What is the measure of the interior angles of a decagon? And what is the measure of each interior angle of a regular decagon?
Stephen La Rocque lui répond.
The last slice of pizza 2006-04-27
Lori pose la question :
I have a pizza. The radius is 10 inches long.The pizza was cut into 16 equal slices. When one slice was left, my sister and I both wanted it, so we agreed to cut it in half, but I like the crust more than she does, so we decided to cut it the "other way." In other words, the two pieces would not be symmetrical. The inside piece would contain all topping, and the outer piece would contain some topping and some crust. How far up the radius from the center of the circle will I need to cut so we will both have an equal area of pizza?
Stephen La Rocque lui répond.
Ladder and wall 2006-04-27
Lori pose la question :
A 12-foot ladder is leaning across a fence and is touching a higher wall located 3 feet behind the fence. The ladder makes an angle of 60 degrees with the ground. Find the distance from the base of the ladder to the bottom of the fence.
Stephen La Rocque lui répond.
Finding the supplementary angles 2006-04-22
Kendra pose la question :
Angles ABC and DBA are supplementary. If m
Stephen La Rocque lui répond.
Perimeter of a triangle atop a square 2006-04-13
Mary pose la question :
There is an (irregular) pentagon that is made up of a square and a triangle. The triangle is directly on top of the square. The triangle is a right triangle. One of the angles is 45 degrees. One of the sides of the triangle is 4. What is the perimeter of the pentagon?
Penny Nom lui répond.
Finding the side of a triangle 2006-04-06
Carole pose la question :
okays, well, im having difficulty finding the side of a triangle. It is a right triangle and the information given is that the hypotonous is 24 and the angle adjacent to the 90 degree is 32. Im trying to find X, which is placed on the bottom leg of the triangle, and have no idea how to do it. can you explain to me how to get the answer, please? i'd like to know for future reference, please. thank you.
Stephen La Rocque lui répond.
The height of an isosceles triangle 2006-04-01
Chris pose la question :
how do you work out the height of an isosceles triangle if i know the length of the base but i don't know any angles or the lengths of the sides?
Penny Nom lui répond.
cos(3X) 2006-03-29
Joshua pose la question :
I'm having trouble proving that cos(3X)=cos3X -(cosX)(sin2 X)
Penny Nom lui répond.
The area of a triangle 2006-03-26
Colby pose la question :
the length of one side of a triangle is 2cm less than twice the length of the altitude to that side. the area of the triangle is 30 cm squared. find the length of the altitude.
first of all, what is an altitude? and how do i find it?

Penny Nom lui répond.
Solve the equation cos x = sin 20 where x is acute. 2006-03-26
Elle pose la question :
Solve the equation cos x = sin 20 where x is acute.
Stephen La Rocque lui répond.
Can an equilateral triangle have an obtuse angle? 2006-03-26
Chris pose la question :
Can an equilateral triangle have an obtuse angle?

I'm thinking not, because all sides must be equal, but
does that also imply that all angles are equal?


Stephen La Rocque lui répond.
The volume of water in a cone 2006-03-21
Ghulam pose la question :
A vessel has the shape of an inverted cone.The radius of the top is 8 cm and the height is 20 cm. Water is poured in to a height of x cm.Show that if the volume of the water is V cubic cm,then V=(4/75)pi x3.
Penny Nom lui répond.
Three towns are located at the vertices of an equilateral triangle 2006-03-20
A student pose la question :
three towns are located at the vertices of an equilateral triangle. The towns are 8, 5, and 3 miles, respectively, from a store. How far apart are the towns?
Chris Fisher lui répond.
Rectangles with the same perimeter 2006-03-14
Kristine pose la question :
If two rectangles have the same perimeter, will they also have the same area? Why
Penny Nom lui répond.
Heights and shadows 2006-03-06
Debra pose la question :
A person is 7 ft tall and his shadow is 10 ft tall. using the same info what is the shadow of a person who is 5ft tall
Stephen La Rocque lui répond.
An equilateral triangle and a regular hexagon 2006-02-28
Trevor pose la question :
An equilateral triangle and a regular hexagon have equal length perimeters. What is the ratio of their areas?
Penny Nom and Stephen La Rocque lui répond.
Cutting off the top of a triangle 2006-02-22
Ken pose la question :
I'm a creative type who needs to find how you can determine the horizontal, relative percentages within a triangle. Attached is a pdf showing approximately the
50% line, but I'd like to know how to determine any percentage (horizontally) within a triangle . I know that if I put a horizontal line though the height of the triangle I won't be showing 50% above and 50% below. The bottom part has to be larger (correct me if I'm wrong).

I'm not concerned about height as I am wanting to break down the percentages into 60% at the bottom, 25% in the middle and 15% at the top.


Penny Nom lui répond.
A triangle of 50p pieces 2006-02-22
Stuart pose la question :

Ok, so i am collecting 50p pieces and arranging them on my desk in the shape of a triangle.
eg
50p
50p 50p
50p 50p 50p
50p 50p 50p 50p

I want to work out how much money I'm saving just by knowing how many rows of coins there are. If i can work out how many coins there are just by knowing how many rows I have I can just divide by 2 to find out the amount in dollars.


Penny Nom lui répond.
Translate and magnify a triangle 2006-02-19
A student pose la question :
Triangle CAT has vertices located at C(-2,5), A(-5,1), T(1,1)

Translate triangle CAT 4 units right and magnify the triangle by 3. List the new coordinates and explain the process of computing the new coordinates.

Graph triangle CAT and the new triangle form part A on the same coordinate plane....

Will the area of the new triangle be 3 times as large as the original?? Explain why or why not

Penny Nom lui répond.
The third side of a triangle 2006-02-09
Clayton pose la question :
How do I find the length of the third side of a triangle if a=30m, b=30m and I need to find c?
Steve La Rocque and Penny Nom lui répond.
How do you find the angles in a triangle? 2006-01-27
Keith pose la question :
How do you find the angles in a triangle if you know the lengths of the sides?
Chris Fisher and Penny Nom lui répond.
how can i find the height of a triangle if i have the base and the hypotenuse 2006-01-27
Kelsey pose la question :
how can i find the height of a triangle if i have the base and the hypotenuse
Penny Nom lui répond.
Subdividing a polygon into triangles 2006-01-26
Adam pose la question :
is there an algorithm to divide a regular polygon into N equilateral triangles having the same area (no limit on N), or if not, an algorithm to divide a regular polygon into N triangles having the same shape and size?
Chris Fisher lui répond.
The third side of a triangle 2006-01-25
Bob pose la question :
Is there any way to obtain the third side of a triangle when 2 sides are the same length (2.18 inches). I also need to find the angles.
Penny Nom lui répond.
Right angles 2006-01-25
AshLee pose la question :
I was recently given a challenge in my Algebra class. My teacher wanted to know about a right angle. He said he would give five bonus points to the person that could bring in information. (I know five bonus points may not seem like a lot but in this class, they are.) I looked on this site and I found out why it was called a right triangle, but my teacher want to know where did that theory come from. I was wondering too... not just because of the bonus points.
Penny Nom lui répond.
The three angles in a triangle 2006-01-23
A student pose la question :
the measure of the 2nd angle in a triangle is 4 more than the measure of the 1st angle. the measure of the 3rd angle is eight more than twice the measure of the 1st angle. find the measure of each angle.
Penny Nom lui répond.
The angles in a hexagon 2006-01-22
Linda pose la question :
My problem is in relation to wood and making a six sided object from it. On my saw, there is a place to set the angle to which you wish to cut. I cannot for the life of me, figure this out. I am starting with a piece of plywood (1/4" x 6" x 18") and need to know what the angle degree would be to make each of the sides match perfectly to form a hexagon. Trial and error just is not working. Can someone help me?
Penny Nom lui répond.
The centroid of a triangle 2006-01-18
Andrea pose la question :
I am trying to find the centriod of a triangle. I have been given the three vertices of the triangle: A (-25, -20), B (-5, -40), and C (15, 30). I believe I calculated the correct midpoints: AB(E) (-5, 5), BC(D) (5, -5), and AC(F) (-15, -30). When I graphed these points I came up with the solution (-5, -10), but I can't seem to write the answer out (equations of the lines, etc) correctly. Any help would be greatly appreciated!
Penny Nom lui répond.
Interior and external angles of a polygon 2006-01-17
Anthony pose la question :

In a regular polygon, the ratio of the interior angles to the exterior angles is 3:1.

(a) Find the measure of the interior angle.
(b) How many sides are there


Penny Nom lui répond.
The height of a triangle from the lengths of the sides 2006-01-16
A student pose la question :
How do you figure out the height of a triangle when all you have is the length of the sides of the triangle?
Claude Tardif lui répond.
How do I find the area with only one sides measurement? 2006-01-10
Amy pose la question :
If you have a right triangle with the 90 degree angle located in the bottom left corner and the triangles most sloped line goes from the top left down to the right bottom. And the length of the slope line is 10(radical sign)with the radicand of 2. How do i find the area with only one sides measurement.
Penny Nom lui répond.
The height of a right triangle 2005-12-21
Sanmantha pose la question :
I am trying to solve for the height of a right triangle. The base is .05 mm, and the apex is 0.5 degrees. I vaguely recall from high school that this should be enough information to solve for height, but I can't remember what equation(s) to use.
Penny Nom lui répond.
A rectangle has a length that is 7cm more than twice the width 2005-12-21
Stephanie pose la question :
A rectangle has a length that is 7cm more than twice the width. The area of the rectangle is 60cm squared. Find the dimensions of the rectangle.
Penny Nom lui répond.
The sum of the angels in a triangle 2005-11-25
Rachel pose la question :
how do you prove, without knowing any of the measurements or degrees, that the three angles of a triangle equal 180? what are the steps for proving that?
Penny Nom lui répond.
Find the measure of each angle 2005-11-25
Bev pose la question :
in triangle abc, angle a is four times as large as angle b, angle c measures 20 degrees less than angle b. find the measure of each angle.
Penny Nom lui répond.
An isosceles triangle 2005-11-14
Chris pose la question :
PX and QY are attitudes of acute triangle PQR, and Z is the midpoint of PQ. Can you write a proof that triangle XYZ is isosceles?
Chri Fisher lui répond.
A variable rectangle 2005-11-08
Mussawar pose la question :
find the lengths of the sides of a variable rectangle having area 36 cm2 when its perimeter is minimum i do not want solution of this question. i would like to know what is mean by variable rectangle.and what is difference between rectangle and variable rectangle.also what is mean by when its perimeter is minimum.
Penny Nom lui répond.
The height of a tower 2005-11-08
Vinita pose la question :
Observers at point A and B, who Stand on level ground on opposite sides of a tower, measure the angle of elevation to the top of the tower to be 33 degrees and 49 degrees respectively. Another point C is 120 m from point B, Triangle ABC =67 degrees and BAC = 31 degrees. Find the height of the tower to the nearest metre.
Penny Nom lui répond.
Triangles with integer sides 2005-11-04
Tammy pose la question :
I am trying to find another pair of integer sided isosceles triangles, not the same as the ones listed below, with equal areas. (5,5,8) (5,5,6)
Chri Fisher lui répond.
What is the angle between 2 non-collinear parallel lines? 2005-10-17
Ben pose la question :
What is the angle (if any) between 2 non-collinear parallel lines?
Chris Fisher and Walter Whiteley lui répond.
What is the sum of the measures of the angles of a decagon? 2005-10-15
Dianna pose la question :
What is the sum of the measures of the angles of a decagon?
Penny Nom lui répond.
Constructing figures 2005-09-20
Kim pose la question :
I would like to know how to draw different shapes:
Regular Octagon with sides of length, 1 unit
Equilateral Triangles, with sides of length 1 unit
Regular Hexagons, with sides of length 1 unit
Isoseles Triangles, with hypotenuse of length 1 unit

Penny Nom lui répond.
A 30-60-90 triangle 2005-09-11
Gary pose la question :
I have the length of only 1 side of triangle with angles of 30-60-90 degrees. How can I find the length of the other 2 sides?
Penny Nom lui répond.
The length of a chord 2005-09-08
A student pose la question :
how do you find the length of a chord given the angle and radius of the circle
Penny Nom lui répond.
The area of a triangle 2005-08-26
Martha pose la question :
I have a triangle that with a base of 60' and the two sides of 37'. I know the formula for area is A=1/2 (b*h) but how do I find the height??
Penny Nom lui répond.
Is 360 Really the correct value? 2005-08-15
Jack pose la question :
Considering the circumference of a "Perfect Circle" with a Diameter of 1 meter would be something like 3.14 meters, why do we use the number 360 to represent the number of degrees within that circumference?

Would it not make more sense to express the degrees in reference to the relationship to the diameter as related to pi?

That is, let's just say our "Perfect Circle" has a circumference of 3.14 meters, therefore, what we now consider as due east would change from 90 Degrees to 78.5 Degrees.

Penny Nom lui répond.
A right triangle 2005-05-18
Bill pose la question :
If I know the base and the slanted side of a right triangle, how do I figure out the height?
Penny Nom lui répond.
Angle of incline 2005-05-15
Kyle pose la question :
What is the degree of incline of a 12 foot plank that goes from 10.5 inches on one end to zero inches on the other?
Penny Nom lui répond.
sin x + sin 2x + sin 3x + sin 4x = 0 2005-05-10
Elia pose la question :
I tried many times, but can't get to solve the following question:
sin x + sin 2x + sin 3x + sin 4x = 0

Chris Fisher lui répond.
Some triangle problems 2005-05-06
A student pose la question :
1.use the heron's formula to find area of triangle ABC to nerest tenth

A.a=21 b=28 c=11

B.a=23 b=28 c=12

2.Find the nearest tenth the altitude of the longest side if a=3 b=3 c=5

3.the length of each side of a rhombus is 10 cm the length of one diagonal is 12 cm find the area to the nearest square centermeter

Penny Nom lui répond.
An isosceles triangle...with 2005-04-23
Shannon pose la question :
If given an isosceles triangle...with
Is there any possible way to do this, without knowing a side, if so, please explain in detail.

Chris Fisher lui répond.
cos(2x) = sin(3x) 2005-04-14
A student pose la question :
Show that if x= 18 degrees, then cos2x =sin 3x. HENCE find the exact value of sin 18 degrees, and prove that cos 36 - sin 18 =1/2.
Andrei Volodin, Claude Tardif and Penny Nom lui répond.
The centroid of a triangle 2005-04-07
Maria pose la question :
Q3)find the coordinates of the centroid of triangle ABC i want your help here to solve the 3rd question i got stuck.
Penny Nom lui répond.
A triangle question 2005-02-26
John pose la question :
H=10' 7and3/4",angleX=30 degrees solve for the adjacent side
Penny Nom lui répond.
Each interior angle of a particular polygon is an obtuse angle... 2005-02-22
Victoria pose la question :
Each interior angle of a particular polygon is an obtuse angle which is a whole number of degrees. What is the greatest number of sides the polygon could have?
Walter Whiteley lui répond.
A 6 sided (hexagonal) pyramid 2005-01-22
Steve pose la question :
im trying to make a 6 sided (hexagon) pyramid, from 6 triangles of 12mm plywood, i know all the angles to cut apart from the one one to join all 6 triangles together. Rough measurements are outer edge (A) of each triangle is 13cm's, length of other 2 sides (B&C) of triangle outside to center is 14cm's with a height of the whole thing together about 6cm's.
Chris Fisher and Harley Weston lui répond.
The area of a triangle 2005-01-21
Amy pose la question :
In tirangle ABC, AB=6, BC=9, BC is the angle bisector of angle ABC and M is the midpoint of BD. If the area of ABC is 30, what is the area of ABM? The height comes out greater than the hypotenuse, but the teacher said that there's an answer for it. Can you show me how? Thanks
Chris Fisher lui répond.
Finding the missing side of a triangle 2005-01-20
Jason pose la question :
I found a geometry problem that reads as follows:In triangle ABC,
Penny Nom lui répond.
An isosceles triangle 2005-01-03
Abraham pose la question :
The question is,"Triangle ABC is not isosceles.Prove that if altitude BD were drawn, it would not bisect AC."My question is If an altitude is drawn wouldn\'t that mean automatically its isosceles because, In a triangle the sides opposite congruent angles(in this case the right angles)are congruent? What am I thinking wrong?
Harley Weston lui répond.
A geometry problem 2005-01-01
Alexandra pose la question :
In triangle ABC, b=40, and angle A= 30 degres. What values of BC will give two solution for angle B?
Penny Nom lui répond.
The area of a triangle 2004-12-30
Perry pose la question :
What is the area of a triangle with dimensions 3"X5"X7"? Could you provide the formula?
Penny Nom lui répond.
A Reuleaux triangle 2004-12-22
Bob pose la question :
I am trying to remember the name given to the regular figure constructed from three arcs. The figure is like an equilateral triangle except with arcs for each side.
Chri Fisher lui répond.
A right triangle 2004-11-24
Bruce pose la question :
In triangle ABC.
Angle ABC is 90 degrees.
Side AB measures 34.
Side BC measures 31.
What does side AC measure?

Penny Nom lui répond.
Is a square a rectangle? 2004-11-21
Carol pose la question :
I am a teacher. In an FCAT sixth grade review test, there was a question to the students to draw a square and then they referred to it as a rectangle.

What is the definition that makes a rectangle a square that can be taught to the students without confusing them.

Walter Whiteley lui répond.
Interior and exterior angles of a polygon 2004-11-16
Aaron pose la question :
How do you find the measure of interior and exterior angles of a regular polygon when you are given the number of sides?
Walter Whiteley lui répond.
A 30-60-90 triangle 2004-11-09
Amy pose la question :
How do you find the length of the hypotenuse of a 30-60-90 triangle when the side opposite the 60 degree angle is 6 inches?
Penny Nom lui répond.
Solving triangles 2004-10-30
Allen pose la question :
Solve the following triangles.

Given

1. B = 20 Degrees, a = 25, b = 16
2. A = 35 Degrees, b = 2, c = 3
3. A = 32 Degrees, C = 44, c = 20

Harley Weston lui répond.
sin(3A) 2004-10-20
A student pose la question :
Express sin3A in terms of sinA and cosA.
Penny Nom lui répond.
Two gears 2004-10-14
Lindsay pose la question :
"There are two gears, a small one on the left and a larger one on the right. The gear on the right makes 1 revolution. The gear on the left makes two revolutions. Suppose the gear on the right is turned through an obtuse angle. Will the gear on the left make a full turn?"
Penny Nom lui répond.
The length of a rectangle 2004-09-26
Rebecca pose la question :
My question is each side of a square S is 18 inches long and the area of S is at least 3 times the area of a rectangle that is 9 inches wide. What is the greatest possible length in inches of the rectangle?
Penny Nom lui répond.
Supplementary angles 2004-09-23
Rosemary pose la question :
Is the term supplementary angles only applicable to 2 angles (ie. a pair of angles) or can it be used when talking about 3 or more angles that add to make 180 degrees?
Penny Nom lui répond.
The hypotenuse of a right triangle 2004-09-20
Shannon pose la question :
I am trying to find the hypotenuse of a right triangle with only the length of the opposite side. What is the formula as I don't have the length of the adjacent side? Can I compute it without knowing what the other two angles are?
Penny Nom lui répond.
Fabricating with pipe 2004-09-19
Gil pose la question :
I need to fabricate a 10 section, 4" pipe circle with an inside diameter of 40". I would like to know what angles would apply and how to find them,
Penny Nom lui répond.
Finding the height of a triangle 2004-09-14
A student pose la question :
I have to calculate the height of a triangle. The base is 6 (units)say ab One of the top sides is 4 (units)say ca The other side is 2 (units) say cb I don't know the angles.

How to find the height?

Chris Fisher and Penny Nom lui répond.
Area of a triangle 2004-08-17
Barkie pose la question :
I'm going into tenth grade and I have a triangle who's sides measure 3ft, 4ft, and 5ft. I need to find the area however I don't know the height, or how to get it.
Chris Fisher lui répond.
Constructing a triangle 2004-08-08
KV pose la question :
HOW TO CONSTRUCT A TRIANGLE ABC, WHEN ITS ALTITUDES AD,BE,CF ARE GIVEN
Chris Fisher lui répond.
The law of sines 2004-08-01
Joy pose la question :
How do you solve this? Do you solve this triangle using the law of sines of the law of cosines? (ASA)

A=120DEG. B=40DEG c=35 cm I keep getting different answers.

Penny Nom lui répond.
Water in a cone 2004-07-28
A student pose la question :
A vertically inverted cone( i.e. vertex down) has a radius 7 inches and height 24 inches. Water is filled to one third of its height .Find the ht of water when cone is turned upside down
Penny Nom lui répond.
The third side of a triangle 2004-07-28
Annette pose la question :
How do you find the length of a triangle side if you know two sides?
Penny Nom lui répond.
The tangent of theta 2004-07-10
Jacob pose la question :
P is a point on a unit circle with coordinates(0.6,0.8). Find tan of theta. My book shows me how to do it,"tan of theta=opp./adj.=0.8/0.6=4/3,"and leaves it as that's the answer(4/3).When do we know from a problem to find the angle measure (in this case, the angle measure of theta) and how do we know when to give something like 4/3 without converting it to the angle measure?
Penny Nom lui répond.
The base of a triangle 2004-05-25
Ralph pose la question :
With a 30 degree angle at the top of a triangle and a height of 15 what is the base?
Penny Nom lui répond.
Some trig expressions 2004-05-23
A student pose la question :
Prove:

sin A + sin B = 2sin(A+B/2)cos(A-B/2)

cos A - cos B = -2sin(A+B/2)sin(A-B/2)

cos A + cos B = 2cos(A+B/2)cos(A-B/2)

sin A - sin B = 2cos(A+B/2)sin(A+B/2)

Penny Nom lui répond.
The perimeter of a triangle 2004-05-21
A student pose la question :
In a triangle i have the length of a line and it's opposite angle.how can i calculate perimeter?(the angles are not right)
Penny Nom lui répond.
The dimsenions of a room 2004-05-18
Estella pose la question :
I have a room that is rectangular the area is 159.5 feet, the perimeter is 51 feet what would the dimensions of the room.
Penny Nom lui répond.
Angles of elevation and depression 2004-05-18
Anjum pose la question :
what is the difference between an angle of elevation and angle of depression?
Penny Nom lui répond.
The height of a triangle 2004-04-27
Danielle pose la question :
what is the definition of height of a triangle?
Penny Nom lui répond.
A Fibonacci triangle 2004-04-25
Marcelle pose la question :
Is it possible to construct a triangle with sides that are three consecutive Fibonacci numbers?
Walter Whiteley lui répond.
The sum of angles in a triangle 2004-04-06
A student pose la question :
How can u prove sum of angles in a triangle equal to 180 degrees?
Penny Nom lui répond.
Trisecting an arbitrary angle 2004-04-06
Joe pose la question :
Where can I submit my effort on trisecting an arbitrary angle with only a straightedge and a compass? I can do it but I do not have the smarts to prove it.S
Chris Fisher lui répond.
Maximizing the area 2004-03-27
Petey pose la question :
Please could you tell me why for my coursework (where I have to find the largest area that a fence 1000m long can cover) why I should only test equilateral and isoceles triangles? We were told NOT to do right angled triangles but I was wondering why not?
Penny Nom lui répond.
Area and perimeter 2004-03-12
Sandy pose la question :
The rectange has an area of M square units and a perimeter of 2M units. What is the value of x?

The length of the rectange is 3, while the width of the rectange is x-1.

Penny Nom lui répond.
Sum of the angles in a pentagon 2004-03-11
Ashish pose la question :
What is the sum of all the measures of the angles of a Pentagon
Penny Nom lui répond.
A geometry problem 2004-03-04
Jennifer pose la question :
I need help with this problem: Square ABCD has side length 2. A semicircle with diameter AB is constructed inside the square, and the tangent to the semicircle from C intersects side AD at E. What is the exact length of CE?o
Chris Fisher lui répond.
Napoleon's theorem 2004-02-27
David pose la question :
How do i prove this : For any triangle, if you make 3 equillateral triangles using the sides of the the original triangle, the central points of the 3 tringles another triangle that is equillateral.z
Chris Fisher and Penny Nom lui répond.
The dimensions of a rectangle 2004-02-21
Jane pose la question :
A rectangular label has an area of 176cm2 and a perimeter of 54cm. Find the dimensions of the label.
Penny Nom lui répond.
Sin(3x), cos(3x) and tan(3x) 2004-01-28
Jon pose la question :
What is the identity for cos3x, sin3x, and tan3x? In class, we learned double angel identities and were asked to find out the identity to these three trig functions. If you can help, please do. Also, i know that the cos4x- sin4x is the same as cos2x. Is cos8x-sin8x = cos2x also true? Thank you.s
Chris Fisher lui répond.
Reflex angles 2004-01-09
Sonya pose la question :
My daughter is searching for examples of reflex angles. We already have the hands of a clock but still need another example. Can you help us.
Chris Fisher lui répond.
The angles in a regular polygon 2003-12-21
Ernie pose la question :
If i have a measure of one interior angle of a polygon, how can i find the number of sides it has?
Penny Nom lui répond.
Finding angles 2003-12-02
Jason pose la question :
I AM TRYING TO SOLVE A TRIG PROBLEM AND HAVE FORGOT HOW TO DO IT. WHAT I HAVE IS A RIGHT TRIANGLE WITH SIDE A BEING 14 FEET AND SIDE B BEING 3 FEET, USING PYTHAGOREAMS THEOREM SIDE C SHOULD EQUAL 14.318 FEET ON A RIGHT TRIANGLE BUT I AM TRYING TO REMEMBER HOW TO FIND MY ANGLES OTHER THAN THE ONE THAT IS 90 DEGREES.
Penny Nom lui répond.
The area of a triangle 2003-12-01
A student pose la question :
Find the Area of Triangle ABC
A(-3,2)
B(4,0)
C(0,8)

Penny Nom lui répond.
Area of an equilateral triangle 2003-11-25
Jared pose la question :
Can you help me understand why the area of a equilateral triangle is the square root of 3 divided by 4 times the lenght of the side squared?
Penny Nom lui répond.
An octagon shaped bed frame 2003-11-23
Trish pose la question :
My son and I are making an octagon shaped bed frame. We are going nuts trying to figure out what angle to cut the boards to make an outline of an octagon. It seems that the 8 inside angles of the 8 "corners" are 120*, but what is the angle that the 2x6 wood should be cut so that they will angle together to form the outline of the octagon?
Penny Nom lui répond.
A rectangle on a disk 2003-10-29
Arthur pose la question :
How do I go about solving the following problem: What is the width of the largest rectangle with a length of 16 inches you can cut from a circular piece of cardboard having a 10 inch radius?
Penny Nom lui répond.
Squares in a rectangle 2003-10-21
Raj pose la question :

Draw a rectangle with sides of 3 and 4. Divide the sides into 3 and 4 equal parts respectively. Draw squares joining the points on the sides of the rectangle. You will have 12 small squares inside the 3 x 4 rectangle.

If you draw a diagonal of the rectangle, it will intersect 6 of the the 12 smaller squares.

Similarly, if you have a 4 x 10 rectangle, the diagonal would intersect 12 of the 40 squares inside the rectangle.

Is there an algebric equation that determines the number of squares that will be intersected by the diagonal of a rectangle?


Chris Fisher lui répond.
Points inside a triangle 2003-10-10
Laura pose la question :

is there a way to establish, if a given point at coordinates (x,y) is within an area defined by three other sets of coordinates forming a triangle ABC?

is T(x,y) within triangle A(x,y)B(x,y)C(x,y)


Chris Fisher lui répond.
Two chords 2003-10-07
Lori pose la question :
Chords AB and CD of circle O intersect at E. If AE=4, AB=5, CE=2, Find ED.
Penny Nom lui répond.
A rectangle in a circle 2003-09-27
Abdu pose la question :
A rectangle ABCD is inscribed in a circle. If the length of AB is 5 and length of BC is 12, what is the area of circle outside the rectanlge ABCD?
Penny Nom lui répond.
Area of a rectangular room 2003-09-11
Kelly pose la question :
The area of a rectangular room is 238 square feet. The width is 3 feet less than the length. What are the dimensions?
Penny Nom lui répond.
Water in a cone 2003-08-12
Adrienne pose la question :

Water is poured into a tank in the shape of an inverted right circular cone.ð The height of the tank is 8 m and its radius at the top is 4 m.

a. Draw and label a picture to represent this situation.ð (I know how to do this)

b. Identify all variable quantities. (h = 8m, r = 4m)

c. Find an equation that relates the variable quantities, and reduce the number of variable quantities to two.

I was thinking about the equation V = 1/3 pi r2 h, which is the Volume of a cone, but I am stumped as to how I am supposed to "reduce the number of variable quantities to two." Can you point me in the right direction?


Penny Nom lui répond.
Cutting some wood 2003-07-25
Betty pose la question :
My husband is building a six sided wood circle and would like to know the angle to cut the pieces.
Penny Nom lui répond.
A triangle in the complex plane 2003-07-10
Scott pose la question :
The vertices O and A of an EQUILATERAL triangle OAB in the complex plane are located at the origin and 3 + 3i. Find all possible values for the complex number representing the vertex B. Give the location of B in both polar and cartesian form (to 2.d.p)
Penny Nom lui répond.
Write sin(3x) in terms of sin(x) 2003-05-05
A student pose la question :
Write sin 2x in terms of sin x
Penny Nom lui répond.
The height of an equilateral triangle 2003-04-06
Rosa pose la question :
If Each side of an equilateral triangle is 10 m. What is the height?
Penny Nom lui répond.
A triangle and a circle 2003-03-21
Jynks pose la question :
We need a formula that we can use to figure this out for work. We aren't math wiz's or students. Basically we know 3 points in space of a triangle, we know the length of each side and the length of the line from apex to base line. Each point of the base line ends upon the circumference of a circle. IS three a way to work out the radius of that circle.
Penny Nom lui répond.
An equilateral triangle 2003-03-17
Shirley pose la question :
An equilateral triangle is one in which all three sides are of equal length. If two vertices of an equilateral triangle are (0,4) and (0,0), find the third vertex. How many of these triangles are possible?
Penny Nom lui répond.
Can a square be considered a rectangle? 2003-02-27
Carla pose la question :

Can a square be considered a rectangle? (since opposite sides are same length and parallel)

Would a regular hexagon or octagon be considered a parallelogram since its opposite sides are parallel? or does a parallelogram HAVE to have only 4 sides?


Penny Nom lui répond.
A regular polygon 2003-02-26
Melissa pose la question :
The measure of each interior angle of a regular polygon is eight times that of an exterior angle of the polygon. How many sides does the polygon have?
Penny Nom lui répond.
The area of an isoceles triangle 2003-02-07
A student pose la question :
I have to find the area of an isoceles triangle with one angle side of 30 degrees, and length of base 5. Could you please help me solve this problem?
Penny Nom lui répond.
Triangle perimeter 2003-01-30
An Aunt pose la question :

Hi I'm the aunt of a middle school child. He need help with two problems and I don't understand how to show he how to do them. Can you help me.

  1. How do you find the perimeter for X=4, X=0.7, and X=5/6?

  2. And an acute triangle that bottom is (3X+1) cm, left side is (X+4) cm and the right is (2X+5)cm.

Can you show me not only the answer but also the steps that you used.


Penny Nom lui répond.
Why quadratic? 2003-01-21
Dan pose la question :
Why are equations of the form aX2 + bX + c =0 call quadratic? The quad indicates 4 but the power is 2.
Penny Nom lui répond.
Radians 2003-01-16
Erikson pose la question :
I am a student in the 10th grade and attending advanced math at my high school. I was assign to do a report about the unit circle and the radian. But there seems to be no information available about the history of the radian; who first found out about them, which civilizations used it if any. Well, hopefully you'll assist me in this troubling question. Thank you for your kind consideration.
Penny Nom lui répond.
Collinear Points 2003-01-13
Gary pose la question :

Which of the 4 points are collinear when you construct the following concurrent lines or rays of a triangle?

  1. P(1), the point where the angle bisectors intersect.
  2. P(2), the point where the altitudes (or extensions) intersect (inside or outside of the triangle).
  3. P(3), the point where the medians intersect.
  4. P(4), the point where the perpendicular bisectors (or extensions) of the three sides of a triangle intersect.


This is for my 9-12 high school class in geometry.
My name is Gary


Thanks for your help.
Gary


Harley Weston and Chris Fisher lui répond.
Successive coefficients in Pascal's Triangle 2002-12-27
Quincy pose la question :
There is a formula connecting any (k+1) successive coefficients in the nth row of the Pascal Triangle with a coefficient in the (n+k)th row. Find this formula
Penny Nom and Walter Whiteley lui répond.
The height of a triangle 2002-11-29
Dean pose la question :
Could you please tell me the formular for me to calculate the height of a triangle. I have the angles and side lengths. I am trying to calculate the height of an isosceles triangle, does this make a difference from a normal triangle or is the formular the same.
Penny Nom lui répond.
How would you find the length of the chord? 2002-10-31
A draftsperson pose la question :
If given the length of an arc and the distance from the midpoint of the arc to the midpoint of a chord, how would you find the length of the chord and the radius of the arc. The chords endpoints are the same as the the arcs endpoints.
Penny Nom lui répond.
An isosceles triangle 2002-10-30
Stan pose la question :
What two different base lengths can an isosceles triangle have with sides on both remaining at 13 inches? How do I show this?
Paul Betts and Peny Nom lui répond.
Is a square a rectangle? 2002-10-20
Nona pose la question :
Are squares rectangles? Third grade math teacher told a student this was true. Grandmother needs confirmation on this!
Claude Tardif and Harley Weston lui répond.
A Circle is evenly divided into six equal triangles 2002-09-16
Marilynn pose la question :
A Circle is evenly divided into six equal triangles leaving an area between the outside of the circle and the one side of the triangle. This area is measured as 3.14. What is the length of the radius, one line on the triangle?
Paul Betts lui répond.
The area of a triangle 2002-09-07
Phill pose la question :
How do you find the area of a equilateral and other triangles?
Penny Nom lui répond.
The tangent to a curve and the tangent of an angle 2002-08-26
A teacher pose la question :
Is there a relationship between the tangent of a curve(line touching the curve at one point) and tangent (the trigonometric function)?
Chris Fisher lui répond.
Place six numbers around a triangle 2002-07-19
Monika pose la question :
I need to place six numbers around a triangle, as such that one number is on each corner, and one number on each side making three numbers in one line, adding up to two hundred exactly. The number I have to use are, 40, 50, 60, 70, 80, 90. I have to use each one once, there for, each number cannot be repeated.
Claude Tardif and Chris Fisher lui répond.
An equilateral triangle 2002-06-11
Sarah pose la question :
Hi. My name is Sarah. I'm a secondary student taking a Math 30C course by correspondence. The question has two parts.

The first part is: Draw an equilateral triangle XYZ. Draw the altitude from X to YZ. Choose any point P inside the triangle or on the triangle. Draw perpendiculars from P to the sides of the triangle. The Second part is:

Measure the altitude h and the 3 perpendiculars s, t, and u to the nearest mm. Repeat as many time as is necessary until you can state a generalization concerning h, s, t, and u. If you could help me, it would be greatly appreciated.

Penny Nom lui répond.
Successive coefficients in the nth row of Pascal's Triangle 2002-06-10
Tim pose la question :
There is a formula connecting any (k+1) successive coefficients in the nth row of Pascal's Triangle with a coefficient in the (n+k)th row. find this formula.
Penny Nom lui répond.
Bob swam across a river 2002-05-22
Torri pose la question :
Bob swam across a river 420 ft wide. A strong current carried him 580ft downstream as he swam. Find x, the distance bob actually swam.
Penny Nom lui répond.
Find the angle measures 2002-05-18
Amanda pose la question :
In triangle ABC; the measure of angle A is 20 degrees more than twice the angle B. The measure of angle C Is five times angle B. Find the angle measures.
Penny Nom lui répond.
The law of cosines and obtuse angles 2002-05-09
Bryant pose la question :
The question that I am pondering is that I need to derive the law of cosines for a case in which angle C is an obtuse angle.
Penny Nom lui répond.
Moving a triangle 2002-04-18
A student pose la question :
find the verticles of a triangle after it is translated 2 units to the left and then is reflected across the graph of y=x+2. The original verticles of the triangle are (2,0), (3,2), and (6,2).
Peny Nom lui répond.
Area of a triangle 2002-04-07
Puzzled pose la question :
What is the surface area of a triangle 13 ft x 13 ft x 18 ft?
Penny Nom lui répond.
Two triangles 2002-04-03
Scott pose la question :
Consider 2 triangles: Triangle PMB and Triangle PLA.

Triangle PLA is contained within Triangle PMB.

Side LA is parallel to Side MB.

Point L is located on Side PM. Point A is located on Side PB.

If the ratio PL:LM = 5, then what is PB:PA ??


harley Weston lui répond.
Pascal's Triangle 2002-04-02
Brian pose la question :
It's about (a+b)x. I remember there a triangle with numbers to remember for a faster solution. Can you please teach me?
Penny Nom lui répond.
A triangle in a circle of radius 6 2002-03-26
Marko pose la question :
In a circle of radius 6, a triangle PQR is drawn having QR = 8 and PQ = 10. Determine the length of PR
Chris Fisher lui répond.
The isosceles triangle of smallest area 2002-03-08
Lettie pose la question :
can you find the isosceles triangle of smallest area that circumscribes a circle of radius of one?
Walter Whiteley lui répond.
Two circles inscribed in a rectangle 2002-02-27
Amina pose la question :
Given a rectangle with dimensions L=6, H=5. Two circles are inscribed such that they touch each other(circles are adjacent to each other) and also their circumferences touch 2 sides of the rectangle. One of the circles has radius=4. Find the radius of the other circle.
Penny Nom lui répond.
sin 2x = cos 3x 2002-02-25
Allan pose la question :
solve:

sin 2x = cos 3x

Primary question: how do you handle the cos 3x?


Paul Betts and Chris Fisher lui répond.
A right-angled triangle 2001-12-13
Pamela, Jeff, Kayleigh and Jacob pose la question :
A right-angled triangle has one side that is 8 cm shorter that the hypotenuse. The other side is 1 cm shorter than the hypotenuse. Find the length of the sides.
Penny nom lui répond.
A clap of thunder 2001-11-15
Frustrated Mom pose la question :
While getting a recipe for the Thanksgiving feast. The teacher was talking on the phone with a friend who lives four miles north of her. She saw a flash of lightning through the window: fifteen seconds later, she heard a clap of thunder. Ten seconds after that she heard the thunder over the phone. Where did the lightning strike in relation to the teacher's house. (There are two possible answers. Sound travels about 1/5 mile per second. Some people say it's not good to be on the phone in a thunderstorm).
Claude Tardif and Penny Nom lui répond.
Prove that the medians of triangle PQR meet in a central point G. 2001-11-11
Jessica pose la question :
Use a scalene triancle with one point (0,b) on the y axis, another point (2c, 0) on the x-axis and the last point, (2a,0) on the x-axis. Prove that the medians of triangle PQR meet in a central point G.
Penny Nom lui répond.
A circle and triangle overlap 2001-11-09
Tara pose la question :
A circle and triangle overlap as shown.the area of the circle is three times the area of the triangle.If the common region is removed,then the area of the rest of the circle would be 14 sq cm more than the area of the rest of the triangle.How many sq cm are in the area of the complete triangle.
Penny Nom lui répond.
A circle and a triangle 2001-11-09
Tasha pose la question :
I have a circle that has an equalateral triangle inscribed in it. The tip of the triangle (B) is at the center of the circle with the other corners (A & C) extending to the sides of the circle. I need to know the equation to find the linear length of AC. I also need to find the cordial length of the circle from point C to A.
Penny Nom lui répond.
Third side of Acute Triangle 2001-11-09
Don pose la question :
I gave this problem to my children, 9th and 12th grade and they are not(me too!) sure of the answer. I am trying to determine the length of the third side of an acute triangle. I know two of the lengths but I do not know the angles. Is there a proper formula to use to find the length of the third side? Thank you.
Chris Fisher and Penny Nom lui répond.
Applied geometry 2001-11-02
Jenny pose la question :
Where can I find some hands-on activities for my Applied Geometry classes? I want to do more activities with them that allow us to get out of the classroom. However, I want to use activities that use only inexpensive equipment because I usually buy the equipment myself.
Walter Whiteley lui répond.
Where is the fourth point? 2001-10-24
Mike pose la question :
Four points are placed at random on a piece of paper. Connect the three points of the triangle of the largest area. What is the possibility that the fourth point is in the triangle?
Penny Nom lui répond.
A right angled triangle 2001-10-23
Carissa pose la question :
How do you : Triangle ABC is right-angled at A. Calculate the length of the third side if b = 3 and c = 6, express the answer as a surd in its simplest form.
Harley Weston lui répond.
Area of a quilt 2001-10-14
Jack pose la question :
  1. how would you find the area of the quilt? which is a square

  2. how would you fined the area of each square?

Penny Nom lui répond.
A circle and a triangle 2001-10-04
Christina pose la question :
The points (3,4), (9.-2), and (-3,-2) define a circle and a triangle.
  1. find the areas of the circle and the triangle. Find the difference between their areas.

  2. Find the length of a side of a square with the same area as the triangle.

  3. Find the length of a side of a Square with the same are as the circle.

Penny Nom lui répond.
The triangle inequality 2001-09-27
A student pose la question :
    
           __   __   __ 
Prove that AX + XB = AB for any X on AB.  

Chris fisher lui répond.
An acute scalene triangle 2001-09-24
Kim pose la question :
Hi- I'm on a mission--I'm suppose to have a group of children write up a hypotheses about the altitudes and angle biscetors of the follwing 3 types of triangle--equilateral, isosceles and acute scalene triangle.

Is there such a thing as a acute scalene triangle and why or why not.


Chris Fisher lui répond.
Similar triangles 2001-09-08
Dave pose la question :
I am standing on the bank of a river ( whose banks are parallel here) directly opposite a boathouse, B, on the opposite bank. I walk along the bank of the river past a signpost, S, until I reach a point C distant 60 metres from where I started walking. I then walk away from the bank, at right angles to the bank, until I reach a shady tree at D. Attached to teh tree is a sign stating that this spot is 45 metres from the signpost. C is 36 metres beyond S and B and S are in line from D.

(a) How far did I walk away from the bank of the river??

(b) Calculate the width of the river?


Penny Nom lui répond.
Bisecting angles 2001-08-27
Monica pose la question :
Ray QS is the bisector for angle PQR. Find the measure of angle PQS and PQR if the measure of angle SQR is 52 degrees.
Penny Nom lui répond.
Standard angles 2001-08-05
Nagaraj pose la question :
Why 0o , 30o , 45o , 60o ,and 90o are taken as standard angles in Trigonometry? Why can't we take some other angles as standard angles?
Chris Fisher lui répond.
Radian measure 2001-07-26
Amy pose la question :
i have to find out what is meant by the radian measure of an angle and compare it to the measure of an angle in degrees.
Harley Weston lui répond.
Rhombus 2001-07-16
William pose la question :
Calculate the internal angles of a rhombus given measurments of all four sides only.
Walter Whiteley lui répond.
An inequality involving triangles 2001-06-12
Sandra pose la question :
The triangle inequality guarantees that the sum of the lengths of two sides of a triangle is greater than the length of the third. As a consequence, if x and y are legs of a right triangle, with x less than or equal to y, and z the hypotenuse, then x + y is greater than z, so x is greater than z - y. Under what circumstances will x is greater than 2(z - y) be true?
Chris Fisher and Penny Nom lui répond.
Geometry problems involving triangles 2001-06-07
Sandi pose la question :
Find the radius of the largest circle contained in a right triangle whose legs are 8 and 15 and hypotenuse is 17. If the right triangle has legs a and b and hypotenuse c, find an expression for the radius of the circle.
Penny Nom lui répond.
Election in Angletown 2001-06-07
Zoe pose la question :
At the last school election in Angletown 4620 votes were cast. Candidate Acute received 236 more votes than Candidate Obtuse. Candidate Right received 698 votes more than Candidate Acute. Candidate Straight received 256 votes less than Candidate Right. How many votes did each receive?
Penny Nom lui répond.
The angles in a triangle 2001-05-11
Nikki pose la question :
Find the measure, to the nearest degree, of each angle of a triangle with sides of the given lengths.

26, 35, 40


Penny Nom lui répond.
Triangles and fractions 2001-04-27
Constance pose la question :
My name is Constance and I am thirteen years old (I am a student). The question that I am queering about I don't understand why you do ONE HALF x the base x the width WHEN YOU WANT TO FIND the area of a triangle? My second question is if you multiply one half and 10 together why does it come out as 5?
Penny Nom lui répond.
Constructing an equilateral triangle 2001-04-14
Peggy pose la question :
Please send directions to make an equilateral triangle in plane geometry. I want each student to draw two, cut them out, and place them together to form a Jewish Star.
Penny Nom lui répond.
Squares on a chess board 2001-04-11
Tom pose la question :
It was once claimed that there are 204 squares on an ordinary chessboard (8sq. x 8sq.) Can you justify this claim? "PLEASE" include pictures.

How many rectangles are there on an ordinary chessboard? (8sq. x 8sq.) "PLEASE" include pictures.


Penny Nom lui répond.
The bond angles of a tetrahedral polygon 2001-03-14
Nishi pose la question :
how do i prove (a simply as possible) why the bond angles of a tetrahedral polygon are 109.5 degrees? *i already have two explanations that i don't understand. one is about "theory of dot products" and "vectors" and a hook-like symbol w/ a cosine, and the other has an incomprhensible diagram w/ difficult notation- PLEASE BE SIMPLE! thanks sooo much
Harley Weston lui répond.
The angle of elevation 2001-03-08
Jeffrey pose la question :
At a Certain time, a vertical pole 3m tall cast a 4m shadow. What is the angle of elevation of the sun?
Harley Weston lui répond.
How tall is the tree? 2001-03-02
Ronda pose la question :
a tree's shadow is 42 ft. long. There is a stop sign that is right next to it and it is 18 ft. tall and it's shadow is 12 ft. long. How tall is the tree?
Penny Nom lui répond.
Angles in a polygon 2001-02-17
Joan pose la question :
How many sides does a polygon have if its smallest interior angle is 120 degrees and each sucessive angle is 5 degrees greater than the predecessor?
Penny Nom lui répond.
The sum of the angles in a polygon 2001-01-23
A student pose la question :
What is the sum of the measures (interior angles) in an octagon ... heptagon ... decagon.
Penny Nom lui répond.
The hypotenuse of a right triangle 2001-01-22
Phillipe pose la question :
How do you find the hypotenuse of a right triangle?
Penny Nom lui répond.
A lampost and its shadow 2000-12-24
Laura pose la question :
A lamppost line EC casts a shadow line AC. A 30 cm ruler line DB has been moved from A so that it's shadow falls just within the shadow of the lamppost.
  1. Suppose the length of the ruler's shadow is 42 cm. What is the slope of the imaginary line AE?

  2. Suppose the lamppost's shadow is 15 m. long. How tall is the lamppost?

Penny Nom lui répond.
Is this a right triangle? 2000-12-08
Alicia pose la question :
How would you set-up and answer a problem like these one? Triangle ABC has vertices A(-2,2), B(1,-2), and C (1,2). Use slopes to determine if the triangle is a right triangle.
Penny Nom lui répond.
The aspect ratio of a rectangle 2000-12-04
Ron Delavigne pose la question :
The aspect ratio of this rectangle is 4:3. That is A to B is 3. And B to C is 4. If I know the lenght of A to C is 19 inches, how can I find the length of A to B, and B to C.
Penny Nom lui répond.
Triangles and trigonometry 2000-11-30
Mose pose la question :
If I have a right triangle, and I know the lengths of all three sides, is there a formula that will allow me to determine the measurements of the 2 non right angles?
Harley Weston lui répond.
What are adjacent angles that equal 360 called? 2000-11-22
David pose la question :
I know that supplementary angles add to 180 degrees and that commplementary angles add up to be 90 degrees, but what are adjacent angles that equal 360 degrees called?
Chris Fisher lui répond.
Find the 3D angle 2000-11-18
Jacky pose la question :
Included is the diagram. I am trying to find out the angle of ABC. Is it possible? How?
Penny Nom lui répond.
Length of a shadow 2000-11-01
Jessy pose la question :
A man who is six feet tall is walking away from a street light that is fifteen feet tall. How long is the man's shadow when he is ten feet away from the light?
Penny Nom lui répond.
Connecting to a water line 2000-10-20
Vanja pose la question :
My question is...A house is to be connected to a new water main that runs along the line y=2/3x-1. The connection point at the house has coordinates (2,9), where the units represent metres. What lenght of plastic pipe is needed to connect to the water main at the closest point?
Penny Nom lui répond.
Trigonometry 2000-09-02
david pose la question :
determine the sum of the angles A,B where 0 <= A , B <= 180 (degrees)

sinA + sinB = sqr(3/2) , cosA + cosB = sqr(1/2)


Chris Fisher lui répond.
The angle of rotation 2000-08-03
Jay pose la question :
I have the following information Given.

(X1, Y1) Origional Point
(X2, Y2) Origional Point After a Rotation
(Xa, Xb) Center of Rotation

What formula would I use to figure out the angle the point was rotated?


Chris Fisher and Harley Weston lui répond.
The circumference of a circle 2000-07-30
Not a student pose la question :
An equalateral triangle is enclosed in a circle. The three corners touch the edges of the circle. One side of the triangle is 12. What is the circumference of the circle?
Penny Nom lui répond.
A semi-circle and a triangle 2000-07-28
Ben pose la question :
A semi-circle and an isosceles triangle ABC have the same base AB and the same area. The equal angles in the triangle are BAC and CAB. I have to find the value of each of these angles.
Harley Weston lui répond.
Rectangles and algebra 2000-06-13
Kirstin pose la question :
A rectangle's length is 4 more than twice its width. The area of the rectangle is 336m squared. What is its length?
Paul Betts and Penny Nom lui répond.
Projecting a line segment onto a plane 2000-06-08
Monica pose la question :
What is the measure of the angle determined by a 14 inch segment and its projection into a plane if the length,in inches, of the projection into the plane is 7 inches?
Penny Nom lui répond.
A centroid problem 2000-06-02
Kerstin pose la question :
An isoceles triangle has sides measuring 13 cm, 13 cm, and 10 cm. Find the distance from the centroid to the vertex of a base angle.
Harley Weston lui répond.
Adjacent Angles 2000-05-21
Katherine Keys pose la question :
Can a straight angle be an adjacent angle to another angle?
Chris Fisher lui répond.
The pythagorean theorem 2000-05-19
Lauren Fitzgerald pose la question :
how do you find the length of th hipotnuse( or however you spell that word). i understand you have to add the two sides. but when i do add i always end up with this way off answer. i donot understand at all.
Paul Betts lui répond.
Supplementary angles 2000-05-09
Suzanne pose la question :
We know that: Supplementary angles are two angles whose sum equals 180 degrees and complementary angles are two angles whose sum equals 90 degrees. Are supplementary and complementary angles necessarily adjacent? or can they be non-adjacent?
Chris Fisher lui répond.
sin(7pi/12) 2000-05-04
Kristel pose la question :
What is the exact value of sin 7pi/12?
Chris Fisher and Paul Betts lui répond.
Monica's geometry problem 2000-04-27
Monica pose la question :
Given: ABCD is a square; AX is perpendicular to BY
Prove: Angle 1 is congruent to Angle 3

Chris Fisher lui répond.
The area of a triangle using calculus 2000-04-15
Todd Bowie pose la question :
Hi, I am not a student but am reviewing calculus for an upcoming interview. I would like to know how to derive the area of a triangle using calculus. Thanks!
Patrick Maidorn lui répond.
Magic triangles 2000-04-11
Sandy pose la question :
My tutoring student brought math homework today in the form of a "magic triangle". There are three spaces along each side for missing numbers. The sums of the numbers along each of the 3 sides should be the same. Use the numbers 4 through 9. Don't use any number more than once. The sum of the numbers on each side should be 20. What is the logic behind solving a problem of this kind?
Claude Tardif and Harley Weston lui répond.
The side length ratios of some triangles 2000-04-04
Alexis Lockwood pose la question :
I am doing a project for my Math 30B class regarding the side length ratios of 45-45-90 degree and 30-60-90 degree triangles. I would really appreciate any assistance in answering the following questions, or even direction to an appropriate web site or resource on the matter.
Harley Weston lui répond.
Why a Right angle? 2000-04-03
Joseph Mizerek pose la question :
I was wondering why a 90 degree angle is called a Right angle. I mean why isn't called a left angle.
Haragauri Gupta lui répond.
Reflex angles 2000-03-22
D. Reed pose la question :
What is the name of an angle that exceeds 180 degrees?
Penny Nom lui répond.
An equilateral triangle in a circle 2000-03-11
Michael Setlik pose la question :
An equilateral triangle is drawn within a circle such that all three points of the triangle just touch the inside of the circle. Given the diameter of the cicle as six inches what is the length of the sides of the triangle?
Harley Weston lui répond.
Angle of Intersection of Two Lines 2000-03-02
Veronica Patterson pose la question :
I am having a real hard time trying to figure out this problem. Could you please help me! The homework question says to find the acute angle of intersection between the two lines y=3x+1 and y=(1/2)x-1. (It also says to use the results of a problem I had already figured out.) That problem was to use information from a picture shown that tan(theta sub1-theta sub2)= ((m sub2- m sub1)/(1+(m sub1 * m sub2))). I used the difference identity of tangent to figure out the answer. Any help on this problem would be greatly appreciated.
Harley Weston lui répond.
Proportionality in a triangle 2000-03-01
Courtney Smith pose la question :
I would appreciate assistance with the following problem In triangle ABC,segment MN divides sides(segment)AC and (segment)AB proportionally. If the coordinates are A(3,7),M(0,10) and N(8,22) and if AM:MC = 3:1, find the coordinates of B and C.
Penny Nom lui répond.
Building a pyramid 2000-02-26
Francis X. Hines Jr. pose la question :
I am presently trying to build a pyramid. I can understand that the base has 90 degree angles on the first plane which is the outline of the square that makes up the floor.

As close as I can figure the slope of each wall face is 35 degrees or 35.7 to be exact if I am correct by using 360 as the total of the three interior angles.Now , I run into a compound angle where the corners meet what would be the angle created by the two 35 degree angles that would allow for the 90 degree edge to continue.

Because I'm working in three dimensions I also need to be sure that my math would be correct when I substract 35 from 90 to aquire the angle of the narrow edge as to allow for a 90 degree surface to be present ..to allow for another level to be added with only the base line being shortened. I hope you can understand what it is that I'm asking assistance with.I would greatly appericate your help.


Chris Fisher lui répond.
Triple angle formula 2000-02-23
Sara pose la question :
Can one derive a triple angle formula for sine and cosine? If so, how?
Chris Fisher lui répond.
A triangle and a circle 2000-02-23
wendy pose la question :
If a triangle of base 6 has the same area as a circle of radius 6, what is the altitude of the triangle? I am having trouble with altitude.
Penny Nom lui répond.
Is a square a rectangle? 2000-02-15
Jaireh pose la question :
This is something that aroused a debate in class: A rectangle was defined as a parallelogram with 4 right angles. A square was defined as a parallelogram with 4 congruent sides and 4 right angles.

I need written and conclusive proof that some rectangles can or cannot be squares. I tried insisting that some of them can.. but without proof nobody will listen.


Walter Whiteley lui répond.
Euclidean Math puzzle 2000-01-24
Margaret Matthews pose la question :

(Check out this web-site: Simeon's Triangle Puzzle )

I have tried to figure out how this could be, because everything I know about it tells me it can't be. However, I can't seem to make it NOT work.

Two right angle triangles. They are each cut up into four identical pieces. In the first, all the pieces fit together so that there are NO empty spaces; in the second, presumed to be identical in size to the first, the pieces are slightly rearranged, and now, there IS a space in the triangle.


Patrick Maidorn and Claude Tardif lui répond.
Three dimensional rectangle 2000-01-11
Dennis Murphy pose la question :
I would like to find out the name of a Three dimensional rectangle.
Harley Weston lui répond.
Angles 2000-01-06
Rayna pose la question :
I am doing a presentation report on angles which has to be fun and entertaining as well as educational. I am having problems locating resources on angles that give me ideas of fun entertaining projects. My lecture is to be about 20 minutes long infront of a class of 11th and 12th graders. Please Help if you can.
Claude Tardif and Walter Whiteley lui répond.
Multinomial coefficients 1999-12-03
Suraj Das pose la question :
Is there a formula for the expansion of (a+b+c) to the nth power? Does it have to do with Pascal's triangle?
Penny Nom lui répond.
An area problem 1999-11-29
Vicky Birch pose la question :
The length of a rectangle is three times it's breadth. If the length were 6 meters less, and the breadth were 6 meters more, the rectangle would be a square. Make a mathematical model and solve the resulting equation to find the breadth of the original rectangle. What is the area of the square?
Penny Nom lui répond.
Area of a quadrilateral 1999-11-19
Zane Cram pose la question :
I need the formula to calculate the area of an irregular sided rectangle. Each side has a different measurement or length.
Walter Whiteley lui répond.
Transforming a triangle 1999-11-02
Wilder Maldonado pose la question :
The height, H, of a triangle is increased by a lenth M, How much must be subtracted from the corresponding base,b, so that the area of the new triangle is one-half that of the original triangle.
Penny Nom lui répond.
Isosceles triangles 1999-10-12
Amber pose la question :
In defining the types of triangles, our class was stumped by a question asked by one of the student. Maybe you could help. The definition of an equilateral triangle is a triangle with three congruent sides. The definiton of an isosceles triangle is a triangle with at LEAST two congruent sides. The question is, if an isosceles triangle only requires at Least two of the sides to be congruent, could an equilateral triangle be called an isosceles triangle?
Penny Nom, Walter Whiteley and Chris Fisher lui répond.
Two 12-sided polygons 1999-09-25
Kelly Boulton pose la question :
Two 12-sided polygons are similar. A side of the larger polygon is 3 times as long as the corresponding side of the smaller polygon. wHAT IS the ratio of the area of the larger polygon to the area of the smaller polygon.
Walter Whiteley lui répond.
Degrees and triangles 1999-09-09
Sandra Mills pose la question :
Are there any triangles which are not 180 degrees?

I am also in need of information on the history of degree measure for an angle.
Walter Whiteley lui répond.

A Trigonometry Question 1999-08-28
Diane Simms pose la question :
My question is can the following be factored. I am a teacher who needs the factors to this right away. 2 Sin2X + 2 SinX CosX - 1= 0
Harley Weston lui répond.
Sin 4A 1999-06-22
Ryan Cochrane pose la question :
If sinA = 4/5, and A is a first quadrant angle, find sin4A
Harley Weston lui répond.
Girth 1999-05-26
Carolyn Bulkley pose la question :
I am trying to explain to my son (who is in the 8th grade) how to figure girth. I'm afraid I have just confused him. Is there a simply formula to figure the girth of a box.

for example: I have a box that is 27" L X 22" W X 21" H.
Penny Nom lui répond.

An equilateral triangle on a square 1999-04-26
Ed pose la question :
My Grade 8 class and I were discussing the solution to the following problem:

What is the area of the largest equilateral triangle that can be drawn on a 5 cm square.

We used 5 cm as the base of our triangle and then drew the other two legs of 5 cm each to make the equilateral triangle. We then drew an altitude from the upper vertex to the base of the triangle. Using the law of Pythagoras with side a of 2.5 and side c of 5 we calculated side b to be 4.3 cm (the altitude). Therefore the area of the triangle would be 5 x 4.3 divided by 2 or 10.75 square cm.

The answer key to this resource says I am wrong.

What do you think? Have we interpreted the question incorrectly?
Chris Fisher and Harley Weston lui répond.

A ladder problem 1999-04-22
Michael Blade pose la question :
There is a cube box 3feet x 3feet x 3ft resting against a vertical wall on level ground. Resting against the outside corner of the box is a ladder 10 feet tall, this ladder is of course resting on the ground but also against the outside corner of the box and rests on the wall.

The question- the ladder is divided into two unequal section bounded by the box to the ground and the box to the wall. what are those dimensions?
Penny Nom lui répond.

Area of a triangle from vertex coordinates 1999-04-21
Mark Tyler pose la question :
I'm no schoolkid, but I liked your answers about triangles. You might enjoy a quick look at this, the kids may too.

I was working on a Voronoi dual where I had to calculate the areas of very many triangles expressed as vertex coordinates, so I derived the following very direct formula:

A = abs((x1-x2)*(y1-y3)-(y1-y2)*(x1-x3)) for triangle (x1,y1)(x2,y2)(x3,y3)

I've never seen this in a textbook. Is it original? I doubt it, the proof is only a few lines long.

Regardless, it may be fun for the kids, even if it's not on the curriculum.
Walter Whitley lui répond.

Universal Area 1999-04-08
Karen Richardson pose la question :
I need a formula for area that works for a square, a rectangle, a parallelogram, a trapezoid, and a triangle.
Jack LeSage, Chris Fisher and Harley Weston lui répond.
Angles in Polygons 1999-01-21
Jen pose la question :
  • How do u find the interior angles of a pentagon when you are given 4 of the angles and you need to find the fifth?

  • If you are given the measure of each exterior angle of a regular polygon, how do you figure out how many sides the polygon has?

  • i need all information on polygons and how to find their angles!!!

Jack LeSage lui répond.
The area of a triangle 1998-12-26
Elysia pose la question :
How do you figure the square inches of a triangle?
Harley Weston lui répond.
Two sides and a bisectrix. 1998-11-11
Victor Grinshtein pose la question :
I am looking for someone who can tell me how to construct a triangle by 2 sides and a bisectrix using a compass and a ruler.
Chris Fisher lui répond.
Triangular Numbers 1998-10-30
Matt pose la question :
i would like to know about triangular numbers and it history i would also like to know about the history of prime numbers thank you
Chris Fisher lui répond.
Complementary and Supplementary Angles 1998-10-21
Christina Saunders pose la question :
I am in 9th grade and my math teacher wanted us to find out why complimentary angles are called complimentary and why supplimentary angles are called supplimentary. I have looked everywhere and asked numerous people, but I have yet to find an answer. My math teacher said it had something to do with trigonometry. Do you have an answer for me?
Chris Fisher and Penny Nom lui répond.
Concurrent Lines in a Triangle 1998-08-10
Chris Woolf pose la question :
The question is Name four types of concurrent lines, rays, or segments that are associated with triangles.
Chris Fisher lui répond.
Area and Volume 1998-07-28
James Pulver pose la question :
I am going into 12th grade and am practicing for the SAT II. I have come across a problem that I cannot solve. It states that the front, side , and bottom faces of a retangular solid have areas of 24 square centimeters, 8 square centimeters, and 3 square centimeters, respectively. What is the volume of the solid. I need to know how to solve similar problems so is there a formula to go from area to volume?
Jack LeSage lui répond.
Area of a Triangle. 1998-03-05
Amanda pose la question :
How do you figure out the area of a triangle? You already have the perimeter and height
Penny Nom lui répond.
The origin of angles 1998-02-24
Marc Poulin pose la question :
I'm currently teaching angles to students in grade 10 and I've been asked what's the origin of the terms degrees, radians and gradians.

I know that the radians come from the sexagesimal numerical system of the Babylonians but my kids wanted to know dates and persons who would have brought these terms first.
Harley Weston lui répond.

The sum of the angles of a triangle is 180 degrees 1998-02-19
Quin Liu pose la question :
How do you prove that the sum of the angles of a triangle is 180 degrees? Is there a proof? what is it?
Chris Fisher lui répond.
Area of a triangle. 1998-02-01
Jodi Blucher pose la question :
Is there a formula for the area of an equilateral triangle knowing the length of the sides?
Chris Fisher and Harley Weston lui répond.
Trigonometric functions 1997-12-21
Calvin Cheng pose la question :
My name is Calvin and I have a year 12 question for you to help me with.

From a point S, the angle of elevation of the top of a tower due north of it is 20 degrees. From R, due east of the tower, the angle of elevation is 18 degrees. S and R are 100m apart. Find the height of the tower.
Harley Weston lui répond.

Pythagorean Triples. 1997-12-04
Shameq Sayeed pose la question :
I've got a couple of problems which I hope you'll be able to solve for me.

I'm investigating pythagorean triples, and I have found a trend for the triples themselves, and thus have been able to form a general equation, i.e. a=2x+1, b=2x^2+2x, and c=b+1. Now, I sure this equation works, because I've tried it out and have come up with triples that adhere to a^2 + b^2 = c^2. But I was wondering WHY c=b+1. Is it possible to have c=b+2, and if not why not? THAT is the first problem.
Chris Fisher lui répond.

Shimin's Geometry Problem 1997-12-02
Ong Shimin pose la question :
ABCD is a rectangle. X and Y are the midpoints of BC and CD respectively. W is a point on AB such that AW : WB = 2 : 1. Z is a point on AD such that AZ : ZD = 2 : 1.

WY intersects XZ at O. If the area of triangle WOZ is 84 centimeters squared, find the area of triangle XOY.
Chris Fisher lui répond.

A geometry problem 1997-11-20
Herman pose la question :
When produced, two equal chords AB and CD of a circle meet at P in an angle of 24 degrees. If H is the mid-point of AB and K is the mid-point of CD, calculate the size of angle HKD.
Penny Nom lui répond.
A Geometry Problem 1997-09-18
Rebecca Henry pose la question :
A circle is centered at the vertex of the right angle of an isosceles triangle. The cirlce passes through both trisection points of the hypotenuse of the triangle. If the length of a radius of the circle is 10, find the area of the triangle.
Chris Fisher Harley Weston lui répond.
Finding the Mine 1997-06-23
Billy Law pose la question :
Tom is gold prospector. On his last trip out from town, he headed 35 degree South of West to a lake where he had lunch. The lake was 24 km out of Town. He then headed due East for 35 km before Doubling back on bearing of 15 degree South of west for 20 km to reach his mine. By converting to Cartesian coordinate before doing vector additions do the following:

a) Calculate the position of the mine from town in term of a distance and a direction.
...
Harley Weston lui répond.

The angle between two tangents. 1997-06-09
Felix Ho pose la question :
Two tangents are drawn from the origin to the circle (x)(x)+(y)(y)-4x-6y+9=0. If the angle between the tangents is m, fine the value of tan(m).

P.S. (x)(x)=square x
Harley Weston lui répond.

The Area of a Trapezoid. 1997-05-07
Mary George pose la question :
I am doing the area of trapezoids and mixed polygons and I was wondering if you can help me figure out this problem.

A line segment drawn parallel to a leg of a right triangle divides the other leg into segments of 3cm and 6 cm and the hypotenuse into segments of 5cm and 10 cm. The two figures formed are a triangle and a trapezoid. Find the area of each.

I would appreciate if you would email me back the solution.
Harley Weston lui répond.

A Geometry Problem 1997-04-09
Gina M. Pisco and Rebecca Henry pose la question :
Three segments of 3, 4, and 5 inches long, one from each vertex of an equilateral triangle, meet at an interior point P. How long is the side of the triangle?
Richard McIntosh lui répond.
Names of the Polygons 1997-03-04
Manuel pose la question :
What do you call a 13 sided polygon?
Is there a list of the different names?

Walter Whiteley and Harley Weston lui répond.
Triangles, The Pythagorean Theorem and Pizzas. 1997-02-23
Sherryle Mathis pose la question :
I am a graduating senior presently teaching geometry as part of my student teaching. I will do my CUP on Right Triangles and Pythagorean theorem. I am looking for a fun activity as part of my unit plan.
Walter Whiteley lui répond.
A triangle problem 1996-12-19
S. Johnson pose la question :
Given that Triangle ABC is a right triangle and Circle O is inscribed in it find the radius of Circle O, totally in terms of a, b, and c.
Penny Nom lui répond.
Sides in a Regular Polygon 1996-12-06
Rick Moss pose la question :
If you are given the measure of each interior angle (162 degrees) of a regular polygon. How many sides does the polygon have?
Penny Nom lui répond.
Height of a Hotel 1996-11-07
Irene pose la question :
"Irene" is to determine the # of floors in a hotel 500 feet up the street. Irene is on the 10th floor of an office building and can measure the angle of elevation to the top of the hotel, 57 degrees. Her view of the entire building is obstructed. If the street rises at an angle of 8 degrees from the office building to the hotel and the average distance between floors is 11 feet, how many floors are on the hotel?
Penny Nom lui répond.
Why is a circle divided into 360 degrees? 1996-09-30
Kurtis Kredo pose la question :
I was recently wondering why a circle has been divided in to 360 degrees. When I asked my physics teacher he could not think of an answer. His guess is that it probably has to do with people long ago using the base 6 number system. I have a small inkling that it has to do with easy conversion or usage with radians or grads.
Chris Fisher lui répond.
An application of Pythagoras' theorem 1996-04-09
Mike pose la question :
We'd like to know what practical applications there may be for the Pythagorean theorem.
Penny Nom and Maxine Stinka lui répond.
Area=Perimeter 1996-04-02
Jason Pelowski pose la question :
When will the area and the perimeter of a right triangle be numerically equal?
Penny Nom and Maxine Stinka lui répond.
Pascal's Triangle 1996-02-19
Richard pose la question :
Do you know of any resources that might help us make use of the numeric relationships in Pascal's triangle on a fairly simple basis?
Denis Hanson lui répond.
Euclid's Pythagorean proof 1996-02-14
Sean pose la question :
What is Euclid's proof of Pythagoras' theorem?
Harley Weston lui répond.
 
Page
1/1

 

 


Centrale des maths reçoit une aide financière de l’Université de Regina et de The Pacific Institute for the Mathematical Sciences.

CMS
.

 

accueil centre de ressources accueil Société mathématique du Canada l'Université de Regina PIMS